You are on page 1of 195

CONTENT

ĐỀ THI VÀO 10 CHUYÊN AMSTERDAM


1 Năm học 2018 - 2019………………………………………………………………… 2
2 Năm học 2017-2018 - Môn thi: Tiếng Anh (Điều kiện) …………………………….. 8
3 Năm học 2017-2018………………………………………………………………….. 13
4 Năm học 2016-2017 - Môn thi: Tiếng Anh (Điều kiện) …………………………….. 20
5 Năm học 2016-2017 - Tiếng Anh Chuyên…………………………………………… 25
6 Năm học 2016-2017 - Thi thử Tiếng Anh Chuyên…………………………………… 34
7 Năm học 2015-2016 - Thi Tiếng Anh Chuyên……………………………………….. 40
8 Năm học 2014-2015 - Thi thử Tiếng Anh Chuyên…………………………………… 45
9 Năm học 2014-2015 - Thi thử Tiếng Anh Không Chuyên…………………………… 50
10 Năm học 2013-2014 - Thi thử Tiếng Anh Không Chuyên…………………………… 54
11 Năm học 2013-2014 - Thi thử Tiếng Anh Chuyên…………………………………… 59
12 Năm học 2013-2014 - Tiếng Anh Điều kiện…………………………………………. 62
13 Năm học 2013-2014 - Tiếng Anh Chuyên…………………………………………… 68
14 Năm học 2012-2013 - Tiếng Anh Điều kiện…………………………………………. 74
15 Năm học 2012-2013 - Tiếng Anh Chuyên…………………………………………… 79
16 Năm học 2011-2012 - Tiếng Anh Không Chuyên…………………………………… 83
17 Năm học 2011-2012 - Tiếng Anh Chuyên…………………………………………… 88
18 Năm học 2011-2012 - Tiếng Anh Điều kiện…………………………………………. 92
19 Năm học 2010-2011 - Tiếng Anh Không Chuyên…………………………………… 96
20 Năm học 2010-2011 - Tiếng Anh Chuyên…………………………………………… 102
21 Năm học 2010-2011 - Tiếng Anh Chuyên (23-6-2010) …………………………….. 107
22 Năm học 2009-2010 - Tiếng Anh Chuyên…………………………………………… 112
23 Năm học 2009-2010 - Tiếng Anh Chuyên (25-6-2009) …………………………….. 117
24 Năm học 2008-2009 - Đề thi OLYMPIC hệ không chuyên…………………………. 121
25 Năm học 2007-2008 - Kỳ thi tuyển sinh CHU VĂN AN & HÀ NỘI AMSTERDAM 125
26 Năm học 2006-2007 - Kỳ thi tuyển sinh CHU VĂN AN & HÀ NỘI AMSTERDAM 129
27 Năm học 2005-2006 - Kỳ thi tuyển sinh CHU VĂN AN & HÀ NỘI AMSTERDAM 132
28 Năm học 2001-2002 - Kỳ thi tuyển sinh CHU VĂN AN & HÀ NỘI AMSTERDAM 135
TRƯỜNG THPT CHUYÊN NGUYỄN HUỆ
1 Ngày thi: 13 tháng 3 năm 2016 - Tiếng Anh Điều kiện……………………………… 140
2 Ngày thi: ___ tháng 4 năm 2016 - Tiếng Anh Điều kiện…………………………….. 146
3 Ngày thi: 10 tháng 4 năm 2016 - Tiếng Anh Chuyên………………………………… 152
4 Ngày thi: 08 tháng 5 năm 2016 (Đề số 1) - Tiếng Anh Điều kiện……………………. 158
5 Ngày thi: 08 tháng 5 năm 2016 (Đề số 2) - Tiếng Anh Chuyên……………………… 163
ĐÁP ÁN……………………………………………………………………………………. 169

1
SỞ GIÁO DỤC VÀ ĐÀO TẠO KỲ THI VÀO LỚP 10 CHUYÊN ANH
(Đề thi gồm có 06 trang) NĂM 2018 - 2019
MÔN: TIẾNG ANH
Thời gian làm bài: 120 phút (không kể thời gian phát đề)
SECTION I: LISTENING (3.0 points) (You will hear twice for each part)
Part 1. You win hear a man, David, being interviewed about his life as a professional footballer, for each
question, put a tick in the correct box.
1. How long has David been a professional football play?
A. One match B. Two years C. four years
2. What is in a normal day for David?
A. Fitness framing and tactics.
B. fitness training and a full match.
C. Fitness, training tactics and a full match.
3. What do the team not watch videos about?
A. The opposition. B. Warming up. C. Their own performance.
4. What does David say about the diet of a footballer?
A. it is often unpleasant and bad.
B. It has lots of rice, meat and pasta.
C. Footballers have to be careful about what they eat.
5. What is true about David’s free time?
A. He spends most of his free time with his friends
B. He has very little free time, except in the summer.
C. He usually does not manage to see his family.
6. What does David say about his future ambitions?
A. He firstly wants to secure a regular place in the team.
B. He wants to play for a European team in the next two years.
C. He never thinks about playing in the World Cup.
Part 2. You will hear a woman talking on the radio about fours of historic houses in the south of England,
for each question, fill in the missing information in the numbered space.
HISTORIC TOURS
South Elmharn House.
Built: (7)__________century by the bishops of Norwich
Improved: 16th century by a group of rich (8) __________
Features: many old, valuable, wall paintings.
Remains of a small Norman (9) __________
Tours: Including a walk 2 p.m £ 12. tea / coffee, meal bookable.
Haughley Hall
Built: 14th century outside ruined castle
Improved: 18th century.
Features: Once owned by (10) __________II, currently owned by a Lord.
Secret (11) __________in the walls. Two tunnels now closed
Tours 11.30 or 2 p.m £ with traditional (12) __________, £ 12 with tea. Group

2
welcome.
Bedfield House
Built: (13) __________century by the church.
Improved: Mid 15th century.
Features Signs that protect against witchcraft are on (14) __________and surfaces.
Gardens are joins by (15) __________
Tours: 10.30 a.m or 2.30 p.m, £13.50 with tea/coffee and cakes. Groups welcome.

3
SECTION II: LEXICO - GRAMMAR (7.0 points)
Part 1: Questions from 16 to 36. (3.0 points - 0.15/ each). Choose the best answer.
16. I’m expecting to have this book__________soon this year.
A. published B. publishing C. to publish D. publish
17. __________your advice, I was able to avoid lots of trouble on my trip abroad.
A. Thanks to B. Despite C. Even though D. Instead of
18. We have a party tonight and Daisy is worried about__________.
A. what to wear B. which wearing C. that she wears D. these wearing
19. As a rule, new comers__________a party at the end of the first month of their stay.
A. was held B. hold C. have held D. will hold
20. The most interesting films__________for the festival will be shown next week.
A. are chosen B. having chosen C. chosen D. being chosen
21. However__________about the results, the children still talk merrily.
A. disappointed are they B. disappointing they are
C. disappointment D. disappointed they are
22. A child__________to talk does not learn by being corrected all the time.
A. learned B. learns C. learning D. to learn
23. Some businesses use famous cartoon__________to advertise their products.
A. person B. people C. characters D. samples
24. The first thing you should do when you lose your bank card is to call your bank and ask them to close
your__________.
A. account B. house C. loan D. debt
25. “Oh no! I can’t find my credit card!” “__________.”
A. Thank you for letting me know
B. Don’t worry. They will be back very soon
C. It is a nice surprise. You should be glad about it
D. Chill out. Try to remember when you last used it
26. - “What would you like to drink?” - “__________.”
A. Yes, please B. Milk, please C. No, thanks D. OK
27. The number of cars on the roads__________increasing, so we need to build more roads.
A. is B. was C. were D. are
28. __________the class size is our school’s immediate aim.
A. Reduces B. Reduce C. Reducing D. Reduced
29. He__________being given a receipt for the bill he had paid.
A. asked to B. demanded C. insisted on D. required
30. Some__________actions have been taken to help protect wildlife at this national park.
A. survival B. disposal C. postal D. remedial
31. The demand for energy is__________more rapidly than ever.
A. exported B. decreased C. raising D. increasing
32. His performance was__________; the audience was delighted.
A. unmarked B. faultless C. worthless D. imperfect
33. But why did the police suspect you? It just does not make__________to me.
A. reason B. right C. sense D. truth
34. In the end I__________the form in disgust, and threw it away.

4
A. filled in B. worked out C. tore up D. put off
35. His company, __________him with a car and samples of their products.
A. gives B. supplies C. replies D. places
Part 2: Questions from 36 to 45. (2 points - 0.2/ each)
Use the correct form of the word in brackets to complete the following sentences.
36. These high-heeled shoes are__________for such rough terrain. SUIT
37. Colds are caused by viruses, so in the__________of a virus, you can’t catch a cold. ABSENT
38. You’re too young. This film is__________for children your age. APPROPRIATE
39. __________now think that we may have viruses in our bodies already. SCIENCE
40. In cold weather, for example, blood vessels in the nose get smaller to stop heat
escaping__________this also allows the cold virus to attack the nose or throat more easily. FORTUNATE
41. Victoria Falls in Africa is one of the most__________sights in the world.
42. It’s also an__________place for water sports. SPECTACLE
43. We need you to provide an accurate__________of the situation. CREDIBLE
44. On arrival, it’s the noise that makes the greatest__________. DESCRIBE
45. The__________landscape is also well worth a visit. IMPRESS
SURROUND

Part 3: Questions from 46 to 50.(1 point - 0.2/ each)


Each of the following sentences has an error. Find the errors and correct them.
46. Hung really wishes that he can take part in the game.
47. They’re going to get then house decorate for the May Day.
48. I used to sit next to a man who name is Almed.
49. I’d rather typing this letter than write it because it’s faster.
50. He drives too fast that no one likes to ride with him.
Part 4: Questions from 51 to 55.(1 point - 0.2/ each)
Fill in each of the sentences with correct prepositions.
51. It’s about time you told him the truth!
52. The soccer player was ejected because he had done something that was against the rules.
53. Being a nurse is hard work, especially if you're on call all the time.
54. You’ll have to work very hard if you want to catch up with the rest of your classmates.
55. I came across my lost earring while I was sweeping in the floor.
SECTION III: READING (6.0 points)
Part 1. Questions 56 - 65 (2 points - 0.2/ each)
Read the passage below and choose the best answer to each question.
The first question we might ask is: What can you learn in college that will help you in being an employee?
The schools teach a (56) __________many things of value to the future accountant, doctor or electrician. Do
they also teach anything of value to the future employee? Yes, they teach the one thing that it is perhaps most
valuable for the future employee to know. But very few students bother (57) __________ it. This basic is the
skill ability to organize and express ideas in writing and in speaking. This means that your success as an
employee will depend on your ability to communicate, with people and to (58) ______ your own thoughts and
ideas to them so they will (59) __________understand what you are driving at and be persuaded.

5
Of course, skill in expression is not enough (60) __________itself. You must have something to say in the
first place. The effectiveness of your job depends (61) __________your ability to make, other people
understand your work as they do on the quality of the work itself.
Expressing one's thoughts is one skill that the school can (62) __________teach. The foundations for skill in
expression have to be (63) __________early: an interest in and an ear (64) __________language; experience in
organizing ideas and data, in brushing aside the irrelevant, and above all the habit of verbal expression. If you
do not lay these foundations (65) __________your school years, you may never have an opportunity again.
56. A. large B. great C. far D. lots
57. A. learning B. to learn C. with learning D. learn
58. A. interpret B. give out C. transfer D. present
59. A. both B. not C. as well D. either
60. A. on B. for C. by D. in
61. A. on most B. most on C. much on D. on much
62. A. quite B. hardly C. truly D. really
63. A. lied B. laid C. lain D. lay
64. A. by B. in C. for D. of
65. A. during B. of C. for D. when
Part 2. Questions 66 - 75 (2.0 points - 0.2/ each)
Read the passage below and choose the best answer to each question
In 1826, a Frenchman named Niépce needed pictures for his business. He was not a good artist, so he
invented a very simple camera. He put it in a window of his house and took a picture of his yard. That was the
first photograph. The next important date in the history of photography was 1837. That year, Daguerre, another
Frenchman, took a picture of his studio. He used a new kind of camera and a different process. In his pictures,
you could see everything clearly, even the smallest details. This kind of photograph was called a
daguerreotype.
Soon, other people began to use Daguerre’s process. Travelers brought back daguerreotypes from all around
the world. People photographed famous buildings, cities, and mountains.
In about 1840, the process was improved. Then photographers could take pictures of people and moving
things. The process was not simple and photographers had to carry lots of film and processing equipment.
However, this did not stop photographers, especially in the United States. After 1840, daguerreotype artists
were popular in most cities.
Matthew Brady was one well-known American photographer. He took many portraits of famous people. The
portraits were unusual because they were lifelike and full of personality. Brady was also the first person to take
pictures of a war. His 1862 Civil War pictures showed dead soldiers and ruined cities. They made the war
seem more real and more terrible.
In the 1880s, new inventions began to change photography. Photographers could buy film ready-made in
rolls, instead of having to make the film themselves. Also, they did not have to process the film immediately.
They could bring it back to their studios and develop it later. They did not have to carry lots of equipment. And
finally, the invention of the small handheld camera made photography less expensive.
With a small camera, anyone could be a photographer. People began to use cameras just for fun. They took
pictures of their families, friends, and favorite places. They called these pictures “snapshots”.
Documentary photographs became popular in newspapers in the 1890s. Soon magazines and books also
used them. These pictures showed true events and people. They were much more real than drawings.

6
Some people began to think of photography as a form of art. They thought that photography could do more
than show the real world. It could also show ideas and feelings, like other art forms.
66. The first photograph was taken with__________.
A. a small handheld camera B. a very simple camera
C. a daguerreotype D. new types of film
67. Daguerre took a picture of his studio with__________.
A. a new kind of camera B. a very simple camera
C. special equipment D. an electronic camera
68. The word “this” in the passage refers to the__________.
A. carrying of lots of film and processing equipment
B. stopping of photographers from taking photos
C. fact that daguerreotype artists were popular in most cities
D. taking of pictures of people and moving things
69. The word “ruined” in the passage is closest in meaning to “
A. poorly-painted B. heavily-polluted C. terribly spoiled D. badly damaged
70. The word “lifelike” in the passage is closest in meaning to “__________”.
A. moving B. realistic C. touching D. manlike
71. The latest invention mentioned in the passage is the invention of__________.
A. handheld cameras B. processing equipment
C. daguerreotypes D. rolls of film
72. The word “handheld” in the passage is closest in meaning to “__________”.
A. handling manually B. held by hand C. controlling hands D. operated by hand
73. Matthew Brady was well-known for__________.
A. inventing daguerreotypes B. the small handheld camera
C. taking pictures of French cities D. portraits and war photographs
74. As mentioned in the passage, photography can__________.
A. print old pictures B. convey ideas and feelings
C. show the underworld D. replace drawings
75. Which of the following could best serve as the title of the passage?
A. Different Steps in Film Processing B. Story of Photography
C. Photography and Painting D. Story of Famous Photographers
Part 3: Questions 76 - 85 (2.0 points - 0.2/ each)
Read the text and fill in one word which best fits each gap.
SAFE CAMPING
Camping in the country is usually great fun, but sometimes things can go wrong. Accidents can happen, so it
is essential to think about safety both before you go (76) __________while you are there. This will prevent
your fun camping trip turning (77) __________something less pleasant.
Firstly, you need to plan ahead. Check out the weather (78) __________a few days in advance and watch
out for any reports of fires in the area you are thinking of going to. Bring an emergency kit in (79) ______ you
or anyone with you has an accident or illness while you are there.
Choose your camp (80) __________carefully, avoiding any places where there is risk of flooding before you
put up your tent, make (81) __________there are no sharp objects on the ground, or ants’ or wasps’ nests
nearby.

7
In order to keep insects out of the tent, close it whenever you go in or out. If you need a camp (82) ____ for
cooking, be careful not to build it anywhere near your tent, and before you go to bed, remember to put it (83)
__________completely, preferably with lots of water.
After meals, pick up any bits of food that may (84) __________left on the ground, as these can attract
insects - or larger creatures. It also makes senses, for the same reason, to keep unused food in closed containers
away from the camp. You don’t want a hungry bear or another (85) __________suddenly appearing in your
tent!
SECTION IV: WRITING (4.0 points)
Part 1: Questions from 86 to 90. (2 point- 0.4/ each)
Finish each of the following sentences in such a way that it means the same as the sentence printed before
it.
86. The result of the match was never in doubt.
→ At no________________________________________________.
87. I didn’t know you were coming, so I didn’t wait for you.
→ If I__________________________________________________.
88. When I picked up my pen I found that the nib hadbroken
→ On__________________________________________________.
89. Sally finally managed to get a job.
→ Sally finally succeeded__________________________________.
90. Fiona was so disappointed that she could not keep on working.
→ Such________________________________________________.
Part 2: Questions from 91 to 92.(2 points - 0.4/ each)
Complete the second sentence so that it has a similar meaning to the first sentence, using the word given.
Do not change the word given.
91. I haven’t decided to continue my study in a foreign country. MADE
→ I haven’t________________________________________my study in a foreign country.
92. I can’t understand Doris and what she has done. SENSE
→ I can’t________________________________________Doris and what she has done.
93. I think you should complain about that horrible meal. WERE
→ If I______________________________________a complaint about that horrible meal.
94. I have a good relationship with my neighbors. ON
→ My neighbors________________________________________together
95. I don’t usually remember people’s surnames. MEMORY
→ I don’t________________________________________people’s surnames.

8
SỞ GIÁO DỤC VÀ ĐÀO TẠO HÀ NỘI KỲ THI TUYỂN SINH LỚP 10 THPT
NĂM HỌC 2017-2018
ĐỀ CHÍNH THỨC Môn thi: Tiếng Anh (Điều kiện)
Ngày thi: 10 tháng 6 năm 2017
Thời gian làm bài: 120 phút

A. PHONETICS (1.0 pt)


I. Choose the word whose underlined part is pronounced differently from those of the others by circling A,
B, C or D.
1.A. council B. mountain C. country D. compound
2.A. bay B. statue C. ancient D. cave
3. A. bottle B. cotton C. both D. hot
4. A. children B, chorus C. chicken D. check
5. A. hates B. dates C. bakes D. smiles
II. Choose the word which has a different stress pattern from those of the others by circling A, B, C or D.
1. A. government B. property C. temptation D. beautiful
2. A. flexible B. invention C. promise D. musical
3. A. summary B. arrange C. relation D. eliminate
4. A. density B. negative C. organize D. enormous
5. A. religion B. separate C. decorate D. popular
B. VOCABULARY AND GRAMMAR (4.0 pts)
I. Choose the best option (A, B, C or D) to complete each of the following sentences.
1. “Would you mind__________the books back to the library for me” - “Not at all!”
A. taking B. takes C. take D. took
2. He has__________experience in teaching.
A. several B. little C. few D. both
3. When the typhoon happened, the communal house__________totally flooded.
A. is B. are C. was D. were
4. Facebook was built on the__________of earlier social network sites like MySpace and Bebo.
A. succeed B. successful C. successfully D. success
5. Vietnam is one of the biggest__________countries in the region.
A. exportinng rice B. rice export C. rice exportation D. rice-exporting
6. “__________proposal was accepted?” - “It’s John’s”.
A. What B. Who C. Whose D. How
7. Telephone__________by Alexander Graham Bell in 1876.
A. invents B. is invented C. invented D. was invented
8. I enjoy__________to foreign countries to learn more about different cultures and peoples.
A. travelling B. to visit C. to travel D. visiting
9. “I don’t want this holiday”. She said; “I’d rather__________money”.
A. had saved B. save C. will save D. saved
10. These teenagers spent most of their time playing computer games; __________they lost their interests in
daily activities.

9
A. though B. such C. although D. as a result
11. __________if the Great Barrier Reef Marine Park was a World Heritage Site
A. They knew B. She asked C. We said D. You told
12. How many festivals__________in your country every year?
A. celebrate B. celebrated C. celebrating D. are celebrated
13. There are__________flights from Hanoi to HCM city. You can go whenever you like.
A. day B. daily C. much D. every
14. Of the two boys, Harrt is__________
A. smart B. smartest C. the smarter D. the smartest
15. __________Betty, I’m not surprised she won the prize.
A. To know B. Knowing C. To have known D. To be know
II. Give the correct forms of the words in the brackets to complete the passage.
The question “What makes a manager (1.success) __________?” often comes. First of all, a manager needs
the right kind of (2.profession) __________support. In other words, a good manager requires the (3.assist)
__________of a good secretary who needs to have the right kind of (4.secretary) __________ skills. These
include the (5.able) __________to use modern technology, such as computers. There are (6.addition)
__________skills that are just as important. More and more managers need secretaries who can speak several
foreign languages (7.fluent) __________. Success or failure in international (8.negotiate) ________ may
depend on this. This (9.require) __________is often stressed in job advertisements today. Many (10.apply)
__________are rejected because the candidates lack such skills.
III. Complete the passage by writing one suitable preposition into each gap.
Swimming is generally considered to be one of the best ways of exercising the whole body (1) ______ risk
of injury. According (2) __________health expects, it can also ease back pain and even reduce blood
pressure. There is only one problem: to enjoy all the benefits of swimming, you have to do it properly. If you
visited your local swimming pool and just watched, you might be supried to see how badly many people swim.
Poor technique may result (3) _______a number of factors including anxiety, the swimmer’s lack (4)
__________body awareness, or just a concern about geeting their hair wet. What’s more, swimming
techniques do not always improve with practice. (5) __________the contrary, once people have fallen (6)
__________bad habits, they tend to be stuck with them.
But help is (7) __________hand. For all those keen to learn to swim properly, and so that advantage of all
the health benefits, there is now something known as the Shaw Method, developed (8) __________a former
competitive swimmer, Steven Shaw. Shaw encourages people to think (9) ________their swimming technique
and to concentrate (10) __________things with breathing correctly and making sure that arm and leg
movements work together to than against each other.
IV. Circle the correct option (A, B, C or D) that is closest meaning to the underline word
1. In spite of her difficult living conditions, she worked extremely hard.
A. quite B. very C. fairly D. excellently
2. Pierre often helped her, and they devoted all their time to working in their laboratory.
A. dedicated B. sent C. used D. offered
3. Which channel do you recommend to someone who likes animals?.
A. suggest B. tell C. speak D. talk
4. She was brought-up in a well-off family, so she can’t understand the problems we are facing.
A. kind B. wealthy C. broke D. poor
5. What makes computer a miraculous device?

10
A. strange B. superhuman C. wonderful D. famous
C. READING (2.5 pts)
I. Read the passage and fill each gap with the suitable word
The technique of telling a story through a sequence of pictures, though associated with modern cartoons,
was (1) __________fact in use about 500 years ago. Some wonderful examples of these early cartoons can
now be seen at an exhibition in London, all of were produced to order for wealthy clients. (2) __________
makes this art form so interesting is that it flourished in one small part of Europe - Flanders, which today is a
region in northern Belgium. Many of the tiny pictures were a little larger than a postage stamp. They (3)
__________painted by hand in books about the size of a modem paperback. The artists, whose skills were
rewarded by high salaries, worked slowly, and the buyers sometimes (4) __________to wait years for the work
to be completed. In the (5) __________of one four-volume example, the buyer waited for (6) ______ than a
decade. While the majority (7) __________these cartoons depict religious stories, other subjects are illustrated,
(8) __________. Episodes from history were popular, and (9) __________were fairy tales. And yet the artists
had a visual style that seems oddly modem. They made full use of perspective and techniques familiar to us
today from films, such as following a long shot (10) __________dramatic close-up.
II. Choose the best option (A, B, C or D) to complete the following passage.
The ancient Greeks spoke of Seven Wonders of the World. Likewise, some engineers (1) __________
this list of the seven wonders of our modern world. The panama Canal, begun in the 188s, wasn’t finished (2)
__________1914. It join the Atlantic and Pacific oceans. Engineers describe the canal as a victory of humans
over geography. Workers dug huge amounts (3) __________land and tamed rivers. For 40 years after it was
completed in 1931, the Empire State Building, in New York City, (4) __________the tallest building in the
world. Amazingly this 102 story building was constructed in just 410 days. The Netherlands North sea
Protection works is (5) __________victory over geography. (6) __________Netherlands, which is below sea
level, was often flooded by the north sea and by rivers. Then in 1972, a dam was built to shut out the sea.
Twenty years later, dams and canals were built to control the rivers.
The Golden Gate Bridge, completed in 1937, was also a challenge for workers: San Francisco Bay has very
(7) __________wind and rough waves . The Golden Gate is still the world tallest bride. It has enough steel to
wrap around the world three times.
At 1,815 feet, the CN Tower,(8) __________Toronto, Canada, is one of the world's tallest free-standing
buildings. Completed in 1976, the tower is used for TV and radio broadcasting. The Itaipú Dam, completed in
1984, goes across the Parana River at the Brazil-Paraguay border. It is the world’s largest hydroelectric plant.
Completed in 1994, the Channel Tunnel joins France and England. The “Channel” is a (9) __________
convenience: Cars, buses, and trucks are (10) __________carried through the tunnel by train.
1. A. came up to B. came up with C. came out with D. came down with
2.A. until B. at C. on D. to
3. A. on B. in C. of D. for
4. A. is B. was C. had been D. is considered
5. A. another B. other C. others D. an
6. A. B. An C. The D. Some
7.A. heavy B. great C. strong D. big
8. A. on B. at C. to D. in
9. A. remark B. remarked C. remarkable D. remarkably
10. A. entirely B. wholly C. all D. whole
III. Read the passage and decide whether the following statements are True (T) or False (F).

11
1. In the past, from the young age, Korean girls were taught to follow their career in society.
2. Most of the first modern schools in Korea were established by Western Christian missionaries.
3. The achievement in the economic growth of Korea has not been made by women.
4. The constitution of the Republic of Korea recognizes equality between men and women in
education, work and public life.
5. The “Equal employment act” passed in 1987 gave women the same chances in hiring and
promotion.
In traditional Korea society’s role were confined to the home. From a young age, women were taught the
virtues of subordination and endurance to prepare for their future roles as wife and mother. Women, in general,
could not participate in society as men did, and their role was limited to household matters.
The situation began to change with the opening of the country to the outside world during the late 19th
century. During this period, modem .schools were introduced, mostly by Western Christian missionaries. Some
of these schools were founded with the specific goal of educating women. These educated women began to
engage in the arts, teaching, religious work, and enlightening other women. Women also took part in the
independence movement against the Japanese occupation, and displayed no less vigor, determination , and
courage than the men.
With the establishment of the Republic of Korea in 1948, women achieved constitutional rights for equal
opportunities to pursue education, work, and public life. There is no doubt that the female labor force
contributed significantly to the rapid economic growth that Korea achieved during the past three decades. An
increasing number of women work in professional fields.
By 2004, among those graduating from elementary school, 99.5 percent of girls continued their education in
middle school. The comparable figures for high school and university was 87.6 percent. In terms of
characteristics of the female labor force, by 2004,22.6 percent of female employees were serving in
professional or managerial positions.
With an increasing number of women entering professional jobs, the government passed the “Equal
Employment Act” in 1987 to prevent discriminatory practices against female workers in regard to hiring and
promotion opportunities.
D. WRITING (2.5 pts)
I. Rewrite the following sentence using the given words so that it has the same meaning as the original one.
1. “We met professor Stephen Marshall yesterday”, said Pater.
→ Peter said that______________________________________.
2. Although he has an American name, he is in fact French.
→ Despite___________________________________________.
3. My mother allowed me to go out even though it was very late.
→ My mother let______________________________________.
4. My head teacher has a good sense of humour, and he usually makes everyone laugh by telling funny stories.
→ My head teacher who___________________________________.
5. We haven’t moved anything since they sent you away.
→ Nothing______________________________________________.
6. It seems to me that her playing has developed amazingly in the past year.
→ Her playing seems______________________________________.
7. They have recycled waste-paper into newsprint.
→ Waste-paper___________________________________________.
8. Alcohol is not good for your health and cigarettes are not, either.

12
→ Neither_______________________________________________.
9. “Have you done this sort of work before?”- she asked me.
→ She wanted to know_____________________________________.
10. I’d prefer you not to stay up late at night, Bob.
→ I’d rather_____________________________________________.
II. Use the glen words to make meaningful sentences. You can make additions or changes if necessary
1. The students/ do/ homework/ five p.m/ yesterday/afternoon.
→ _____________________________________________________.
2. People/ use/ ballpoint 5.pens/ for/ many/ years/
→ _____________________________________________________.
3. Sometimes/people/have/ communication problems/ because of / language barrier.
→ _____________________________________________________.
4. Mr. Parker/ disappointed/ that/his son/ failed/University entrance exam.
→ _____________________________________________________.
5. Would/ mind/ If/ I/ use/ your bike/ a minute?
→ _____________________________________________________.
III. Rewrite the following sentence using the words cue in the bracket (don’t change the word)
1. Joining the summer camp with the student is very interesting. (It)
→ _____________________________________________________.
2. You need to improve your handwriting considerably. (needs)
→ _____________________________________________________.
3. I remember my parents taking me to the Green Park last summer holiday. (taken)
→ _____________________________________________________.
4. You should take the train instead of the bus. (were)
→ _____________________________________________________.
5. Just after solving one problem, I was faced with another. (Hardly)
→ _____________________________________________________.

-The end-

13
SỞ GIÁO DỤC VÀ ĐÀO TẠO KỲ THI TUYỂN SINH LỚP 10 THPT
HÀ NỘI NĂM HOC 2017-2018
Môn thi: Tiếng Anh (Chuyên)
ĐỀ CHÍNH THỨC Ngày thi: 11 tháng 6 năm 2017
Thời gian làm bài: 120 phút

A. LISTENING (2.0 pt)


Part 1: You win hear a part of a talk about the invention of the microwave oven. For question 1-1-10,
complete, the sentence with a word or short phrase. You will listen twice.
THE INVETION OF THE MICROWAVE OVEN
The invention of the microwave oven began when a chocolate peanut bar (1) __________in Percy Spencer’s
pocket.
Spencer had previously invented a method for (2) __________the tubes used in radar equipment.
Spencer’s first experiment involved putting (3) __________near to some radar equipment.
In his next experiment, an egg was put into a kettle and it (4) __________. The first microwave oven was set
up in (5) __________in Boston in 1946.
The first microwave oven got its name as a result of (6) __________at the company.
One problem with the first microwave oven was that (7) __________did not change colour in it.
When a micro wave oven that could be placed on top of a (8) __________was produced, (9) __________
began to (10) __________.
Part 2: You will hear some advice from a British programme adviser twice. As you listen, fill in the gaps
with the relevant words in the notes below.
When you first arrive in Britain you will be given file name and telephone extension number of the (1) ____
who will be administering your programme. It will be helpful if you make a note in your diary of this (2) ____
and also if you make an (3) __________in advance by telephone whenever you want to see your programme
officer.
If your base is to be outside London, you will be given (4) __________about reaching your destination.
Please follow these carefully and, again, keep a note of them in your diary.
Indicate whether the following statements are true or not by writing T (true); F (false) or NG
(not given) in the space.
5. __________If you bring money to Britain, you’d better carry large sums cash on your person.
6. __________Your money will be more secure if you convert it into travellers cheque.
7. __________It’s not safe to deposit your money with hotels or hostels.
8. __________It’s wise to put your purchases in your shopping bag.
9. __________You should never put anything into your own pockets or bag until it has been paid for.
10. _________All shops provide a receipt for you.
B. LEXICOLOGY AND GRAMMAR (3.0 pts)
1. a. Circle the best option A, B, C or D to complete the following sentences.
1. A good dictionary is indispensable__________any language majors.
A. to B. on C. at D. with
2. I expect the new trend will soon__________here.
A. catch up B. catch on C. take on D. identify with
3. I really admire the here of the film. He’s so__________.

14
A. dangerous B. adventurous C. foolhardily D. instinctive
4. My supervisor is angry with me. I haven’t done all the work that I__________last week.
A. should have done B. can have done C. may have done D. have done
5. He was a generous friend, but as a businessman he drove a hard
A. bargain B. affair C. contract D. business
b. Choose the best option (A, B, C or D) to indicate the words CLOSEST in meaning to the underline word
in each of the following sentences.
6. Corn, domesticated by the American Indians, was brought to Europe by Columbus.
A. trained B. cultivated C. implanted D. reared
7. I’m a bit wary of giving people my address when I don’t know them very well.
A. cautious B. notorious C. harmonious D. hilarious
8. I was having such a good time that I was reluctant to leave.
A. constant B. disorderly C. continuing D. hesitant
9. He testified that he had seen the man leaving the building around the time of the murder.
A. examined B. gave evidence C. disapproved D. denied
10. The group was involved in several subversive activities.
A. charitable B. rebellious C. preposterous D. questionable
II. Join one word on the left with one word on the right to make compound nouns. There is one extra in each box.
Food job work Tax stories meter
Fairy parking income Satisfaction poisoning tale

1. The more money you earn, the more__________you have to pay to the government.
2. On my last holiday, I had terrible__________as a consequence of eating some bad fish.
3. Many people are more interested in__________than in earning large amount of moneys.
4. Don’t forget to put some money in the__________, or the traffic wardens will give you a ticket.
5. When I was young, my parents used to tell me__________about witches and giants and other strange
creatures.
III. Give the correct form of the words given to complete the sentences.
1. A male employee working in the post room of a large company in the United Kingdom received a _____ for
wearing jeans to work. (Suspend).
2. The next time you go to the supermarket, don’t forget to buy the biggest bottle of kitchen cleaner you can
to__________your work surfaces. (Infect)
3. She was severely_________by the long illness she had. (Weak)
4. She hadn’t been famous as a__________until her 17th novel was published (novel)
5. Levels of__________were certainly not below average, yet cutting boards and dishcloths were found to
contain bacteria in far greater number than elsewhere in the home. (clean)
IV. Think of one word only which can be used appropriately in all three sentences.
1. __________
- Ever since John broke the window, he’s been in the teacher’s__________books.
- Rumor has it that he quit the country leaving nothing but__________debts behind.
“ Poor Mark! I feel really__________about his being laid off on his birthday.
2. __________
- It is very difficult to fully__________certain pieces of jazz music until you’ve heard them a good few times.
- Although I’m always ready to lend a hand, Marta never really seems to__________the things I do for her.

15
- Given the market for decorative antiques, I’d say that the value of the beautiful vase is set to__________
considerably in the next few years.
3. __________
- The Smiths were pretty quick off the__________to sell when they heard houses price were falling.
- I think that what you said to Rita about her taste in furniture really hit the__________
- Poverty and unemployment have certainly left a__________on this area.
4. __________
- With less than a minute of the football match to go, Phil managed to__________the ball into the back of the
net.
- I think the best course of action would be to jump into the car and__________for Bristol as soon as possible.
- Louise has been asked to__________an investigation into what went wrong on the night of accident
5. __________
- Initially, few companies saw any potential in computers designed for the__________rather than the office
enviroment.
- Although extremely independent and well able to look after themselves, cats are generally classified
as__________animals.
- Over the years, the proportion of foreign stories in this newspaper has declined as people have become more
engrossed in__________issues.
VI. Choose one given verb and-give the correct preposition or adverb to make suitable phrasal verbs to fill in the
gaps below. Remember to use the correct forms of the verbs and each verb once only. There are two extra verbs.
Steal make bear explain keep answer hang

1. We expect that the accused__________his actions in the highest court in the land.
2. If you can__________me a little longer, I’ll try to explain the reasons behind our actions.
3. The naughty boy__________with the last of the sweets at the party last week.
4. I suppose I would spend a lot of time just__________with friends in the town centre.
5. I don’t have much money these days, so its hard__________my music collection,
C. READING (2.5 pts)
I. Choose the best option A, B, C or D to complete the passage.
Less than a third of advertising executives believe their campaigns help to sell products-a survey has
revealed. Instead, they are plagued by self-doubt and security and dream of (1) __________it all up for another
career. In spite of their “whiz-kid” (2) __________, the survey of 600 advertising employees from 500
agencies found the industry weary from (3) __________. Forty percent of women said sex (4) _______ was
rife and 73 percent described advertising as a “terrible career” for working mother because of its
unpredictability and 12-hour days. Although spending on advertising by major companies has increased, only
28 % of advertising employees were prepared to say that they (5) __________believed the campaigns were (6)
__________. Dominic Mills, the editor of the industry’s trade magazine campaign, which (7) ___ the survey,
was shocked by the result. “It is (8) __________to think that so many people in advertising believe their
efforts are falling (9) __________of the mark but this is (10) __________what people privately think” she
said. Between 1990 and 1992 a fifth of people in the advertising industry lost their jobs. Asked what job they
would most like to do instead, the survivors said they would become barristers, writers, actors or artists.
1. A. taking B. giving C. making D. putting
2.A. image B. view C. appearance D. aspect
3. A. overalls B. overwork C. overpay D. overdose

16
4. A. persecution B. operation C. oppression D. discrimination
5. A. genuinely B. authentically C. correctly D. rightfully
6. A. sufficient B. capable C. effective D. proficient
7. A. carried B. conducted C. performed D. executed
8. A. shattering B. smashing C. striking D. staggering
9. A. far B. out C. beyond D. short
10. A. clearly B. clear C. conspicuously D. accurate
II. Give one suitable word to complete the passage
Most people who move to a foreign country or culture may experience a period of time when they feel very
homesick and have a lot of stress and difficulty functioning in the new culture. This feeling is often called
‘culture shock’ and it is important to understand and learn how to cope with culture shock if you are to (1)
__________successfully to your new home’s culture.
First of all, it’s important to know that culture shock is normal. Everyone in a new situation will go through
some form of culture shock, and the extent to which they do is determined by factors such as the difference
between (2) __________, the degree to which someone is anxious to adapt to a new culture and the familiarity
that person has to the new culture. If you go, for example, to a culture that is far different from your own,
you’re likely to experience culture shock more sharply than those who move to a new culture knowing the
language and the behavioural (3) __________of the new culture.
There are four general stages of cultural adjustment, and it is important that you are aware of these stages
and can recognise which stage you are in and when so that you will understand why you feel the (4) _____ you
do and that any difficulties you are experiencing are temporary, a process you are going through rather than a
constant situation.
The first stage is usually referred to as the excitement stage or the ‘honeymoon’ stage. Upon arriving in a
new environment, you’ll be interested in the new culture, everything will seem exciting, everyone will seem
friendly and helpful and you’ll be overwhelmed with impressions. During this stage you are merely soaking up
the new landscape, taking in these impressions passively, and at this stage you have little meaningful
experience of the culture.
But it isn’t (5) __________before the honeymoon stage dissolves into the second stage - sometimes called
the withdrawal stage. The excitement you felt before changes to frustration as you find it difficult to cope with
the problems that arise. It seems that everything is difficult, the language is hard to learn, people are unusual
and unpredictable, friends are hard to (6) __________, and simple things like shopping and going to the bank
are challenges. It is at this stage that you are likely to feel anxious and homesick, and you will probably find
yourself complaining about the new culture or country. This is the stage which is referred to as ‘culture shock’.
Culture shock is only temporary, and at some point, if you are one of those who manage to stick it (7)
__________, you’ll transition into the third stage of cultural adjustment, the ‘recovery’ stage. At this point,
you’ll have a routine, and you’ll feel more confident functioning in the new culture. You’ll start to feel less
isolated as you start to understand and accept the way things are done and the way people behave in your new
environment. Customs and traditions are clearer and easier to understand. At this stage, you’ll deal with new
challenges with humour rather than anxiety.
The last stage is the ‘home’ or ‘stability’ stage - this is the point when people start to feel at home in the new
culture. At this stage, you’ll function well in the new culture, adopt certain features and behaviours from your
new home, and prefer certain aspects of the new culture to your own culture. There is, in a (8) __________, a
fifth stage to this process. If you decide to return home (9) __________a long period in a new culture, you may
experience what is called ‘reverse culture shock’. This means that you may find aspects of your own culture

17
‘foreign’ because you are so used to the new culture that you have spent so long adjusting to. Reverse culture
shock is usually pretty mild - you may notice things about your home culture that you had never noticed
before, and some of the ways people do things may seem odd. Reverse culture shock rarely (10)
__________for very long.
III. Read passage and choose which of the paragraphs A-F to fit into the numbered gaps in the following
magazine article. There-is one extra paragraph which doesn’t fit in any of the gaps.
If you consider yourself to be particularly environmentally friendly, there is a community which may
interest you. Here, in a large multi-home development known as BedZED, you can find architecture which is
truly green. Buildings come with thick windows and walls, which regulate temperature at a comfortable level
throughout the year. The south-facing windows collect heat and light from the sun, as do solar panels fitted onto the
exterior. Not only that, but BedZED is stylish, and every flat comes with a private garden.
1.
As far as countries in the West are concerned, the buildings that people live and work in consume far more energy
than transport, for example. However, architecture need not consume so much energy, nor produce so much in the
way of CO2 emissions. The intelligent design of the housing st BedZED housing demonstrates that buildings can be
made environmentally friendly, without particularly high costs or advanced technology.
2.
And indeed, the BedZED community -which has some 84 homes-is really rather cost-effective because of economies
of scale: the more homes you build, the less you pay proportionately for the materials and construction of each
individual home. This, as well as the other benefits, is why BedZED is receiving more and more attention.
3.
The technology used in the BedZED design could be implemented far more than it currently is, across
different forms of architecture. It is neither a challenge, nor costly to install solar panels, triple-glazed
windows, or to insulate floors and walls better. Indeed, according some estimates, it would be easy to reduce
the energy consumption of most of our buildings by up to 20% if we just used more effective design.
Indeed, according to one researcher from the European Commission in Brussels, who works on energy
efficiency, it would be possible to achieve a great deal simply by using existing technologies. In large part, the
drive for these changes has come from Europe. Here, government are becoming more concerned about the
dangers of relying too much on our current energy and aware of the need to meet energy-reduction goals.
Many governments have given financial incentives for using energy- efficient design in the construction industry,
and have also tightened regulations. Moreover, a European Union directive now requires house builders to present
evidence of how they are meeting energ-efficiency guidelines.
4.
It also seems that governments are becoming more involved in the search and development of environmentally
friendly designs. At one laboratory in California, a team has experimented with architectural designs such as windows
which become darker on sunny days, thereby reducing the amount of heat coming into the home. This would, in
theory, offer significant savings for people who make heavy use of air-conditioning in hot, sunny climates. Interesting
initiatives have been taken eleswhere, too.
5.
We still have not reached a situation in which the general public fully accepts such measures. From a
marketing perspective, it can still be difficult to convince customers that energy efficient products are
worthwhile. However, as energy prices rise, this is sure to change.
A. Government elsewhere in the word are playing their part, too. In the US, the Energy Star programme
provides standards for the energy efficiency of consumer products, from home construction to computers and

18
kitchen appliances. This has resulted in energy-efficient products becoming commonplace, and indeed, an
attractive choice for consumers.
B. Both of these regions still make every possible attempt to meet energy consumption guidelines as
governments around the world attempt to come to grips with the threat of global warming. There seems every
likelihood that this project could lead the way-one can only hope that others will follow.
C. In India, for example, a New Delhi-based non-profit organization has helped to create systems whereby
small villages can use waste products from farming, and convert them into power. And in Sweeden, there is
research into how heat from the ground can be used to provide hot water or heating for homes.
D. Although it might seem like a state-of-the-art paradise for the super-rich, it’s actually an estate of
affordable housing built between 2000 and 2002 in a suburb of London. It can’t be said that the people who
live here are all eco-warriors, but they are part of a growing tendency to find buildings which use less energy.
E.This essentially means finding out how to Increase efficiency In the least complicated manner possible
BedZED, for example, was planned so that even if the homes need more energy, despite their eco-friendly
designs, there is still a power plant based on the site. This plant, which uses waste materials, can meet any
remaining energy demands from residents.
F.Recently, this has been coming from Asia in the form of Indian and Chinese visitors. Also, more zero-enery
communities are under construction elsewhere in the UK, as well as In the USA.
D. WRITING (2.5 pts)
I. Complete each of the following sentences in such a way that it means the same as the sentence before
I. I suppose you didn’t enjoy that party so long, didn’t you?
→ You can’t__________________________________________.
2. It was her lack of confidence that surprised me.
→ What I found_______________________________________.
3. The only reason why she got promotion is that she’s competent.
→ Were it____________________________________________.
4. That rumour about the politician and the construction contract is absolutely false.
→ There is__________________________________________.
5. The passenger said to the taxi driver “Hurry up!” and he added “I must be at the station on time”.
→ The passenger told_________________________________.
II. Rewrite the sentence by using the word given in such a way that it means the same as sentence printed
before it. Do not alter the given word in any way.
1. Attendance at the additional evening lectures is not obligatory for students. (under)
→ ____________________________________________________________________
2. All are eligible for the contest. There is no discrimination of race and sex. (regardless)
→ ____________________________________________________________________
3. They couldn’t deceive customers although they used attractive advertisement. (Whatever)
→ ____________________________________________________________________
4. I don’t care if she doesn’t write to me. (difference)
→ ____________________________________________________________________
5. I don’t know how he is going to accept the fact that he has lost his job. (terms)
→ ____________________________________________________________________
III. Write an essay: Some people say that poor students or students who come from rural areas often find it
difficult to have access to university education. What should be done to solve this problem?. Give reason for
your answer. You should write at least 250 words in this space.

19
----The end----

20
SỞ GIÁO DỤC VÀ ĐÀO TẠO HÀ NỘI KỲ THI TUYỂN SINH LỚP 10 THPT
NĂM HỌC 2016-2017
ĐỀ CHÍNH THỨC Môn thi: Tiếng Anh (Điều kiện)
Ngày thi: 9 tháng 6 năm 2016
Thời gian làm bài: 120 phút

PART A. PHONETICS, GRAMMAR AND VOCABULARY (4.5 pts)


I. Circle word whose underline part is pronounced differently from the rest (0.5 pt)
1. A. worthy B. brother C. weather D. healthy
2. A. provision B. conclusion C. pension D. precision
3. A. escape B. graze C. transit D. chase
4. A. measured B. weighed C. started D. played
5. A. oranges B. changes C. dances D. notes
II. Circle the correct option (A, B, C or D) that is closest meaning to the underline word (0.5pt)
6. He often makes unusual behavior. They think he is odd.
A. dangerous B. friendly C. strange D. humorous
7. Oil is one of the principal sources of energy.
A. most amazing B. most important C. most confusing D. most difficult
8. No one can survive for very long without water.
A. reproduce B. prosper C. exist D. transcend
9. The actress had to raise her voice in order to be audible in the balcony.
A. spoken B. heard C. dramatized D. appreciated
10. Dictators do not tolerate opposition of any kind.
A. understand B. justify C. execute D. accept
III. Circle the best option (A, B, C or D) to complete the following sentences (0.5 pt)
11. Yuck, this milk stinks! I think its gone__________.
A. off B. on C. up D. down
12. I’ve got to stay at home doing the decorating__________going away on holiday.
A. in spite of B. because of C. instead of D. on behalf of
13. It’s no use__________a language if you don’t try to speak it.
A. to learn B. learned C. learning D. learn
14. If you to London tomorrow, call me first.
A. are going B. were going C. had gone D. had been going
15. The simplified edition, __________ is shorter than the original, is easier to read.
A. it B. that C. what D. which
IV. Give the correct form of the words in brackets to compiler the text (1.0 pt)
Have you ever got really caught up in the excitement and emotion of a good action film, and wondered in
amazement how film stars manage to perform (16.danger) __________acts like jumping off buildings or
driving at great speed? Of course, it is only a momentary feeling as it is no secret that the real (17.perform)
__________are almost invariably stunt men or women, who can earn a very good (18.live) __________by
standing in for the stars when necessary. The work is (19. incredible) __________demanding, and before
qualifying for this job they have to undergo a rigorous training programme and (20.proof) __________then
ability in a number of sports including skiing, riding and gymnastics.

21
Naturally, the (21.safe) __________of the stunt performer is of the utmost importance. Much depends on the
performer getting the timing exactly right so everything is planned down to the (22.tiny) __________
detail. In a scene which involves a complicated series of actions, there is no time for (23.care) __________
mistakes. A stunt man or woman often has only one chance of getting things right, (24.1ike) __________ film
stars, who can, if necessary, film a scene (25repeat) __________- until it gains the director’s approval.
V. Complete this letter by writing one suitable preposition in each gap. (1.0pt)
Dear Jodi,
Sorry I haven’t been in touch with you for such a long time, but it’s been a few busy moths. Earlier this year,
I received good news that my great-aunt had given me a cottage. Apart (26) __________seeing her a couple of
times at my parents’ house, I didn’t really know her. When I saw the cottage, I just fell in love with it. It’s
close to a beautiful little village and looks out (27) __________the sea. My great aunt used it as a holiday
home, and I’ve decided to do the same. Unfortunately, it’s been badly looked (28) __________, so I’ve had to
spend most weekends this year sorting the place out.
I knew it was (29) __________need of some work, and at first I thought I could get away with giving it a
quick coat of paint. But I soon realized it was much a bigger job. There were holes in the roof, and the
windows frames were so rotten, some of the panes of glass were in danger of falling out.
I was walking around the village on Saturday, wondering what best to do, when who should I run (30) __
but Barney Adams. Do you remember him from school?. As luck would have it, he now works in the village as
a builder and decorator. We got talking and he said he’d come over and look at the house.
Naturally, I took him up (31) __________his offer. He got really enthusiastic about it. He talked me into
replacing all the windows, and he’s put in new central heating system in place (32) __________the old coal
fires. I’ve had to prevent him (33) __________extending the kitchen which he was keen to do. He’s checked
the roof out and fortunately that doesn’t need replacing. Thanks to Barney, the house is now looking brilliant,
and (34)comparison with other builders, he doesn’t charge very much.
The next project for me is to clean (35) __________the mess in the garden, as it’s completely overgrown. If
you want to come over and help me sometime, feel free. You’ll always be very welcome.
Love
Emily
VI. Give the correct form of the verbs in brackets (1.0pt)
36-37. If you (keep) __________the environment clean, you (live)a healthier life.
38-39. Mr Smith (teach) __________at this school since he (graduate)in 1980.
40. Milk (usually/keep) __________in the fridge.
41-42. (You/ever be) __________to Singapore?
Yes, I (go) __________there with my classmate last year.
43-44-45. Yesterday thieves (break) __________into the house and (steal) __________a lot of fur coats while
the guests (dance) __________.
PART B. READING (3.0 pts)
I. Read the passage and fill each gap with ONE suitable word (1.0 pt)
We all wish we could remember things more easily, but sometimes we can’t remember anything at all. What
(46) __________you do in this situation?. (47) __________one day you woke up and discovered that you had
completely lost your memory. How would you feel exactly?. I had to face up to this recently after I fell off my
bike and hit my head. I woke up in hospital and the doctor asked me what my name was. I thought about it
(48) __________a moment and then said “I wish I (49) __________!”.Then I tried to get up and said, “It’s

22
time I was at college” “I’d (50) __________you didn’t do that”, said the doctor. “If you move too much, you
(51) __________probably fall over. I’m going to give you an injection. I (52) ______
it doesn’t hurt too much!”. “I (53) __________you hadn’t said that,” I replied. “Now, I’m worried!”. Then I
noticed my label on my wrist with my name on it. If I (54) __________not noticed this, I wouldn’t have
known who I was! When my memory came back a few hours later, I realized I had a math test the next day!. If
(55) __________I had lost my memory for a little bit longer!
II. Read the text about Internet security. Some parts of the text have been removed. Complete the text by
choosing letter (A-I) to fit each gap. There is 1 extra letter you don’t need (0.8 pt)
TIPS TO PROTECT YOURSELF WHILE SHOPPINNG ONLINE
TIP 1
Make sure you are using a secure server and browser with industry security standards before you enter credit
card information online. First look at the address in the address bar and (56) __________. Then
look at the bottom of the Internet window and make sure there is a closed padlock.
TIP 2
Use companies you know. Anyone can open (57) __________and close it tomorrow. Here are several things
to research (58) __________with unknown company. (59) __________for the company and compare this with
information from the domain register. Check for a return policy. Check to see if (60) __ Are they s member of
Better Business Bureau Online (BBBO)?
TIP 3
When shopping we all hope that we (61) __________but sometimes there are problems that arise with what
we ordered. Check the company’s return policy online before you order, so you will (62) __________
TIP 4
Keep information about your order. Also (63) __________such as return policies, company information,
specific produce information and warranty information.
Trust your instincts-if it sound too good to be true.. ..it probably is!
A. Print out other information you may need F. Make sure there is contact information
B.check that it starts with “https” G. a well selected password
C. there have been any complaints H. get what we ask for
D. a store online in minutes I. before doing business
E.know what to expect

III. Read the article about environment in Australia. Choose headings for the paragraphs , (64-75) from the expressions
in the box below. There are some extra ones. (1.2 pts)
Coastal erosion Overfishing Invasive species Urban development Nuclear accidents
Deforestation Intensive agriculture Waste treatment Overgrazing Oil spills

64-65. __________
When large numbers of trees are cut down, the salinity of the soil can greatly increase. Salt water draining
from such areas can affect downstream water quality.
66-67. __________
Pasture mismanagement is one of the main pressures on biodiversity. The unsustainable use of grazed
pasture without giving plants time to recover has modified vast areas of grasslands. This kind of continued
defoliation has led to desertification and erosion.

23
68-69. __________
Australia’s fisheries are already close to collapse because of this activity, and the problem is getting worse.
There are two major factors which account for the problem: slow regeneration of marine populations and
depletion of fish stocks by commercial over-exploitation.
70-71. __________
It is estimated that Australia gains around 20 new pests each year. Examples include cane toads, willow
sand, more recently, red fire ants. Historically, feral cats and foxes have been a cause of local extinction sand
reductions in range for native species through a combination of habitat modification and predation. Weeds are
an equally significant pressure on ecosystems, with more than 2,500 species of introduced plants now thriving
in the wild in Australia. They have invaded every part of the landscape.
72-73. __________
Bio-intensive farming is affecting Australia’s coasts and oceans, particularly estuaries and environments
near the shore. Estimates are that each year almost 19,000 tonnes of phosphorus and 141,000 tonnes of
nitrogen are discharged into rivers flowing to the coast.
74-75. __________
Of continuing concern for Australia is population growth along the country’s coastline. The formation of
massive metropolitan centres with intensive population density on Australia’s coasts could displace much
valuable biodiversity and “high-value” agriculture land.
PART C. WRITING (25 pts)
I. Complete the second sentence so that it has similar meaning to the first sentence (1.0 pt)
76. Was it enjoyable at the beach, Joe?
→ Did you_____________________________________________________?
77. Mr. Pike was tired so he didn’t go to the meeting.
→ Because of___________________________________________________.
78. The best solution was thought of by Jane.
→ Jane came up_________________________________________________.
79. The firm gave me a rise after I had worked there for a year.
→ The firm raised_______________________________________________.
80. The assistant I spoke to was very helpful.
→ The assistant whom____________________________________________.
81. I won’t believe you, no matter what you say.
→ Whatever____________________________________________________.
82. As soon as he entered the building, the fire alarm went off.
→ No sooner___________________________________________________
83. You will catch a cold if you don’t keep your feet dry.
→ Unless______________________________________________________
84. It was Nick’s advice that saved me from bankruptcy.
→ Had it_______________________________________________________
85. “Why don’t you buy them this picture as a wedding present, Sarah?” Robin said.
→ Robin suggested________________________________________________
II. Complete the second sentence so that it has similar meaning to the first sentence, using the word in
brackets. Don’t change the word given, (1.0 pt)
86. They are supplying our company with furniture. (supplied)
_______________________________________________________________

24
87. No decision has yet been made. (decided)
_____________________________________________________
88. “Are you interested in singing karaoke, Maria?” Nam asked. (whether)
_____________________________________________________
89. Despite looking everywhere, the kids failed to find the tennis ball.(although)
_____________________________________________________
90. “Don’t sit in front of the computer for too long,” our teacher told us (warned)
_____________________________________________________
91. It’s pity we didn’t do more sport when we were at school.(wish)
_____________________________________________________
92. The children have to stay up late because they will have an examination soon. (so)
_____________________________________________________
93. I think you ought to decide right now. (If)
_____________________________________________________
94. The teacher postponed the theater trip until the summer term.(off)
_____________________________________________________
95. The TV programme was so complicated that none of the children could understand it.(too)
_____________________________________________________
III. One of the four underlined words or phrases (A, B, C or D) is incorrect. Circle the mistake and correct
it. (0.5 pt)
96. Beautiful is in the eye of the beholder.
A B C D
97. They cannot go camping right now because they are taking care of their three-weeks-old baby.
A B C D
98. If it rain this afternoon, we will have to cancel the picnic.
A B C D
99. Are you familiar of the latest scientific developments in biology?
A B C D
100. The children were surprised when the teacher had them to close their books unexpectedly.
A B C D

----The end ----

25
SỞ GIÁO DỤC VÀ ĐÀO TẠO HÀ NỘI KỲ THI TUYỂN SINH LỚP 10 THPT
ĐỀ CHÍNH THỨC NĂM HOC 2016-2017
Môn thi: Tiếng Anh (Chuyên)
Ngày thi: 10 tháng 6 năm 2016
Thời gian làm bài: 120 phút

I. LISTENING (2.0 pts)


a. Listen to the conversation and complete the form below. WRITE NO MORE THAN THREE WORDS
ORA NUMBER for each answer. You will listen twice.
Mintons Car Mart-Custome Enquiry
Example Answer Address: 20, Green Banks
Make: Lida (6) __________ .
Engine size: (1) __________ Hampshire Postcode: GU8 9EW
Model: Max Contact number: (7) (for__________
Type of gear: (2) __________ only) 0798 257643
Prefered color: Light bule CURRENT CAR
FINANCE Make: Conti
Customer wishes to arrange: (3) __________ Model: Name: Lion
Part exchange? Yes Year: 1994
PERSONAL DETAILS Mileage: Maximum 70,000
Name: Wendy (4) __________ Colour: Metallic grey
Title: (5) __________ Condition: (8) __________

b. Listen to 4 talks and circle the best option (A, B, C or D) to complete each of the sentence.
Part A. 1. Mary Anning’s discovery was 8. Rachel Carson’s book made people realise
important becaused it showed that__________ that__________.
A. fossils could teach us about the past A. using pesticides was damaging the ecosystem
B. an animal could become extinct B. birdsong is essential to humans
C. the shape of the coast was constantly changing C. humans cannot control nature
2. Mary Anning’s dog died because__________ 9. These days, more and more people are buying
A. it fell off a cliff __________.
B. a large amount of earth and rocks fell on it A. powerful pesticides
C. it got stuck down a hole while looking for fossils B. organic food
3. Mary Anning didn’t write a famous book C. Rachel Carson’s book
because__________. Part D. 10. The speaker is apologetic because___.
A. she was too busy exploring the cliffs A. she doesn’t know much about John Lennon
B. she did not have access to a formal education B. she gets upset when she thinks about John
C. she believed only men could be scientists Lennon
Part B. 4. Kailash Satyarthi first saw the C. John Lennon is an obvious choice of hero
problems of the child slavery when he was______ 11. The speaker likes__________.
A.6 years old B. 11 years old C.26 years old A. all the Beatles’ songs and music
5. Kailash Satyarthi has saved over__________ B.only the Beatles’ music
children from enforced labour. C. only the Beatles’ songs

26
A.18,000 B. 80,000 C. 800,000 12. It’s amazing that song Imagine
6. Because of their work, two of Kailash A. is still so well loved today
Satyarthi’s __________ B. really changed the way things were
A. friends were injured C. contained so many important themes
B. children were attacked
C. co-workers were murdered
Part C. 7. Rachel Carson first specialised in_____
A. human biology
B.the study of insects
C. marine biology

II. LEXICOLOGY AND GRAMMAR (3.0 pts)


a. Circle the best option (A, B, C or D) to complete the following sentence
1. Don’t worry. Your luggage will be looked at__________.
A. after B. up C. at D. upon
2. Never__________till tomorrow what we can do today.
A. put off B. put over C. put back D. put away
3. It’s raining heavily. Please__________come my umbrella.
A. with B. under C. for D. to
4. In our University, we often take__________between two periods.
A. ten-minute breaks B. a break ten minutes
C. a ten-minutes break D. a ten-minute break
5. Everyone knows about pollution problems, but not many people have__________any solution.
A. looked in B. seen through C. got round to D. come up with
6. The doctor took out of her bag an unusual__________but promised her young patient that it wouldn’t
hurt her.
A. instrument B. method C. control D. utensil
7. He went to a school which__________good manners and self-discipline.
A. blossomed B. planted C. harvested D. cultivated
8. The reason why this game attracts so many youngsters is that__________other video games this one is
far more interesting.
A. comparing to B. in compared with C. on comparison to D. in comparison with
b. Choose the best option (A, B, C or D) that is closest in meaning to the underlined words.
1. The mother and father reached a compromise over the custody of their daughter.
A. argument B. deadlock C. agreement D. controversy
2. The child has always adhered to the teacher’s rules.
A. questioned B. heard C. repeated D. followed.
3. Research into hydrogen technology was facilitated by money from Congress.
A. aided B. terminated C. followed D. preceded
4. He has his own inadequacy to blame for being fired.
A. poverty B. personality C. appearance D. insufficiency
5. The time constraints prevented the students from finishing the test on time.
A. periods B. clocks C. signals D. limits
C. Give the correct form of the words given to complete the passage.

27
It is commonly believed that a break from everyday routine can only do you good. Every summer, you can
spot prospective (1.holiday) __________at airports and waiting for car ferries. They are unmistakable - you
can tell them a mile away by then sun hats and hopeful expressions. For all theft optimism, what often actually
happens can be a rude awakening from the blissful holiday dreams of the rest of the year?. Sunburn,
mosquitoes and (2.foresee) ________expenses can make you think twice about how (3.benefit) _________
getting away from it all really is. The fact is, the likelihood of something going wrong is maximized when you
are abroad and, unfortunately, your ability to deal with crisis and catastrophe is often minimized. This could be
because of language problems, (4.familiar) __________with the culture, or simply a different climate, all of
which make everything seem different and unreal.
So, what is the answer? (5.doubt) __________an annual escape from normal working life is a positive thing.
However, the wisdom of seeking an exotic location is (6. question) __________when you think of all the
things that can go wrong.
d. Complete the sentences by finding one word which fits in all three spaces.
1. There is free__________to all museums on our tour, which might be good news for budget conscious
visitors.
- His resignation was instantly interpreted as an__________of failure. Despite frantic efforts to revive the
company, it recorded losses for the third time in a row.
- Because Liam was completely drunk he was refused__________to the disco and had to go home.
2. Please come and__________me in the tennis match.
- Danny is a bit miserable so we really should go round and try to__________her up.
- A__________went up as the Prince entered the stadium.
3. Mos tourists come to this part of the world to__________in the scenery and soothing sounds of the jungle.
- I know it’s Dave’s fourth whisky but I can assure you he can hold his__________
- May I have a soft__________, please?. I’m driving today.
4. Travelling and meetings are__________and parcel of being businessman.
- Mike decided to__________company with Jeff and set up his own firm when the differences between the two
company became too big.
- Family get-togethers have been an integral of this small community’s life for decades.
5. The society organized an__________day to familiarize the public with its international activities.
- I won’t turn a deaf ear to your ideas. I’m always__________to your suggestions.
- The intermediaries’conciliatory gestures helped to_______the way for further peace talks and negotiations.
e. Match the verb in the box with suitable preposition to make phrasal verbs and then give correct forms of
the phrasal verbs to complete the sentences. There are two extra.
Get come talk pay Down with on into
Cut step hold give Apart in out off
1. Don’t try to__________me__________going to the concert. I can’t afford the time.
2. I opened the pack of the watch to put a new battery in, and the whole thing suddenly__________in my hand.
I hope I can get it repaired.
3. The job itself is well-paid and interesting, but commuting to the City everyday really ________ me_______
4. Their lives changed completely once the loan__________as it meant they could treat themselves to meals
out and weekends away.
5. It took me half an hour to__________at the waiting room while he was busy writing a letter.
6. People often manage to advance their careers by__________in__________the right people and telling
them what they want to hear.

28
III. READINGS (2.5 pts)
a. Read the passage carefully and do the tasks.
British merchants established a trading post in Singapore in the early nineteenth century, and for more than a
century trading interests dominated. However, in 1965 the newly independent island state was cut off from its
hinterland, and so it set about pursuing a survival strategy. The good international communications it already
enjoyed provided a useful base, but it was decided that if Singapore was to secure its economic future, it must
develop its industry. To this end, new institutional structures were needed to facilitate, develop, and control
foreign investment. One of the most important of these was the Economic Development Board (EDB), an arm
of government that developed strategies for attracting investment. Thus from the outset, the Singaporean
government was involved in city promotion.
Towards the end of the twentieth century, the government realised that, due to limits on both the size of the
country’s workforce and its land area, its labour-intensive industries were becoming increasingly
uncompetitive. So an economic committee was established which concluded that Singapore should focus on
developing as a service centre, and seek to attract company headquarters to serve South East Asia, and develop
tourism, banking, and offshore activities. The land required for this service-sector orientation had been
acquired in the early 1970s, when the government realised that it lacked the banking infrastructure for a
modem economy. So a new banking and corporate district, known as the ‘Golden Shoe’, was planned,
incorporating the historic commercial area. This district now houses all the major companies and various
government financial agencies.
Singapore’s current economic strategy is closely linked to land use and development planning. Although it is
already a major city, the current development plan seeks to ensure Singapore’s continued economic growth
through restructuring, to ensure that the facilities needed by future business are planned now. These include
transport and telecommunication infrastructure, land, and environmental quality. A major concern is to avoid
congestion in the central area, and so the latest plan deviates from previous plans by having a strong
decentralisation policy. The plan makes provision for four major regional centres, each serving 800,000
people, but this does not mean that the existing central business district will not also grow. A major extension
planned around Marina Bay draws on examples of other ‘world cities’, especially those with waterside central
areas such as Sydney and San Francisco. The project involves major land reclamation of667 hectares in total.
Part of this has already been developed as a conference and exhibition zone, and the rest will be used for other
facilities. However the need for vitality has been recognised and a mixed zoning approach has been adopted, to
include housing and entertainment.
One of the new features of the current plan is a broader conception of what contributes to economic success.
It encompasses high quality residential provision, a good environment, leisure facilities and exciting city life.
Thus there is more provision for low-density housing, often in waterfront communities linked to beaches and
recreational facilities. However, the lower housing densities will put considerable pressure on the very limited
land available for development, and this creates problems for another of the plan’s aims, which is to stress
environmental quality. More and more of the remaining open area will be developed, and only natural
landscape surviving will be a small zone.
* Complete the summary below using words given in the box. There are some extra:
Deregulation recycling decentralisation enetertainment industry labour Service tourism

When Singapore became an independent, self-sufficient state it decided to build up its 1) __________, and
government organisations were created to support this policy. However, this initial plan met with limited
success due to a shortage of 2) __________and land. It was therefore decided to develop the 3) _________

29
sector of the economy instead. Singapore is now a leading city, but planners are working to ensure that its
economy continues to grow. In contrast to previous policies, there is emphasis on 4) ______ In addition, land
will be recovered to extend the financial district, and provide 5) __________as well as housing. The
government also plans to improve the quality of Singapore’s environment, but due to the shortage of natural
landscapes it will concentrate instead on what it calls beautification.

** Write: TRUE (T) if the statement is true according to the passage; FALSE (F) if the statement is false
according to the passage; NOT GIVEN (NG) if the information is not given in the passage
__________6. After 1965, the Singaporean government switched the focus of the island’s economy.
__________7. Singapore’s four regional centres will eventually be the same size as its central business district.
__________8. Planners have modelled new urban developments on other coastal cities.
__________9. Labor-intensive industries are among the current priorities for Singapore’s city planners.
__________10.The government has enacted new laws to protect Singapore’s old buildings.
b. Some paragraphs A-H have been removed from the passage. Put them back to their correct places.
One night, not so long ago, just as I was drifting off to sleep, the phone rang. It was my 19 -years- old son,
who is studying at University in Edinburgh calling to say that he had broken up with his girlfriend at midnight and
he had been wandering around the city ever since, not knowing what to do. I told him to catch the first train home. He
arrived looking a wreck, but after a good sleep and some home cooking, he began to feel his old self again.
1.
Girls I knew then were fairly open with their mothers, but none of my male contemporaries would ever have
admitted asking their mothers for advice. Despite all our talk about how important it was for men to let down
their defenses and learn how to express their feelings, most of us still secretly felt that any man who depended
on his mother too much was a bit of a mummy’s boy.
2.
But things don't work that way anymore....In a world of short-term contract, downsizing and redundancy...Even the
most promising and ambitious of our children will go through many career highs and lows during their twenties and
whenever they hit those depths, many of them will return to the nest...A typical son will continue to be at least partly
dependent on his mother well past the age of 18.
3.

They are also better able to see through the mask of apparent self-confidence. When the boy was growing
up, he always maintained a fairly invincible front. The boy early imaginary play involved sieges, ambushes
and surprise attacks. He starting point, though was always a danger against which the boy needed to defend
himself and yet the boy used the game to convince himself that “I could PREVAIL..”
4.
In his teens, he used many of these same tricks to keep me at a bay...If I drove him anywhere to meet his
friends, the boy insisted I to drop him off out of sight of where they were waiting. They were girlfriends I have
never met, and phone conversations which were all in rude. But occasionally, a confidential mood would come
over him and he would tell me whatever happened to be on his mind.
5.
“And there is another important change” she adds. “Most of us took pains to reassure our sons that it was
okay for them to show physical affection or cry when they were upset. If our boys are not so anxious now
about showing their emotion, our efforts in this are have not been in vain”. This seems to be backed up by
research, which shows that boys call their mothers on their mobile phones more than anyone else.

30
6.
The mothers at the other end of the line often have correspondingly low expectations. But we try to keep our
anxieties at manageable levels by saying; “Alright, you can stay out until four in the morning, but only if you
call me every hour to let me know you’re ok”. Those of us who add the offer, “And if you need a lift, let me know”,
often regret it. Most phone calls will begin: “I’m at the station. When can you pick me up?”
7.
What I didn’t anticipate was for the same thing to happen with my son. I assumed I would lose him, just like
all the experts said. It maybe that they were wrong all alone that the sons have always confided in their
mothers and just made sure that no one else knew. Have I stumbled on motherhood's best kept secret? Even if I
have, it doesn’t diminish her sense of wonder. It still like getting a present you never expected.
A. These days, however, mother can expect to be relied on almost indefinitely for the type of advice that
call on her experience of the outside world. A generation ago, it was accepted that son would eventually
leave their mothers to join the world of men and work. Mothers put their 18years in and then opened the
door to allow their sons to move into jobs for life.
B. Friends told me that they, too, were getting the same volume of confidences. Celia Pyper says this is
normal behavior for today’s boys: “Our sons will tell us more than their fathers told their mother, because
we have brought them up to do so. Our norm has been to empathize with our children when they tell us
about then actions or feelings, whereas the previous generation tended to be shocked”.
C. Many of my friends are surprised at this reluctance of their 20-something sons to break away. But
according to psychotherapist Celia Pyper, the mother-son intimacy is nothing new.
“Mums have always been easier to talk to”, she observes. “They are cuddlier than their father and sons
realize early on that their mother are more accepting of human frailty.”
D. They are not in any doubt about how to respond to the situation. As one friend said of her rather reticent
son: “My job is to give my son courage”. And whilst we might welcome the chance to see more of our
children, one does have the feeling that there is something anti-natural in all this.
E.“But don’t assume that girls are any tougher than boys”, says Celia Pyper. “Daughters need their mothers
too”. Certainly I know how much my siown daughter needs me. But this continuing mother-daughter bond
is something that I expected.
F.This, alone, may not imply closeness, however, Rob rings his mother from university three times a week,
but says he is careful to edit what he tells her. And although he’ll approach her for advice on practical
issues, when it comes to matters of the heart: “Why would go to my mother or father after the mess they’ve
made of their lives?”-he asked.
G. The next afternoon he told me what had happened. Then he told me more. And more. And ever more. A
moment arrived when I couldn’t help asking myself, should I be hearing all of this? It wasn’t that I was
shocked. He reminded me of myself in my own student years, but with one important difference-1 would
never ever have confided in my parents this way.
H. When, the boy got older and had to ride to school on a bus with other children, all too often there were
situation in which he didn’t: I had to teach the boy how to put new defenses so that his rougher classmate
would not see his weaknesses

c. Read the passage and decide which answer A, B, C or D best complete the following statements
THE OLYMPIC GAMES
In ancient Greece, athletic festivals were very important and had strong religious associations. The
Olympian athletic festival held every four year in honour of Zeus eventually lost its local character, became

31
first a national event, and then, after the rules against foreign competitors had been waived, international. No
one knows exactly how far back the Olympic Games go, but some official records date from 776 B.C.
The Games took place in August on the plain by Mount Olympus Many thousands of spectators gathered
from all parts of Greece, but no married woman was admitted even as a spectator. Slaves, women and
dishonored persons were not allowed to compete. The exact sequence of events is uncertain, but events
included boys' gymnastics, horse-racing, field events such as discus and javelin throwing, and the very
important foot races. There was also boxing and wrestling and special tests of varied ability such as the
pentathlon, the winner of which excelled in running, jumping, discus and javelin throwing and wrestling. The
evening of the third day was devoted to sacrificial offerings to the heroes of the day, and the fourth day, that of
the full moon, was set aside as a holy day.
On the sixth and last day, all the victors were crowned with holy garlands of wild olive from a sacred wood.
So great was the honor that the winner of the foot race gave his name to the year of his victory. Although
Olympic winners received no prize money, they were, in fact, richly rewarded by then state authorities. The
public honor also made the strict discipline of the ten- month training period worthwhile. In spite of the
lengthy training, however runners were known to drop dead from strain at the winning post. How then results
compared with modem standards, we unfortunately have no means of telling.
After an uninterrupted history of almost 1,200 years, the Games were abolished in A.D. 394, the Christian
era, because of their pagan origin. It was over 1,500 years before there was another such international athletics
gathering. The Greek institution was revived in 1896 and the first small meeting took place in Athens. After
the 1908 London Olympics, success was re-established and nations sent theft best representatives. In times of
peace, the Games have taken place ever since at four-yearly intervals. In Munich in 1972, competitors from
more than 120 countries were watched by huge crowds.
Nowadays, the Games are held in different countries in turn. The host country provides vast facilities,
including a stadium, swimming pools and living accommodation, but competing countries pay their own
athletes' expenses. Athletic contests are still the main feature, but now many more sports are represented,
women compete, the ancient pentathlon, for example, has been modified into a more comprehensive test, and
the marathon* races, initiated in 1896, are now a celebrated event.
The Olympics start with the arrival in the stadium of a torch, lighted on Mount Olympus by the sun’s rays. It
is carried by a succession of runners to the stadium. The torch symbolizes the continuation of the ancient
Greek athletic ideals, and it bums throughout the Games until the closing ceremony. The well-known Olympic
flag, however, is a modem conception: the five interlocking rings symbolize the uniting of all five continents
participating in the Games.
1. The order of athletic evens at the ancient Olympics__________
A. has not definitely been established
B. varied according to the full moon.
C. was decided by Zeus, in whose honour the Games were held.
D. was considered unimportant.
2. During the Games, on the evening before the moon was full, __________
A. heroes were sacrificed to Zeus.
B. large sums of prize money were distributed to the heroes.
C. all the victors were crowned with garlands.
D. the heroes were honoured with sacrificial offerings.
3. Competitors had to train__________
A. for four years.

32
B. or ten months.
C. until they collapsed exhausted.
D. for periods determined by then state authorities.
4. Modern athletes’ results cannot be compared with those of the ancient runners because__________
A. details were not recorded.
B. they are much better.
C. the ancient runners fell down dead.
D. the Greeks had no means of telling the time.
5. The continuity of the Olympic Games__________
A. was broken in the year A.D. 1200.
B. has never been broken.
C. was interrupted for over 1,500 years.
D. was broken in 1896.
6. Nowadays, the athletes’ expenses are paid by__________
A. the national funds of competing nations.
B. the winners themselves
C. the athletes themselves.
D. commercial organizations.
7. At the beginning of the Games in the host country__________
A. a torch is ignited at sunrise.
B. a lighted torch is brought into the stadium.
C. relays of runners light their torches in the stadium.
D. a torch is ignited by the Greek ambassador.
8. The modern Olympics compared with the ancient ones are__________
A. inspired by the same ideals.
B. completely different in every respect.
C. more restricted in the variety of events.
D. too much concerned with international rivalry.
IV. WRITING (2.6 pts)
a. Finish each of following sentences in such a way that it means the same as the sentence before
1. She didn’t say a word when she left the room.
→ She left_______________________________________________________
2. They thought that the little girl had found out the keys accidentally.
→ The keys______________________________________________________
3. The kids dived straight into the pool when we arrived.
→ No sooner_____________________________________________________
4. “We are not doing enough to protect the environment from pollution these days”. John said
→ John said_____________________________________________________
5. Because the sales have dropped recently, profits have declined.
→ As a result____________________________________________________
b. Complete the second sentence, using the word given. Don’t change the word given in any way.
1. Kack and Rose never agree on how to bring up then children. (eye)
_______________________________________________________________
2. A reliable source has told me that the local newspaper ỈS going to shut down. (authority)

33
_______________________________________________________________
3. Too little interest has been shown in the project to make it viable. (Not)
_______________________________________________________________
4. Our detectives are determined to find out exactly what happened in this case. (bottom)
_______________________________________________________________
5. We were just going to bed when the Earthquake happened. (point)
_______________________________________________________________
c. Write an essay: “The Asian Children Festival 2016 will help children from Asian countries to further
understand one another. To what extend do you agree with this statement?. Give reason for your answer. You
should write at least 250 words in this space.

34
TT BỒI DƯỠNG VĂN HÓA ĐỀ THI THỬ VÀO LỚP 10 CHUYÊN THPT
HANOI - AMSTERDAM Năm học : 2016-2017
Môn thi : TIẾNG ANH (CHUYÊN)
Điểm Chữ kí GK Thời gian làm bài : 120 phút

( Thí sinh làm bài vào tờ giấy nay)


(Không được dùng bất cứ loại từ điển hay tài liệu nào)

PART A: PHONETICS
I. Choose the word whose underlined part is pronounced differently from that of the others.(5 pts.)
(5 pts.)
1. A. facsimile B. transfer C. spacious D. fax _____
2. A. swallow B. switch C. sweet D. sword _____
3. A. scenic B. extinct C. decrease D. coexist _____
4. A. agreed B. boxed C. based D. listened _____
5. A. off B. of C. if D. fly _____
II. Choose the word whose main stressed syllable is placed differently from the others (5 pts.)
1. A. appreciate B. experience C. embarrassing D. situation _____
2. A. excited B. interested C. confident D. memorable _____
3. A. floppy B. embrace C. cotton D. idol _____
4. A. complain B. destroy C. terrify D. imagine _____
5. A. carefully B. correctly C. seriously D. personally _____
PART B: LEXICO AND GRAMMAR
I. Choose the best answer to complete each of the following sentences. Write your answer (A, B, C or D) in
the box provided. (20pts.)
1. The weather is going to change soon; I feel it in my__________.
A. body B. legs C. skin D. bones
2. Before the invention of the Internet, people couldn’t__________of such universal access to information.
A. reminisce B. conceive C. contemplate D. access
3. __________, Americans eat a light breakfast. They don’t eat a lot of food in the morning.
A. By and large B. Fair and square C. Ins and outs D. Odds and ends
4. There has been a recommendation that Peter__________the president of the country.
A. will be elected B. be elected C. is elected D. was elected
5. For a whole month, Muslims__________eating and drinking during daylight hours.
A. abstain from B. keep from C. stay from D. stand from
6. TV advertising in the late afternoon tends to__________young children.
A. target B. point C. focus D. aim
7. He traveled__________for 20 years and then he decided to return home.
A. farther away B. far from it C. far and wide D. farthest of all
8. No matter how angry he was, he would never__________to violence.
A. refuse B. resort C. resist D. resolve
9. Simon__________in me on the understanding that I wouldn’t tell anyone else.

35
A. confided B. intimated C. confessed D. disclosed
10. If you want to be a rock star, talent helps, but what it really__________down to is luck.
A. boils B. revolves C. centers D. refines
11. That argument is no good: it won’t__________
A. hold water B. blossom C. make water D. pass water
12. If you get measles, you will__________in spots.
A. break out B. break up C. break D. break down
13. Insults roll off him like water __________
A. down a drain B. on a tin roof C. into the river D. off a duck’s back
14. Go to the Chinese__________and bring back a grilled pork chop.
A. carry-away B. carry-on C. take-away D. fast-courses
15. As a result of government__________, more jobs were lost.
A. cut-aways B. cut-backs C. cuttings D. drop-backs
16. No one knows precisely how much he earns a month, but $ 5.000 can’t be__________of the target.
A. far B. broad C. wide D. distant
17. She insisted that the reporter__________her as his source of information.
A. not mention B. doesn’t mention C. hadn’t mentioned D. didn’t mention
18. Look, will you stop__________in and let me finish my sentence!
A. plugging B. pushing C. butting D. moving
19. We put his rude manner__________ignorance of our British customs.
A. up to B. down to C. off at D. up with
20. I would rather you__________the office phone for personal purpose.
A. shouldn’t have used B. shouldn’t use C. not to use D. didn’t use
II. Give the correct form of the word in brackets to complete the following passage. Write your answer in
the box provided. (5 pts.)
Mount Mulanje in Malawi is the highest mountain in (1. CENTRE) Africa, part of a range which comprises
no fewer than twenty peaks over 2,500 meters. The range is readily (2. ACCESS) by road and a day’s drive
allow a (3. LEISURE) circumnavigation. More energetic visitors, particularly walkers and climbers, are
rewarded with an experience that is (4.FORGET).
Mulanje is a (5. BREATH) sight, visible for miles around. The giant slab of rock appears to protrude almost
vertically from the plain. This impression is borne out by the existence of the longest sheer rock-face in Africa,
demanding for even the most skilled (6. MOUNTAIN). The explanation for this dramatic geography lies in the
rock: hard granite, very (7. RESIST) to erosion, which contrasts with the softer rocks of the plains.
Most visitors remain on the lower, gentler slopes, making use of forest huts for overnight accommodation.
The trek up the foothills, along clearly defined paths, is not overly (8. CHALLENGE) but may take up to a week. As the
climate cools gradually, almost (9. PERCEPTIBLE), with every few meters of altitude gained, so the full diversity of fauna
and flora is revealed in all its (10. SPLENDID).
1 2 3 4 5
6 7 8 9 10
III. Fill in each blank with a suitable preposition or particle. (5 pts.)
1. We’re all very obliged__________you
2. He escaped by passing himself__________as a guard.
3. He’s quite careless__________danger.
4. When she sets__________an examination, she always tries to avoid crossing the part of a woman.

36
5. I’ve been so anxious__________you.
6. She refused to be a party__________any violence.
7. Embarrassment rooted her__________the spot.
8. This service is free__________charge.
9. __________The devil and the deep blue sea.
10. We cannot afford to take risks when people’s lives are__________stake.
IV. Underline and correct ten mistakes in the following passage. Write the corrections in the column on the right
(10pts.)
Research has found that children took on a supermarket trip make a purchase L1..................
request every two minutes. More than $200 million a year is now spent on advertising L2..................
directly to children, most of them on television. That figure is likely to increase and it L3..................
is in the supermarket aisles that the investment is most likely to be successful. For L4..................
children, the reasons behind their parents’ decisions about that they can and cannot L5..................
afford are often unclear and arguments about how bad sugar is for your teeth are L6..................
inconvincing when compared with the attractively and emotionally persuasive L7..................
advertising campaigns. L8..................
According to Susan Dibb of the National Food Alliance, ‘Most parents concerned L9..................
about what they give their children to eat and have ideas about what food is healthy - L10................
although those ideas are not always accurate. Obviously, such a dialogue among L11................
parents and children is a good thing, because if the only information children are L12................
getting about productivity is from TV advertising, they are getting a very one-side L13................
view. Parents resent the fact that they are competing with the advertising industry and L14................
are forced into the position of repeated disappointing their children.’ The Independent L15................
Television Commission, which regulates TV advertising, prohibits advertisers from L16................
telling children to ask their parents to buy products. L17................

PART C: READING
I. Read the following passage and answer the questions by choosing the options A, B, C or D. Write your
answer (A, B, C or D) in the box provided. (10 pts.)
EXOTIC AND ENDANGERED SPECIES
When you hear someone bubbling enthusiastically about an exotic species, you can safely bet the speaker
isn’t an ecologist. This is a name for a resident of an established community that was deliberately or
accidentally moved from its home range and became established elsewhere. Unlike most imports, which can’t
take hold outside their home range, an exotic species permanently insinuates itself into a new community.
Sometimes the additions are harmless and even have beneficial effects. More often, they make native species
endangered species, which by definition are extremely vulnerable to extinction. Of all species on the rare or
endangered lists or that recently became extinct, close to 70 percent owe their precarious existence or demise
to displacement by exotic species. Two examples are included here to illustrate the problem.
During the 1800s, British settlers in Australia just couldn’t bond with the koalas and kangaroos, so they
started to import familiar animals from their homeland. In 1859, in what would be the start of a wholesale
disaster, a northern Australian landowner imported and then released two dozen wild European rabbits
(Oryctolagus cuniculus). Good food and good sport hunting - that was the idea. An ideal rabbit habitat with no
natural predators was the reality.

37
Six years later, the landowner had killed 20,000 rabbits and was besieged by 20,000 more. The rabbits
displaced livestock, even kangaroos. Now Australia has 200 to 300 million hippityhopping through the
southern half of the country. They overgraze perennial grasses in good times and strip bark from shrubs and
trees during droughts. You know where they’ve been; they transform grasslands and shrub lands into eroded
deserts. They have been shot and poisoned. Their warrens have been plowed under, fumigated, and dynamited.
Even when all-out assaults reduced their population size by 70 percent, the rapidly reproducing imports made a
comeback in less than a year. Did the construction of a 2,000-mile-long fence protect Western Australia? No.
Rabbits made it to the other side before workers finished the fence.
In 1951, government works introduced a myxoma virus by way of mildly infected South American rabbits,
its normal hosts. This virus causes myxomatosis. The disease has mild effects on South American rabbits that
coevolved with the virus but nearly always had lethal effects on O. cunlculus. Biting insects, mainly
mosquitoes and flenses against the novel virus, the European rabbits dies in droves. But, as you might expect,
natural selection has since favored rapid growth of populations of O. cunlculus resistant to the virus.
In 1991, on an uninhabited island in Spencer Gulf, Australian researchers released a population of rabbits
that they had injected with a calclvirus. The rabbits died quickly and relatively painlessly from blood clots in
their lungs, hearts, and kidneys. In 1995, the test virus escaped from the island, possibly on insect vectors. It
has been killing 80 to 95 percent of the adult rabbits in Australian regions. At this writing, researches are now
questioning whether the calcivirus should be used on a widespread scale, whether it can jump boundaries and
infect animals other than rabbits (such as humans), and what the long - term consequences will be.
A vine called kudzu (Puerarialobata) was deliberately imported from Japan to the United States, where it
faces no serious threats from herbivores, pathogens, or competitor plants. In temperate parts of Asia, it is a
well- behaved legume with a well - developed root system. It seemed like a good idea to use it to control
erosion on hills and highway embankments in the southeastern United States. (A) With nothing to stop it,
though, kudzu’s shoots grew a third of a meter per day. Vines now blanket stream banks, trees, telephone
poles, houses, and almost everything else in their path. Attempts to dig up or bum kudzu are futile. Grazing
goats and herbicides help, but goats eat other plants, to, and herbicides contaminate water supplies. (B) Kudzu
could reach the Great Lakes by the year 2040.
On the bright side, a Japanese film is constructing a kudzu farm and processing plant in Alabama. The idea
is to export the starch to Asia, where the demand currently exceeds the supply. (C) Also, kudzu may eventually
help reduce logging operations. (D) At the Georgia Institute of Technology, researchers report that kudzu
might become an alternative source for paper.
1. Based on the information in paragraph 1, which of the following best explains the term “exotic species”?
A. Animals or plants on the rare species list
B. A permanent resident in an established community
C. A species that has been moved to a different community
D. An import that fails to thrive outside of its home range
2. The world itself in the passage refers to
A. most imports B. new community C. home range D. exotic species
3. The word bond in the passage is closest in meaning to
A. move B. connect C. live D. fight
4. According to the author, why did the plan to introduce rabbits in Australia fail?
A. The rabbits were infected with a contagious virus.
B. Most Australians did not like the rabbits.
C. No natural predators controlled the rabbit population.

38
D. Hunters killed the rabbits for sport and for food.
5. All of the following methods were used to control the rabbit population in Autralia Except
A. They were poisoned B. Their habitats were buried.
C. They were moved to deserts D. They were surrounded by fences
6. Why does the author mention mosquitoes and fleas in paragraph 5?
A. Because they are the origin of the myxoma virus
B. Because they carry the myxoma virus to other animals
C. Because they die when they are infected by myxoma
D. Because they have an immunity to the myxoma virus
7. According to paragraph 6, the Spencer Gulf experiment was dangerous because
A. insect populations were exposed to a virus
B. rabbits on the island died from a virus
C. the virus may be a threat to humans
D. some animals are immune to the virus
8. Why does the author give details about the kudzu farm and processing plant in paragraph 8?
A. To explain why kudzu was imported from abroad
B. To argue that the decision to plant kudzu was a good one
C. To give a reason for kudzu to be planted in Asia
D. To offer partial solutions to the kudzu problem
9. Which of the following statements most accurately reflects the author ’s opinion about exotic species?
A. Exotic species should be protected by ecologists.
B. Importing an exotic species can solve many problems.
C. Ecologists should make the decision to import an exotic species.
D. Exotic species are often disruptive to the ecology.
10. Look at (A), (B), (C), (D) in the last two paragraphs. Where the following sentence could be best inserted
in the passage
(A), (B), (C), or (D)? Asians use a starch extract from kudzu in drinks, herbal medicines, and candy.
1 2 3 4 5 6 7 8 9 10

II. Read the text below and write one word in each space to complete it. Write your answer in the box
provided. (10 pts.)
Society has changed in many ways since the introduction of computers, and people’s lives at home and at
the office have been (1) __________Most people are working for fewer hours per week than they (2) __ to,
and manufacturers and advertising agencies are becoming much more interested in (3) ____ people spend this
extra leisure time. One recent report stated that (4) __________ the number of hobbies had not increased; each
hobby had become more
specialized.
A second (5) __________is that nowadays, many managers would rather spend time with their families than
stay (6) __________in the office every day. Home life seems to be just as important as working. Some
companies now make managers (7) __________their annual holidays even if they don’t want to, because this
leads to such an (8) __________-in their performance if they have some rest.
In spite of these changes, some people are working harder than ever before. The standard of exams is getting
higher, and increased competition is (9) __________it harder to get into university than it was 20 years ago.

39
School children and students are now having to work so hard that in many cases they work (10)
__________hours than their parents.
PART D: WRITING
I. Rewrite each of the following sentences using the word(s) given so that its meaning stays the same.
(10pts.)
1. I have frequently made stupid mistakes like that.
→ Many’s_________________________________________________________________
2. I rarely sleep in the afternoon.
→ I’m not in_______________________________________________________________
3. You think that fat people are always jolly, but you are wrong.
→ Contrary________________________________________________________________
4. It was not until five years had elapsed that the whole truth about the murder came out.
→ Not for _________________________________________________________________
5. Erika said I had caused the accident.
→ Erika blamed_____________________________________________________________
6. My grandfather had completely forgotten that he phoned me last night. (RECOLLECTION)
→ My grandfather didn’t have__________________________________________________
7. Many customs restrictions within the EC have been abolished. (AWAY)
→ Many customs____________________________________________________________
8. At the moment I can’t afford to buy a new car. (QUESTION)
→ At the moment____________________________________________________________
9. Students at the school are not allowed to go into the Rainbow Disco. (BOUNDS)
→ The Rainbow Disco _____________________________________________________
10. When they broke the news, she stayed perfectly calm and controlled. (HAIR)
→ When they broke the news __________________________________________________
II. Write a composition about the following topic:
Food-safety violations, including using improved fertilizers and toxic chemicals to produce foods, are
increasing at an alarming rate. What suggestions would you give to solve these problems? You should write
about 250 words. (20 pts.)

---The end---

40
KỲ THI TUYÊN SINH VÀO LỚP 10 THPT HỆ CHUYÊN
NĂM HỌC 2015-2016
Môn thi: Tiếng Anh (CHUYÊN)
Ngày thi: 13-06-2015
Thời gian làm bài: 120 phút
PART I: LISTENING (1.5 pts)
Listen to a talk about healthy eating twice, fin in each blank with no more than three words or a number
- Very important to eat a (1)________. People in Europe and the USA eat (2) ________more sugar now than
in 1800.
- Try to eat plenty of fresh fruit and vegetables with (3) ________servings a day, if possible.
- Avoids food containing a lot of sugar, especially (4) ________drinks.
- Cut down on fat. Eat lean meat, fish and low-fat (5) ________
- Eat no more than (6) ________eggs a week.
- Use lemon juice instead of (7) ________in cooking.
- Try to eat (8) ________three times a day.
- The most nutritious fruit is (9) ________.It contains 165 calories per 100 grams (10) ________protein a milk.
Listen to a dialogue Joanne Rob twice and circle the correct answer A, B, C or D
11. Joanne says that visitors to Darwin are often surprised by
A. the number of young people B. the casual atmosphere
C. the range of cultures D. the number of tourists
12. To enjoy cultural activities, the people of Darwin tend to________
A. travel to southern Australia. B. bring in artists from other areas.
C. involve themselves in production. D. go to the theatre and opera
13. The Chinese temple in Darwin________
A. is no longer used for its original purpose.
B. was rebuilt after its destruction in a storm.
C. was demolished to make room for new buildings.
D. is closed to the tourists
14. The main problem with travelling by bicycle is
A. the climate B. the traffic C. the hills
15. What does Joanne say about swimming in the sea?
A. It is essential to wear a protective suit. B. Swimming is only safe during the winter,
C. You should stay in certain restricted areas. D. You can be trapped in the nets
PART II: PRONUNCIATION, VOCABULARY AND GRAMMAR (3.5pts)
Circle the word that differs from the others in the position of the primary stress
16. A. economical B. personality C. preparatory D. entertainment
17. A. interrupt B. interview C. intervene D. intimation
18. A. surprising B. astonishing C. amazing D. interesting
Circle the word that has underlined part pronounced differently from the others
19. A. scissors B. possess C. dissolve D. tosses
20. A. explain B. complain C. certain D. campaign
Circle the best option A, B, C or D to complete each of the following sentences
21. The teacher said that I would be able to speak English fluently________six months.
A. in B. by C. since D. till

41
22. The police began an________into the disappearance of painting 5 days ago.
A. investigation B. investigatory C. investigate D. investigating
23. They are having their house________by a construction company.
A. to paint B. painting C. being painted D. painted
24. Only after doing his chores________to go meet his fiends..
A. Jim was allowed B. did Jim allow C. was allowed Jim D. was Jim allowed
25. I hope you will________notive of what I’m going to tell you.
A. gain B. keep C. get D. take
26. There were some very strong candidates for the job, but none of them________my expectations
A. took B. gave C. made D. met
27. ________outer space, our earth looks like a “blue planet”.
A. Seen B. Having seen C. Seeing D. Be seen
28. British and Australian people share the same language, but in other aspects they are different as________
A. cats and dogs B. chalk and cheese C. salt and pepper D. here and there
29. Even if that laptop is the most expensive in the store, it doesn’t________mean that it’s the best.
A. severely B. valuably C. doubtfully D. necessarily
30. Graham was exhausted. He________tennis for an hour.
A. had been playing B. played C. has been playing D. was playing
Write an adjective or a noun that is similar in meaning to each of the phrases below
31. not good or pure enough to drink: ________
32. a person who tends to look on the bright side of things: ________
33. lasting or intended to last or be used for a short-time: ________
34. a room or building used for scientific research, experiment, testing, etc: ________
35. a person who is concern about protecting the environment: ________
Complete the following passage by writing the correct form of the words given in brackets
A recent front-page in the British press revealed a truly (36. sensation) ________music discovery-six Haydn
piano sonatas had been found in Germany.
(37. Apparentness) ________the long-lost sonatas were discovered by a German music teacher in the home
of an elderly lady. (38. Strange) ________enough, the manuscripts, which have been pronounced genuine by
several eminent (39. Musicological) ________, were not made (40. Availability) ________their original form,
so not (41. Science) ________test could be carried out in order to verify their (42. Authentic) ________. The
musical world was thrown into a state of great excitement by this news. (43. Prepare) ________were made for
prestigious recording of the sonatas, and an (44. Author) ________article was published in the BBC’s Music
Magazine. Since then, however, a note of doubt has crept in, and experts now say that unless the (45. Origin)
________are handed over very soon, the manuscript must be regarded as a forgery, albeit a very clever one.
Complete each of the following sentence using the correct form of phrasal verbs from the box.
There are two extra ones.
Fall out seize up let off stand for take in stand over bring in

46. Since Sarah________-with her next door neighbor, she had been said.
47. I can’t concentrate with him________me like that.
48. If a machine stops moving or working normally, you can say that it has________
49. Don’t be________by products claiming to help you to lose weight in a week.
50. Sally was________with a warning by the police because it was her first offence.

42
PART III: READING COMPREHENION (2,5 pts)
Read the article below. Five sentences have been removed from it Choose from the sentences A-F the one
which fits each gap. There is one extra sentence you don’t need to use.
According to a recent study, whereas plastic bags were rarely seen at sea in the late eighties and early
nineties, they are now being found almost everywhere across the planet, from Spitsbergen in the Arctic to the
South Atlantic close to Antarctia. They are among the 12 items of rubbish most often found in coastal clean-
ups. [51] ________Windblown plastic bags are so common in Africa that a small industry has appeared:
harvesting bags and using them to make hats and other items, with one group of people collecting 30,000 per
month. In some developing countries they are a major nuisance in blocking the drainage systems of towns and
villages.
What matters is what happens to them after use. Enormous numbers end up being buried or burnt, which is
an enormous waste of the oil products which have gone into their manufacture.
[52] ________Turtles mistake them for their jellyfish food and choke on them; birds mistake them
for fish with similar consequences; dolphins have been found with plastic bags preventing them breathing
properly.
The wildlife film-maker Rebecca Hosking was shocked by the effects of the bags on birds on the Pacific
island of Midway. She found that two-fifths of the 500,000 albatross chicks born each year die, the vast
majority from swallowing plastic that their parents have mistakenly brought back as food. Many local residents
and shopkeepers joined in, and the idea of getting rid of them completely soon spread to other towns and
villages.
Although some people remain unconvinced, it does seem possible that the entire country could eventually
become plastic-bag free. Who could have imagined half-a-century ago that our public places would one day all
become cigarette-smoke free? Or that we would all be using lead-free petrol? Who would have thought even a
decade ago, come to that, that about two-thirds of us would by now be actively involved in recycling? [53]___
What is needed is a general change in consumer attitudes, towards the habit of using re-usable shopping
bags. Older people will remember how this used to be entirely normal as every household had a ‘shopping
bag’, a strong bag which was used to carry items bought in the daily trip to the shops. [54] ________Today,
many of us tend to drive to the supermarket once a week and fill up the car with seven days’ worth of supplies,
for which plastic bags, of course, are fantastically useful. It’s a hard habit to break.
However, there has already been a big drop in plastic bag, use, partly because the leading supermarkets and
other shopkeepers are making a major effort to help us give up the habit, with a whole variety of new ideas.
[55] ________It is clear that habits are starting to change; reusable bags are more visible than they were even
two years ago.
Many believe there should be a tax on plastic bags, and the governments of a number of countries are
considering the idea. What people have in mind is the example of Ireland, where a tax of €0.22 was introduced
on all plastic bags, and the first of its kind in the world.
A. Major changes in public opinion and behaviour can certainly occur.
B. On land they are everywhere, too.
C. These range from cheap ‘bags for life’ offers to bag-free check-outs.
D. Worse still, billions get into the environment, especially the ocean environment, where they become a
terrible threat to wildlife.
E. But there was a very different pattern of household shopping then: the purchase of a much smaller
number of items, on a daily basis, after a walk to small, local shops.
F. She realised then that it was too late to do anything about this man-made disaster.

43
Read the following text and circle the correct answer (A, B, C or D) which best fit each gap.
Tourism is the fastest-growing industry in the world. As well as bringing prosperity to an area, however, it
can also destroy the qualities which (56) ________visitors in the first place. If it is not carefully control,
tourism can also (57) ________problems for local people, as is shown by various examples from around the
world.
When Phuket in Thailand first became a popular tourist (58) ________, people there were unable to cope
with the increase in rubbish that two millions visitors year (59) ________and a huge incinerator had to be
built in the countryside to deal with it. New hotels at Goa in India caused a huge increase in water
consumption, (60) ________many local people to walk considerable distance to get clean water. And Egypt’s
desert landscapes are being destroyed by the little dropped by tourists. Moving sands are difficult to clean, and
the white desert to the West of the Nile Valley may permanently (61) ________
It’s encouraging, therefore, to rad about the (62) ________of certain tour companies who are organizing
environmental holidays in some of the worst hit areas. Regular tours now go to places (63) ________the
Himalayas and Atlas mountains in North Africa with the (64) ________of combining a bit of sightseeing with
the chance to help (65) ________some of the mess left by previous visitors.
56. A. convinced B. persuaded C. attracted D. appealed
57. A. create B. form C. lead D. invent
58. A. position B. destination C. terminus D. departure
59. A. reject B. produce C. establish D. involve
60. A. encouraging B. making C. forcing D. urging
61.A. injured B. touched C. influenced D. affected
62.A. efforts B. obstacles C. strains D. duties
63 .A. as well as B. as for C. such as D. so as to
64. A. hope B. plan C. wish D. aim
65. A. rub out B. care for C. clear up D. break off
Read the following text and fill in each blank with suitable word.
Twenty years ago, kids in school had never even heard of the Internet. Now I’ll bet you can’t find a single
person in your school who hasn’t at least heard of it. In fact, many of us use it on a (66) ________basis and
even have access to it from our homes! The ‘net’ in internet really stands for network. A networks is two or
more computers connected together so that information can be (67) ________, or sent from one computer to
another. The internet is a vast resource for all types of information. You may enjoy using it to do research for a
school project, (68) ________your favorite songs or communicating with friends and family. Information is
(69) ________through web pages that companies, organizations and individuals create and post. It's kind of
like a giant (70) ________board that the whole world uses! But since anyone can put anything on the internet,
you also have to be careful and use your best (71) ________and a little common sense.
Just because you read something on a piece of paper someone sticks on a bulletin board doesn’t mean it’s
good information, or even correct, for that matter. So you have to be sure that whoever (72) ________the
information knows what they're talking about, especially if you're doing research! But what if you’re just
emailing people? You still have to be very careful. If you’ve never met the person that you're (73) ________
with online, you could be on (74) ________ground! You should never give out any personal information to
someone you don't know, not even your name! And just like you can’t believe the information on every
website out there, you can’t rely on what (75) ________you ‘meet’ on the internet tell you either. Just like you
could make up things about yourself to tell someone, someone else could do the same to you!
PART IV: WRITING (2.5 pts)

44
Complete the second sentence so that it has similar meaning to the first sentence.
76. Neil remembered to pack everything except his toothbrush.
→ The only thing___________________________________________________________
77. Peter hadn’t expected to see so many foreigners at the party..
→ It came as_______________________________________________________________
78. Jacqui and I were sitting by ourselves at the back of the coach.
→ Jacqui and I______________________________________________________________
79. We will do everything possible to avoid a disaster.
→ Everything_______________________________________________________________
80. Exhaustion prevented any of the runners from finishing the race.
→ So______________________________________________________________________
Complete the second sentence so that it has similar meaning to the first sentence, using the words given in
brackets. Don’t change the word given.
81. Sarah rang because she was worried about us. (rung)
__________________________________________________________________________
82. There was no point in staying at the party because my friend had left. (worth)
__________________________________________________________________________
83. Terry was rude, but Sue got here revenge on him. (being)
__________________________________________________________________________
84. If you are not careful, you are going to be dismissed. (sack)
__________________________________________________________________________
85. I am looking for a job which is commensurate with my abilities. (level)
__________________________________________________________________________
86-100. In about 200 words, write an essay about the advantages and disadvantages of living in the modern
world.
—The end—

45
TT BỒI DƯỠNG VĂN HÓA ĐỀ THI THỬ VÀO LỚP 10 CHUYÊN THPT
HANOI-AMSTERDAM Năm học : 2014-2015
Môn thi: TIẾNG ANH (CHUYÊN)
Thời gian làm bài: 120 phút

PART A: PHONETICS
I. Choose the word whose underlined part is pronounced differently from that of the others.(5 pts.)
1. A. considerate B. candidate C. associate D. adequate
2. A. warranty B. wasabi C. wallop D. wander
3. A. sanctuary B. manky C. sanguine D. redundant
4. A. modal B. model C. modest D. modern
5. A. bathing B. method C. bathroom D. width
II. Choose the word whose main stress syllable is placed differently from that of others (5pts)
1. A. Europe B. monument C. province D. minority
2. A. substantial B. technological C. exponential D. infiltration
3. A. obvious B. notorious C. credulous D. numerous
4. A. arithmetic B. assassinate C. agriculture D. controvert
5. A. tuberculosis B. mathematician C. communication D. inheritance
PART B: LEXICO AND GRAMMAR
I. Choose the best answer to complete the sentence. (15 pts)
1. The experience of the child in his first year largely_________his character and later personality.
A. determine B. divide C. distinguish D. deny
2. _________is normal used to refer to the treatment and training of the child with the home.
A. Feeding B. Mothering C. upbringing D. Educating
3. Throughout life, each stage depends on the satisfactory_________of the one before.
A. succession B. completion C. perfection D. continuation
4. No country takes_________over another in the U.N.
A. precedence B. decision C. charge D. border
5. Have you seen your horoscope in the newspaper?- No, I haven’t. I’m not interested in_________
A. agronomy B. astrology C. astronomy D. anthropology
6. How exactly did you set_________training the horses to work so well together?
A. up B. to C. loose D. empty
7. There’s a rumour that the Nation Bank is going to_________the Company I work for.
A. takeover B. overtake C. take on D. take off
8. By the year 2015, many people currently employed_________their jobs.
A. have lost B. will have lost C. will be losing D. are losing
9. _________the public’s concern about the local environment, this new road scheme will have to be
abandoned.
A. As regards B. In view of C. In the event of D. however much
10. Recent engineering developments have made_________to recycle plastic soda bottles into
polyester fabric.
A. possible and B. it is possible C. the possible D. it possible
11. You can take the bus_________you don’t mind waiting.

46
A. except B. provided C. unless D. whether
12. He took_________golf when he retired from work.
A. up B. over C. on D. after
13. I’ll_________the idea with the other members in family and let you know.
A. explain B. argue C. discuss D. talk
14. I know for_________that he was at the scene yesterday.
A. exact B. certain C. true D. correct
15. The new system didn’t_________expectations.
A. catch up with B. bring bout C. come across D. come up to
II. Choose the words closest in meaning to the meaning words
Our class went to Pulau Ubin for a camping trip this September. It was the first camping trip for many of us.
The (1) veteran among us was Bart, who had been to a scout camp at Jalan Tiga last year. We (2) anticipated a
lot of fun during the camp, and we were not (3) let down. Our campfire was very (4) memorable. To begin
with, none of us had ever seen such a huge bonfire in our lives. The fire (5) illuminated the dark surroundings
and our faces were a bright orange glow. Our teacher gave a very moving speech at the end of the camp. She
said that she would miss us when we graduated from our primary school.
1. A. amateur B. old hand C. talented D. elite
2. A. predicted B. looked out for C. waited D. prepared
3. A. disheartened B. disappointed C. discouraged D. disagreed
4. A. unforgettable B. interesting C. notable D. momentous
5. A. glowed B. dimmed C. decorate D. lift up
III. Read the passage and fill in each blank with the correct form of the word in brackets. (10 pts)
Modern advertisements contain hidden message (1. Imply)_________in the advertisement showing the
pretty girl in the new car or the smiling children round the packet of washing powder is the message that if we
buy the (2.produce) _________, we also achieve success and happiness. It is a subtle approach since it seeks to
exploit our secret dreams, and it is (3 .escape) _________since advertising is ubiquitous, giant street hoardings
(4.catch) _________jingles on television bombard us from all sides. They brainwash us into believing that we
can realize our ambitions quickly and (5.ease) ______. On the other hand, defenders of advertising say that it is
(6.benefit) _________. Advertising is (7.inform) ______. Advertisements tell us about useful new products.
They (8.bright) _______our lives with color and music. They increase demand, stimulate industry and so keep
prices down. Whether for or against advertising, most people would agree that some kind of watch-dog body,
(9. Appointment) _________by the government or by the advertising
industry itself, is necessary to maintain standards of honesty and to discourage the more blatant types of (10.
lead) _________advertisements.
IV. Complete each of following sentence with one of following verb plus one preposition. Make any changes
to verb tense where necessary. (7pts)
Put, crop, peg, rip, hold, live, meddle, pass, buzz, Back, with, up, off, aside, through, down,
talk around, by, into
1. Ms Ha wanted to wake up, but sleep________her________.
2. We should________our differences and discuss the things we have in common.
3. We are too young to________the war, but we all know how terrible it is.
4. Don’t try to________me________going to the concert. I can’t afford the time.
5. The subject of sex equality seems to________in every discussion lesson in my school.
6. It’s none of your business; please don’t________things that don’t concern you.

47
7. We had difficulty________the tent________in a storm.
8. She________the kitchen making preparation for the party.
9. We can’t________this matter________without protest.
10. I paid too much for those books. I saw them cheaper in another store. I was________.
V. Write one word in each gap to complete the sentence. (5pts)
1. Our new car is________near as economical as our last one.
2. It was very satisfying to watch our house being built and see it________shape.
3. It wasn’t too difficult to find our way through the forest and we only________astray once or twice.
4. Being told I’d got into the University, I wanted to go to really________my day.
5. There was nobody to ask for directions because we were in the________of nowhere.
6. Sorry-I’m late-I was reading and just lost________of time.
7. People in the 16-24________bracket are usually more comfortable with technology than many older people.
8. You have to________people with respect if you want them to respect you,
9. To the________of my knowledge, no one has touched anything on your desk.
10. I’m sure you will do well at University if you’re________in your mind about what you want to achieve.
PART C: READING .
I. Reading the following passage and circle the best answer for each blank.
Getting friends and family to pose for photo is hard enough, but how would you (1) ______with a rabbit, an
owl or a butterfly that simply (2) ________to keep still? Simon King, wildlife film-maker and photographer,
says you don't need any formal (3) ________to get started. “The whole (4) ________is that photographing
wildlife should be fun”. Simon offers the following (5) ________
• Specialize from the start. You're more likely to get good (6) ________sooner if you (7) ________on the type
of wildlife - insects for instance - (8) ________just going off to the woods or park with your camera and
snapping whatever you see.
• (9)________something that isn’t hard to photograph. Choosing an animal that’s hard to (10) ________ or
will run away if it sees you (11) ________unnecessary problems. How about flowers or a group of birds?
• (12) ________second-hand camera shops and local papers for quality (13) ________. You don’t need to
(14) ________a fortune. Simon started with just a second-hand camera that cost around $30. But you will need
a single lens reflex camera,
• Remember it’s the (15) ________- photograph that counts, nor just the subject. (16) ________you’re
composing a picture and try to be as artistic as possible.
1. A. handle B. act C. treat D. cope
2. A. disobeys B. dislikes C. refuses D. avoids
3. A. training B. education C. exercise D. lecture
4. A. Thought B. idea C. dream D. plan
5. A. lessons B. facts C. warnings D. tips
6. A. Progress B. luck C. dream D. plan
7. A. think B. concentrate C. limit D. depend
8. A. more B. other C. better D. rather
9. A. decide B. Pick C. Prefer D. Collect
10. A. spot B. notice C. meet D. glance
11. A. creates B. starts C. puts D. leads
12. A. Visit B. Look C. Find D. Search
13. A. instruments B. equipment C. material D. tools

48
14. A. cost B. make C. spend D. lose
15. A. big B. all C. whole D. full
16. A. think B. guess C. invent D. imagine
II. Read the passage and circle the best answer to each question (8pts)
In most discussions of cultural diversity, attention has focused on visible, explicit aspects of culture, such as
language, dress, food, religion, music, and social rituals. Although they are important, these visible expressions
of culture, which are taught deliberately and learned consciously, are only the tip of the iceberg of culture.
Much of culture is taught and learned implicitly, or outside awareness. Thus, neither cultural insiders nor
cultural outsiders are aware that certain “invisible” aspects of their culture exist.
Invisible elements of culture are important to us. For example, how long we can be late before being
impolite, what topics we should avoid in a conversation, how we show interest or attention through listening
behaviour, what we consider beautiful or ugly- these are all aspects of culture that we learn and use without
being aware of it. When we meet other people whose invisible cultural assumptions differ from those we have
learned implicitly, we usually do not recognize their behaviour as cultural in origin.
Differences in invisible culture can cause problems in cross-cultural relations. Conflicts may arise when we
are unable to recognize others’ behavioural differences as cultural rather than personal. We tend to misinterpret
other people’s behaviour, blame them, or judge their intentions or competence without realizing that we are
experiencing cultural rather than individual differences.
Formal organizations and institutions, such as schools, hospitals, workplaces, governments, and the legal
system are collection sites for invisible cultural differences. If the differences were more visible, we might
have less misunderstanding. For example, if we met a man in a courthouse who was wearing exotic clothes,
speaking a language other than ours, and carrying food that looked strange, we would not assume that we
understood his thoughts and feelings or that he understood ours. Yet when such a man is dressed similarly to
us, speaks our language, and does not differ from us in other obvious ways, we may fail to recognize the
invisible cultural differences between us. As a result, mutual misunderstanding may arise.
1. What is the main purpose of the passage?
A. To explain the importance of invisible aspects of culture.
B. To describe cultural diversity.
C. To point out that much of culture is learned consciously.
D. To explain why cross-cultural conflict occurs.
2. The word “deliberately” in paragraph 1 is closest in meaning to________
A. slowly B. accurately C. intentionally D. randomly
3. The phrase “the tip of the iceberg” in paragraph 1 means that________
A. other cultures seem cold to us
B. visible aspects of culture are learned in formal institutions
C. we usually focus on the highest forms of culture
D. most aspects of culture cannot be seen
4. Which of the following was NOT mentioned as an example of invisible culture?
A. What topics to avoid in conversation
B. What food to eat in a courthouse
C. How late is considered impolite
D. How people express interest in what others are saying
5. The word “those” in paragraph 2 refers to________
A. people from a different culture B. invisible cultural assumptions

49
C. people who speak a different language D. topics that should be avoided
6. It can be inferred from paragraph 3 that conflict results when________.
A. people think cultural differences are personal
B. people compete with those from other cultures
C. one culture is more invisible than another culture
D. some people recognize more cultural differences than others
7. The author implies that institutions such as schools and workplaces________.
A. are aware of cultural differences
B. teach their employees about cultural differences
C. reinforce invisible cultural differences
D. share a common culture
8. Which of the following would most likely result in misunderstanding?
A. Learning about our own culture in school
B. Unusual food being cooked by foreign visitors
C. Strange behaviour from someone speaking a foreign language
D. Strange behaviour from someone speaking our language
PART D: WRITING
I. Rewrite each of the sentences using the words given so that its meaning says the same. (6pts)
1. Why do I always get the boring jobs?. I’m fed up with it. (donkey)
___________________________________________________________________
2. The thief was sent to prison for the crimes he had committed. (time)
___________________________________________________________________
3. My brother kindly lent me his car for the weekend. (favor)
___________________________________________________________________
4. The painter tried out a lot of different techniques. (experimented)
___________________________________________________________________
5. A number of young artists valued old customs and traditions. (see)
___________________________________________________________________
6. Rolando was very sorry he couldn’t celebrate the New Year with his mother
Rolando greatly regretted_______________________________________________
II. Write about the following topic: (20 pts)
The environment problems facing today’s world are great What each individual should do to reduce the
amount of damage being done to the environment?. (At least 200 words)

50
TT BỒI DƯỠNG VĂN HÓA ĐỀ THI THỬ VÀO LỚP 10 CHUYÊN THPT
HANOI - AMSTERDAM Năm học: 2014-2015
Môn thi: TIẾNG ANH (KHÔNG CHUYÊN)
Thời gian làm bài: 120 phút

PART A: PHONETICS
I. Choose the word whose underlined part is pronounced differently from that of the others. (5 pts.)
1. A. who B. we C. wear D. walk
2. A. tool B. moon C. soon D. cook
3. A. imagine B. another C. man D. language
4. A. beaches B. goes C. fishes D. kisses
5. A. comprise B. realize C. promise D. smile
II. Choose the word that differs from the rest in the position of main stress (0,5 pt)
1. A. friendliness B. primary C. tropical D. correspond
2. A. impression B. difficult C. optional D. separate
3. A. introduction B. population C. recreation D. Hinduism
4. A. baggy B. design C. equal D. poetry
5. A. enjoyment B. pagoda C. business D. collection
PART B: VOCABULARY AND GRAMMAR
I. Choose the best answer to complete the sentence. (1,5pts)
1. People aged 65 and over now________nearly 20% of population.
A. comprise B. contain C. consist D. combine
2. “How long have you been with the company?”. “I________there for two years.”
A. will work B. was working C. worked D. have worked
3. Coffee will not grow________a cold climate.
A. in B. on C. at D. with
4. The new country hopes to establish________relations with all its neighbors.
A. friend B. friendly C. friendliness D. friendship
5. Mom always________us to discuss our problem.
A. encourage B. encouragement C. encouraging D. encouraged
6. I wish the sun________shining right now.
A. is B. are C. were D. would
7. The President doesn’t have much time for________
A. relax B. relaxing C. relax D. relaxation
8. We have rooms for 2000 students________campus.
A. in B. on C. at D. among
9. Please write to us for further________of this offer.
A. details B. detailer C. detailed D. detailing
10. It wasn’t very polite________you to keep interrupting me.
A. about B. to C. for D. of
11. I don’t see any________in arriving early at the theatre if the show doesn’t start until 9 o’clock.
A. cause B. reason C. aim D. point
12. We have bought extra food________our friends stay to have dinner.

51
A. in case B. if C. provided D. as long as
13. Cut this cake into six________pieces.
A. same B. alike C. like D. equal
14. “I take no interest in fishing”. “_________”
A. I do, too B. Neither do I C. Aren’t you D. So am I
15. “Excuse me. Is this seat free?” “_________”
A. No, you can’t sit here B. Yes, it is seated by the boy
C. Sorry, it is taken D. Yes, but I don’t know so.
II. Give the correct form of the word in brackets (1 pts)
1. I was hoping this book would help me with my history project, but in fact it wasn’t very (inform)
2. The Internet has an incredibly powerful means of (communicate).
3. My (friend) with Mary is one of the most important things in my life.
4. We all watched (nerve) as our monitor made his speech of thanks.
5. Don’t you think that we should make sure that (able) people have the same rights as everyone else?
6. Hasn’t anyone ever told you that it’s very (polite) to interrupt when other people are talking? .
7. Minh is so sensitive that you have to be very (care) about what you say to him.
8. That red (build) over there is the Science Department.
9. Is military services (compulsion) in your country?
10. (Research) have announced that a major breakthrough has been made.
III. Circle the underlined word/phrase that is incorrect in each sentence and correct it.
1. This is an informal meeting, therefore everyone is casual dressed.
A B C D
2. We should encourage the youth to learn more about our long-lasting traditional.
A B C D
3. Information technology develops in Vietnam in recent years.
ABC D
4. Vietnam is divided into three main regions know as the North, the South and the Central.
A B C D
5. He told me that there is a fire on the street and a group of firefighters worked hard.
A B C D
6. It’s comfortable sitting in a peaceful cafe and listen to music.
A B C D
7. Erika has written many short stories when she left University.
A B C D
7. Passengers are required to arrive to the gate fifteen minutes before departure time.
A B C D
9. The question was not difficult so that many students answered it correctly.
A B C D
10. Camels, that are the most popular desert animals can be found in most deserts.
A B C D
IV. Put each verb in brackets in the correct tense of form.
a. Someone must have taken my bag. I remember (1.leave)_______it by the window and now it (2.go) ___
b. What this word (3. Mean) _______?
- I’ve no idea. I (4. Never see) _______it before. I suggest (5. Look) _______it up in the dictionary.

52
c. Neither industries nor the garbage disposal (6. Account) _______for the population in this city.
d. I (7. Annoy) _______because I have to repeat myself many times while he (8. Not listen)
e. Many project (9. Carry) _______out (10. Ensure) _______road safety in Vietnam now.
PART C: READING
I. Read the following passage in choose the best answer for each blank. (1.5 pts)
Every year, the village of the Pettineo celebrates its unique arts festival. For a few days each summer, artists
from all over Europe ....1....at this village near the north coast of Sicity to....2....the creative atmosphere.
During their stay, they get....3.... with the local people to paint a one-kilometre long picture that runs
the...4....of the high sheet... 5....the painting is done, each visiting artist joins a local family for a big lunch
and, ....6....the meal, the family receives the.....7.....of the painting that the artist has painted. As a
result,...8...few villagers are rich, almost every home has at least one painting by a well-known European artist.
Visitors to the village are eagerly....9....into homes to see these paintings.
The festival was the ....10... of Antonio Presti, a local businessman who....11....it up several years ago. Since
then, Pettineo has....12....a sort of domestic art museum in....13....any visitor can ring a doorbell, go into a
house and....14....a painting. In addition to this exhibition of paintings in people’s homes, for those who have
time to spare, there is an opportunity to ...15... through the display of huge sculptures in the village square.
1. A. group B. crowd C. gather D. combine
2. A. amuse B. enjoy C. entertain D. delight
3. A. linked B. jointly C. combined D. together
4. A. size B. measure C. length D. area
5. A. just B. Once C. Soon D. Only
6. A. in addition to B. in place of C. in common with D. in exchange for
7. A. partition B. section C. division D. region
8. A. though B. despite C. since D. even
9. A. persuaded B. invited C. requested D. attracted
10. A. imagine B. purpose C. thought D. idea
11. A. set B. put C. got D. had
12. A. become B. advance C. grown D. increased
13. A. what B. where C. whom D. which
14. A. wonder B. stare C. admire D. respect
15. A. move B. step C. wander D. march
II. Read the passage carefully and do the tasks that follow. (1 pt)
One of the life’s pleasure is reading. To some, it may sound boring, but for me reading opens up a
whole new world. It can transport me to mystical and unknown lands far away. It allows me to get
acquainted with characters I would have never met in real life. All these are products of my
imagination, and reading is the key that unlocks it. I was never keen on reading when I was young. It
was not until my mother started reading bedtime stories for us every night that I enjoyed my very
first storybook. It was a small, but colorfully-illustrated children’s bible storybook which still sits in
a comer on my bookshelf today.
Gradually, my mother taught us to read and soon, my siblings and I started reading books by authors
such as Enid Blyton, Agatha Christie and Roald Dahl. It was the most exciting discover. We would
peer through these books page by page, totally engrossed. We would read while walking from place
to place, on the bus, at the dinner table, anywhere, as long as we had a book I our hands. Our parents
were very exasperated with us but there was little they could do to stop us. In a way, I would say that

53
we were “addicted” to reading.
Of course, now that we are older and more disciplined, we choose our time to read books wisely. As we
mature, we are also more selective in the kinds of books we read. My brothers love to read stories that tell of
magic, murder and intrigue, while my sister prefers highbrow books that’ are recommended by schools as
literary classics. As for myself, I have a habit of selecting books based on their authors. I read books by
W.Somerset Maugham and John Steinbeck, and like my brother, I enjoy science fiction by J T Tolkien too.
Basically, reading is a pleasure that is cultivated through time, It was a good thing my mother realized the
importance of reading and got us interested in it. In a way, she probably realized that it would keep us out of
trouble too. Now we have accumulated so many shelves of books at home that there is hardly enough time to
read ever single one of them.
1. When the writer says that “reading opens up a whole new world”, she means that reading______
A. enables her to travel to different parts of the world
B. fires her imagination and she imagines herself in other places and meeting strange people.
C. enables her to meet strange characters in real life.
D. holds the key to a new world.
2. The writer became interested in reading after______
A. her mother started reading to her every night.
B. reading the bible story by herself.
C. she started attending school
D. she read a book by Enid Blyton
3. The writer’s parents were “exasperated” with the writer and her brother because they______
A. love reading
B. read the books page by page
C. read the book tirelessly the whole day.
D. read whatever books they could lay their hands on
4. In what way have the writer and her brother become more “disciplined” now that they are older?
A. They choose what they read more carefully.
B. They are more mature.
C. They read different type of books.
D. They read at appropriate times.
5. What was another possible reason suggested by the writer as. to why their mother encouraged them to read?
A. Reading was a pleasurable thing to do.
B. She wanted to fill up the shelves with books
C. She wanted to keep them busy and out of mischief.
D. She wanted to get them interested in reading.
PARD D: WRITING
I. Complete the second sentence so that it has a similar meaning to the first one (1 pt)
1. They will announce the results of the final exam in two more weeks.
→ The results________________________________________________
2. She can not run as fast as her friend.
→ She wishes________________________________________________
3. You are late again. You have already been late once this week
→ It’s the___________________________________________________
4. Due to traffic in Vietnam, foreign tourists prefer walking to driving.

54
→ Foreign tourists would rather_________________________________
5. “Let me take you home on my motorbike”, John said to me.
→ John offered______________________________________________
II. Arrange the given words in the correct order to make meaningful sentence (1 pt)
1. Regular, wonders, would, life, exercises, way, at, you, the, for, look, do (12 words)
______________________________________________________________________
2. African, opportunity, It, environment, unique, in, to, is, natural, see, a, wildlife, its (13 words)
______________________________________________________________________
3. What, freedom, teacher, Out, to, to, gave, lot, we,, paint, want, a, art, us, of (15 words)
______________________________________________________________________
4. bright, atmosphere, line, makes, sky, meteor, it, earth’s, when, the, the, a, a, enters, in (15 words)
______________________________________________________________________
5. By, learn, learn, together, parents, parents, more, more, children, children, their, their, about, from, and (16
words)
______________________________________________________________________

---The end---

55
TT BỒI DƯỠNG VĂN HÓA ĐỀ THI THỬ VÀO LỚP 10 CHUYÊN THPT
HANOI - AMSTERDAM Năm học: 2013-2014
Môn thi : TIẾNG ANH (KHÔNG CHUYÊN)
Thời gian làm bài: 120 phút

PART A: PHONETICS
I. Choose the word whose underlined part is pronounced differently from that of the others.ỰL5 pts.)
1. A. campus B. practice C. language D. favorite
2. A. qualify B. scenery C. grocery D. academy
3.A. speak B. need C. heart D. read
4. A. practiced B. learned C. asked D. watched
5. A. school B. scholarship C. chemistry D. children
II. Choose the word that differs from the rest in the position of main stress (0,5 pt)
1. A. material B. university C. majority D. embroider
2. A. fashionable B. convenient C. traditional D. minority
3. A. ethnic B. modem C. practical D. economic
4. A. freedom B. uniform C. pagoda D. favorite
5. A. design B. mention C. prefer D. inspire
PART B: VOCABULARY AND GRAMMAR
I. Choose the best answer to complete the sentence. (1,5pts)
1. It hasn’t rained_______the end of July.
A. in B. from C. for D. since
th
2. In the 18 century, workers like wearing jean cloth because it was strong and did not_______easily.
A. wear off B. wear out C. wear on D. wear down
3. Don’t forget to bring my dictionary when you_______back tomorrow.
A. come B. will come C. are coming D. would come
4. My car isn’t very reliable. It keeps_______
A. break down B. on break down C. on breaking down D. to break down
5. The air in the city_______with traffic fumes.
A. is heavy polluted B. heavily is polluted C. is heavily polluted D. polluted heavily
6. He will never advance in his job_______he improves his language skills
A. if B. otherwise C. only if D. unless
7. Her electricity was cut_______when she did not pay her bill.
A. out B. of C. off D. down
8. “What_______when you saw the man?” the police office asked.
A. have you done B. were you doing C. are you doing D. do you do
9. I know London quite well because I_______there.
A. used live B. used to living C. was used to live D. used to live
10. Do you think I could have_______water, please?
A. a little B. a few C. few D. much
11. As we can wait no longer for the delivery of your order, we have to_______it off.
A. break B. put C. call D. keep

56
12. Children in large families learn how to get_______with other family members.
A. along B. away C. across D. through
13. My sister was singing an old folksong, a favorite of_______
A. her B. her own C. hers D. herself
14. Vietnamese women today prefer modern clothes_______the Ao Dai.
A. for B. in C. to D. with
15. Anna didn’t look very well today, did she? - _______
A. I’m afraid not B. No, she seemed ill C. Yes, she seemed sick D. A and B
II. Give the correct form of the words in brackets to complete the following passage. (lpt)
DIVING DEEPER
Free-diving is a new sport, and it’s extremely (1. danger) _______, which is perhaps why its (2.popular)
_______is growing fast. Free (3.dive) _______are attached to a line, and then they have to take one deep (4.
breathe) _______, dive as deeply as they can and come up (5. immediate) _______.
The present record is 121 meters, held by the British diver, Tania Streeter. Tania trains very (6.careful)
_______before each dive to build up her physical (7. fit) _______. She never dives until she is completely
(8.confidence) _______that she’s ready.
The danger is caused by the great (9. press) _______at those depths. ‘I think that (10. safe) _______
procedures have to be very strict if we want to avoid accidents’, Tiana says.
II. Circle the underlined word/phrase that is incorrect in each sentence and correct it (1 pt)
1. Last summer, I have stayed on my uncle’s farm for two weeks.
A B C D
2. Hurricane Andrew, that swept through southern Florida in 1992 killed 41 people and made thousands
homeless. A B C
D
3. It’s raining hard, so that we can’t have lunch in the garden.
A B C D
4. When I was ten, I could swim but I wasn’t able dive.
A B C D
5. Scientists have spent years to collect information about UFO.
A B C D
6. My aunt said that she was leaving for Hanoi tomorrow.
A B C D
7. Don’t forget to call home as soon as you will arrive at your destination.
A B C D
8. Getting enough sleep is important in order not fall asleep in class.
A B C D
9. She is angry because of you didn’t tell her truth.
A B C D
10. The Tuoi Tre is a daily newspaper which is wide read by both teenagers and adults.
A B C D
PART 3: READING
I. Read the passage and choose the best answer for each blank (1,5 pts)
Every year, the village of Pettineo celebrates its unique arts festival. For a few days each summer artists
from all over Europe (1)_______at this village near the north coast of Sicily to (2) _______the creative

57
atmosphere, During their stay, the artists get (3) _______with the local people to paint a one-kilometre long
picture that runs the (4) _______of the high street. (5) _______the painting is done, each visiting artist joins a
local family for a big lunch and, (6) _______the meal, the family receives the (7) _______of the painting that
the artist has painted. As a result, (8) _______few villagers are rich, almost every home has at least one
painting by a well-known European artist. Visitors to the village are eagerly (9) _______into homes to see
these paintings.
The festival was the (10) _______of Antonio Presti, a local businessman who (11) _______it up four years
ago. Since then, Pettineo has (12) _______a sort of domestic art museum in (13) _______any visitor can ring a
doorbell, go into a house and (14) _______,a painting. In addition to this exhibition of paintings in people’s
homes, for those who have time to spare, there is an opportunity to (15) _______ through the display of huge
sculptures in the village square.
1. A. group B. crowd C. gather D. combine
2. A. amuse B. enjoy C. entertain D. delight
3. A. linked B. jointly C. combined D. together
4. A. size B. measure C. length D. area
5. A. Just B. Once C. Soon D. Only
6. A. in addition to B. in place of C. in common with D. in exchange for
7. A. partition B. section C. division D. region
8. A. though B. despite C. since D. even
9. A. persuaded B. invited C. requested D. attracted
10. A. imagine B. purpose C. thought D. idea
11. A. set B. put C. got D. had
12. A. become B. advanced C. grown D. increased
13. A. what B. where C. whom D. which
14. A. wonder B. stare C. admire D. delight
15. A. move B. step C. wander D. march
II. Choose the phrase (A-G) that best fits each gap in the following passage. (1 pt)
Stamp collecting is one of the most popular collecting hobbies in the world. Young people, old people, rich
people, (1)_______Stamp collecting has been called “the hobby of kings and the king of hobbies.” King
George V of England, Franklin D. Roosevelt, (2) _______
No one knows exactly when stamp collecting started. It probably occurred right after the first stamp was
issued. We do know that the United Kingdom issued the first stamps (3) _______by an Englishman named
Mount Brown. Since then catalogs of stamps have been published in almost every country. Many great books
(4) _______
A large number of people like to collect stamps just for the pictures of odd (5) _______Some people collect
stamps of one country only. Others do not care about the types of stamps collected as long as they have fun.
A. And magazines about stamps have also been published
B. And no one suggests giving it up
C. And poor people in every country collect stamps
D. And famous stamps are on offer
E. And the stamps catalogue was published in 1864
F. And many other famous people have collected stamps
G. And out-of-the-way places and things.
PART IV: WRITING

58
I. Complete the second sentence so that it has a similar -meaning to the first one (2 pts)
1. I’ve already planned my next holiday. (arrangements)
→ I’ve already___________________________________________________.
2. We did not enjoy our walk along the seafront because it was so windy. (prevented)
→ The strong wind_______________________________________________.
3. “Don’t sit in front of the computer for too long,” my mother told me. (warned)
→ My mother___________________________________________________.
4. Mary didn’t find it difficult to pass her driving test. (difficulty)
→ Mary had____________________________________________________.
5. The basketball coach will make him train very hard. (made)
→ He will______________________________________________________.
6. I regret not telling you the truth. (told)
→ I wish_______________________________________________________.
7. I don’t know the name of the woman. I spoke to her on the phone. (whom)
→ I don’t______________________________________________________.
8. She tried to stay cheerful although she felt sick, (spite)
→ She tried____________________________________________________.
9. You mustn’t miss this opportunity to visit London. (advantage)
→ You must___________________________________________________.
10. People don’t realize how important it is to conserve energy, so they do nothing about it.
→ If people____________________________________________________.
II. Arrange the given words in the correct order to make meaningful sentence (1 pt)
1. traditional/ dinner/ Christmas/ Christmas/ consists/ and/ The/ roast/ pudding/ of/turkey (11 words)
_______________________________________________________________________________
2. always/ their/ villagers/ streets/ clean/ village/ The/ throw/ the/ is/ in/the/ never/ so/rubbish (15 words)
_______________________________________________________________________________
3. children/adults/they/often/who/hard/ Unlike/ flexible/ change/it/to/are/ the/find/speak/way (16 words)
_______________________________________________________________________________
4. earth/world’s/enough/The/provide/energy/gets/for/population/power/that/cannot/solar/the/the/to (16 words)
_______________________________________________________________________________
5.conclusions/ you/ you/ they/ your/ about/ people’s/ draw/ dress/ reactions/ Although/ will/ based/ will/ way/
vary/ to/ the/ dress/ on. (20 words).
_______________________________________________________________________________

---The end---

59
TT BỒI DƯỠNG VĂN HÓA ĐỀ THI THỬ VÀO LỚP 10 CHUYÊN THPT
HANOI - AMSTERDAM Năm học : 2013-2014
Môn thi: TIẾNG ANH (CHUYÊN)
Thời gian làm bài: 120 phút

PART A: LEXICO-GRAMMAR
1. Choose the correct answer to complete each sentence (8 pts)
1. Children and old people do not take kindly to having their daily_______upset.
A. habit B. method C. routine D. custom
2. They bought the land with a_______to building a new shopping mall.
A. view B. purpose C. goal D. reason
3. Have you taken_______a single word I’ve been saying to you?
A. over B. up C. in D. off
4. _______the invention of the steam engine, most forms of transport were horse-drawn.
A. Akin to B. Prior to C. In addition to D. With reference to
5. The_______of two houses proved such a financial burden that they were forced to sell one.
A. upkeep B. upshot C. upsurge D. uproar
6. My brother’s_______keeps me awake at night.
A. sneering B. snoozing C. snarling D. snoring
7. I’m afraid I’m rather_______about the existence of ghosts.
A. skeptical B. partial C. adaptable D. incapable
8. Thank you for your valuable_______to our meeting last week.
A. contribution B. enrichment C. participation D. investment
9. On Saturday afternoon, you will see most of the students from the school_______shops in the town.
A. hanging round B. walking over C. running down D. turning up
10. After the three hours, the heavy rain began to_______
A. loosen up B. ease off C. back away D. pass on
11. Because his argument was so confusing, _______people understood it.
A. clever B. many C. few D. less
12. When they_______for the beach the sun was shining, but by the time they arrived it had clouded over.
A. set off B. went off C. went out D. left out
13. He has made a mistake, and must face the_______
A. hardship B. music C. noise D. play
14. It wasn’t serious; he did it_______
A. by fun B. funny C. for fun D. to fun you
15. When beer is flat you don’t get much_______
A. head B. top C. bubble D. lift off
16. The mud was_______-deep.
A. foot B. sock C. ankle D. toe
II. Give the correct forms of the words in brackets to complete the following sentences (10 pts)
BECOMING A PARENT
Very little in our lives prepares us for (1. Parent) _______. Suddenly, our life is turned upside down and
sorts of (2.familiar) _______demands are placed on you. How we ourselves were treated by our parents in our

60
(3) _______ (YOUNG) can have an (4) _______ (APPRECIATE) effect on who we become as parents. Our
own (5) ______(OBSERVE) of how our parents responded to us creates a model of parenting that is (6)
_______ (INTIMATE) connected to the kind of parents we become. It’s not uncommon for people to show the
same child-rearing (7) _______ (CHARACTER) as their own parents. If your father was an (8)
_______(SYMPATHY) figure who always seemed too busy to care about how you felt, then there’s a chance
you will repeat the same behavior. If your mother was utterly (9) ______(SELF) in her devotion to her
children, there’s a chance that you too will be equally giving and do all that is (10) _______ (HUMAN)
possible for your off spring.
III. Underline and correct ten mistakes in the following passage. (10 pts)
Research has found that children took on a supermarket trip make a purchase L1………..
request every two minutes. More than $200 million a year is now spent on advertising L2………..
directly to children, most of them on television. That figure is likely to increase and it L3………..
is in the supermarket aisles that the investment is most likely to be successful. For L4………..
children, the reasons behind their parents’ decisions about that they can and cannot L5………..
afford are often unclear and arguments about how bad sugar is for your teeth are L6………..
inconvincing when compared with the attractively and emotionally persuasive L7………..
advertising campaigns. L8………..
According to Susan Dibb of the National Food Alliance, ‘Most parents concerned L9………..
about what they give their children to eat and have ideas about what food is healthy L10………
although those ideas are not always accurate. Obviously, such a dialogue among L11………
parents and children is a good thing, because if the only information children are L12………
getting about productivity is from TV advertising, they are getting a very one-side L13………
view. Parents resent the fact that they are competing with the advertising industry and L14………
are forced into the position of repeated disappointing their children.’ The Independent L15………
Television Commission, which regulates TV advertising, prohibits advertisers from L16………
telling children to ask their parents to buy products. L17………

IV. Complete each of the following sentences with one verb and one preposition. Be sure to use the correct verb tense
and form. (7pts)
Take, drop, bring, hold, come, put, call Back, apart, in, out, onto, down, aside

1. When the clock stopped, he_______it_______, found what was wrong, and out the whole thing together
again.
2. I have a part-time job in a shop, which_______about 500 USD a month.
3. Ms Ha wanted to wake up, but sleep_______her_______
4. The caretaker clearly and decisively_______on the side of his best friend.
5. She completely broke down and felt hot tears_______her cheek.
6. We should_______our differences and discuss the things we have in common.
7. The fire brigade should always be_______to a house life.
V. For each question, write one word which can be used in all three sentences (5 pts)
1. Oh, tell Carrie that I send my_______when you see her.
When my mom and dad met, my mum says it was_______at first sight.
My_______for travelling comes from the fact that we moved around a lot when I was young.
2) One of the reasons why Damian and I get on so well is that we have a lot in_______

61
The architect told us that it was_______practice to start building before getting all the permits.
There are some words which are_______to a number of the languages spoken in the area.
3) I don’t really see myself settling down any time in the_______future.
People came from_______and far to attend the celebrity wedding of the year.
We just about managed to get to the airport on time, but it was a_______thing.
4) The two countries are similar with_______to the divorce rates.
Of course we argue, but I don’t think we’re any .different from other couples in that_______
Todd lost a lot of people’s_______when they saw how he treats his wife.
5) One of the important_______of the law is that it recognises the rights of unmarried partners.
In order to find the woman’s missing husband, the police asked if he had any distinguishing_______
I was quite impressed by the safety_______on Bill’s new car.
PART II: READING
I. Read the following magazine article carefully and answer the questions below.
COLORS OF OUR LIFE
(1) Color is one of the first thing we notice about someone or something. Many people think that color is just
about “look” and they don’t really give enough thought to the deeper meanings or importance of color.
Animals and plants use color as a sign. For example, bright red flowers are a sign to bird that says “Come
here”, I’ve got something for you. On the other hand, bright yellow stripes on an insect say “Don’t eat me, I’m
poisonous”.
Of course if people wear these colors, we aren’t saying “don’t eat me” or “go away” but colors in fashion do
say something.
(3) Warm or bright colors give a feeling of lightness or friendliness and are considered to be positive. This is
not to say that darker colors are necessarily negative, but they may give more of a feeling or seriousness. But
what do individual colors say to people?. There’s no easy answer to this because colors mean different things
to different people, but there are a few generally accepted traditional ideas or perceptions about colors.
(4) In Western cultures, while traditionally has meant purity, but it can also symbolize truth, goodness or a new
beginning. Black is often associated with darkness or seriousness and is the color traditionally worn at
funerals. Red can represent blood, passion, anger, or power. Yellow represents vibrancy, radiance, and
sunshine while blue is a cooler, calmer color. Green and brown stand for nature.
(5) There is no such thing as a color that everyone agrees is nice or appealing. You might love blue and find it
calming but your friend might think it is cold and unfeeling. Your boss might like wearing red because she
feels it is exciting. You might hate it because you find it aggressive. However, when she wears red with blue,
you find it less of a problem. Psychologists feel that the important issue about color and feeling may not be the
colors themselves but combinations of colors. Combining one color with another may change the signal we
give.
(6) Experts say that good dressing consists of two colors, so when you next get dressed in the morning, have a
long think about which two you will choose.
a. Read the definitions and find the words refer to in the text. The paragraph where the words can be found
and the first letters of the words are given to you. (7 pts)
1. Containing a harmful material or substance, harmful if touch or swallowed.(adj, para 1) p_______
2. Relating to the way something has been done for a long time. (adj, para 3) t_______
3. The way to see, hear, understand things. (noun, para 3) p_______
4. To be connected or linked to something. (verb, para 4, two words) a_______
5. Ready or likely to attack, (adj, para 5) a_______

62
6. The joining or mixing together of two things. (noun, para 5) c_______
7. Be made up of. (verb, para 6, two words) c_______
b. Indicate whether statement is true (T) or False (F) (8pts)
1. According to the text, most people don’t really understand the importance of color.
2. Animals and plants use color as .a way of sending message.
3. In nature, bright yellow stripes for example, generally mean “I’m beautiful”
4. When humans wear colors, they are sending the same messages as those sent in nature
5. Light colors such as yellow are considered to be serious color
6. Colors can mean different things, but some meanings are similar for most people
7. The text indicates that single colors can have more than one meaning.
8. Psychologists think that the most important thing about color is that it looks nice
II. Read the following passage carefully and choose the best answer for each blank. (15pts)
A YEAR WITH OVERSEAS VOLUNTEERS
I was with overseas volunteers (OV) for a year after leaving university, and I was sent to an isolated village
in Chad, about 500kilometres from the capital N’Djamena. Coming from a ...1... country, I got quite a shock,
as conditions were much harder than I had ...2.... But after a few days I soon got used to ... 3... there. The
people were always very friendly and helpful, and I soon began to appreciate how ...4... the countryside was.
One of my jobs was to supply the village with water. The well was a long walk away, and the women used
to ...5... a long time every day ...6... heavy pots backwards and forwards. So I contacted the organisation and
arranged to ...7... some pipes delivered. We built a simple pipeline and a pump, and it worked first time. It
wasn’t perfect-there were a few ...8..., but it made a great difference to the villagers, 9... had never had running
water before. And not ...10... did we have running water, but in the evenings it was hot, because the pipe had
been ...11... in the sun all day.
All in all, I think my time with OV was a good experience. Although it was not well-paid, it was well ... 12...
doing, and I would recommend it to anyone who was ...13... working for a charity.
Finally, there’s one more reason why I’ll never ...14... working for OV. A few months before I left, I met
and fell in love ...15... another volunteer, and we got married when we returned to England.
1. A. rich B. comfortable C. well-paid D. luxurious
2. A. felt B. planned C. found D. expected
3. A. live B. lived C. living D. lived
4. A. beautiful B. good-looking C. handsome D. sweet
5. A. spend B. spent C. spends D. spending
6. A. fetching B. wearing C. carrying D. holding
7. A. make B. let C. have D. allow
8. A. breaks B. leaks C. splits D. punctures
9. A. which B. that C. they D. who
10. A. hardly B. scarcely C. only D. also
11. A. lied B. lay C. laying D. lying
12. A. worth B. value C. cost D. price
13. A. considering B. thinking C. going D. planning
14. A. regret B. feel sorry C. miss D. lose
15. A. at B. to C. for D. with
PART III: WRITING

63
I. Use the word given in brackets and make any necessary additions to complete the next sentence in such a
way that it is as similar as possible in meaning to the original one. (10 pts)
1. Harry was close to winning the big race. (ace)
→ Harry was_____________________________________________.
2. He is becoming quite famous as a pianist. (name)
→ He is_________________________________________________.
3. I’m sure young children will love this new film. (appeal)
→ I’m sure this___________________________________________.
4. I found her explanation very difficult to believe. (implausible)
→ She__________________________________________________.
5.Erika said that the situation at work was like a family argument. (likened)
→ _____________________________________________________.
II. It seems that the air in some big cities is seriously polluted. Give some causes of air pollution in Hanoi
and suggest some solutions to this problem. (200 words) (20 pts)

---The end---

64
SỞ GIÁO DỤC VÀ ĐÀO TẠO HÀ NỘI KỲ THI TUYỂN SINH VÀO LỚP 10 CHUYÊN THPT
Năm học : 2013-2014
Môn thi: TIẾNG ANH (ĐIỀU KIỆN)
Thời gian làm bài: 120 phút
PART I: PHONETICS-GRAMMAR AND VOCABULARY (5.5 PTS)
I. Choose the word whose underlined part is pronounced differently from that of the others.(0,5 pts.)
1. A. great B. bread C. break D. steak
2. A. too B. food C. soon D. good
3. A. theory B. therefore C. neither D. weather
4. A. see B. saw C. sort D. sure
5. A. laughed B. smiled C. cried D. phoned
II. Circle the best option A, B, C or D to complete each of the following sentences and write your answers
into the box below.(2,0 pts.)
6. We will give him the messages as soon as he______
A. Phones B. will phone C. will have phoned D. is phoning
7. He didn’t want to______late for the interview.
A. arrive B. arriving C. to arrive D. having arrived
8. You’d better go to bed early, ______?
A. Hadn’t you B. Didn’t you C. Wouldn’t you D. Don’t you
9. The helicopter landed______the roof of the building.
A. at B. in C. over D. on
10. John______for me at the airport next Sunday.
A. waited B. wait C. will wait D. has waited
11. I prefer______TV to reading book.
A. watch B. watched C. watching D. to watch
12. The house needs______
A. repaired B. to repaire C. repaires D. repairing
13. That is the man______car has just been stolen.
A. who B. which C. that D. whose
14. You will fail the exam______you study hard.
A. when B. if C. if not D. unless
15. My sister wishes she______go abroad this summer vacation.
A. can B. could C. will D. shall
16. Would you mind______me your bicycle please?
A. lending B. lend C. to lend D. lent
17. Let’s go out now______ ? The weather is so nice.
A. won’t you B. shan’t we C. will you D. shall we
18. She used to______her living by delivering vegetables to local hotel.
A. gain B. get C. earn D. win
19. Shall we travel to Huong pagoda by bus this Sunday? - Yes, ______
A. I shall B. please C. let’s D. let’s not
20. Remember______the door when you go out.
A. to lock B. locking C. locks D. locked

65
21. I was so busy that I couldn’t take part______the picnic with you.
A. on B. at C. to D. in
22. If you______the lottery, what would you do?
A. win B. wins C. won D. to win
23. You need a modem to get______to the internet.
A. access B. information C. programe D. connection
24. Paper money______for over a thousand years.
A. used B. is used C. has used D. has been used
25. When Lan looked into her purse, she found her ring had been______.
A. stolen B. thieved C. robbed D. broken
III. Give correct form of the words (1.0 pt)
26. Nam has a large (collect) ______of stamp.
27-28. We are very (pride) ______of our son who always takes (carefull) ______of others.
29-30.The dress is very (beauty) ______but it is very (expensive) ______.
31. (Lucky) ______we’ll arrive before the dark.
32. He is so (hope) ______about his bright future.
33. You can save energy and money because of having a lot of (invent) ______.
34. The solar water-heating units are environmentally (friend) ______.
35. Let me know when you make a (decide) ______about what to wear for the wedding.
IV. Give correct form of the verbs in the brackets to complete the sentences. (1.0 pt)
36. If I (be) ______- you, I’d leave without you.
37. My father had my bike (repair) ______yesterday.
38-39. “Have you read any books (write) ______by Margaret?”.
- No, I (never hear) of her.
40-41. I (write) ______a letter to my grandmother when the door bell suddenly (ring) ______
42. We’d rather you (lend) ______Brenda some more money.
43-44-45. When I (buy) ______the ticket yesterday, I (realize) ______that I (not have) ______any money.
V. Fill in each gap below with a suitable preposition. (1.0 pt)
46. The school was named______a young hero, Vo Thi Sau.
47-48______the end, we may stay at home and look the house.
49. The music is too loud. Could you turn it______a little, please?.
50. “Give these papers______my secretary”, said Mr Pike.
51-52-53. When I knocked______the door, the manager opened it and asked me to wait______him______ a
few minutes.
54-55. Despite the late departure______the train, it arrived______time.
PART II-READING (2.5 pts)
I. Read the passage carefully, then fill in each blank with the most suitable word. There are two extra words you don’t
need to use. (2.0 pts)
Makes oil is either fit transport

Go whereas ride about instead gets


Driving a car can be expensive. The bicycle, on the other hand, (56)_______a cheap form of (57) _____
because it doesn’t need any fuel. In fact, it costs practically nothing after you’ve bought it. It also helps keep
you (58) _______because you get exercise when you ride it. Another good thing (59) _______a bicycle is that

66
it doesn’t pollute the air. If everyone rode bicycles (60) _______of driving cars, we wouldn’t be using up the
world’s (61) _______so quickly.
While the bicycle has these advantages, it has some disadvantages too. It is convenient only for relatively short
journeys, (62) _______you can travel quite a long way in comfort in a car. Another problem is that the cyclist
is not protected from the weather and (63) _______wet when it rains.
Cycling isn’t very nice in heavy traffic,(64) _______. The best place for a bike ride is a quiet country lane.
Main roads and city streets are often so busy that it needs some courage to (65) _______a bike on them.
II. Read the passage carefully, then fill in each blank with the most suitable word (0,5 pt)
There are at least 2,000 different languages in the world. Of all (66) _______languages, English is the most
widely used. It is used (67) _______business people, airline pilots and sea captains all over the world. It is also
the first language of sports and science. So it is very important to learn English.
Chinese is also an important language because it has the greatest (68) _______of speakers. There are not
only one billion people living in China today, but also many Chinese people living (69) _______China.
Chinese is widely spoken in many parts of Asia and Africa. Russian is spoken in both Europe and Asia. French
is widely understood in Europe and in parts of Canada, Africa and Asia.
From more than 2,000 languages, the United Nations has (70) _______six of them for business. They are
Arabic, Chinese, English, French, Russian and Spanish.
PART III. WRITING (2.0 pt)
I. Rewrite each of the following sentences in such a way meaning as previous sentences (1.0 pt)
1. Someone scratched my car on the door last week. (was)
_________________________________________________________________________
2. The man walked into my office yesterday. I didn’t know him. (who)
_________________________________________________________________________
3. The TV programme was so complicated that the children could not understand it. (too)
_________________________________________________________________________
4. You ought to see the dentist about that tooth. (If)
_________________________________________________________________________
5. This is the best meal I’ve ever eaten. (never)
_________________________________________________________________________
II. Arrange the words and phrases given in order to make meaningful sentences. (1.0pt)
6. Walk/ the/ V too/ is/ will/ bus/ catch/ far/ /I/ to/ so/ think/ It (14 words)
_________________________________________________________________________
7. school/ a/ she/ go/ had/ Jane/ from/ headache/had/ home/ because/ to. (12 words)
_________________________________________________________________________
8. A/ people/ swept/ and/ the/ heavy/ village/ through/ many/ were/ storm/ killed. (12 words).
_________________________________________________________________________
9. Before/ had/ the/ different/ said/ everything/ Ben/ day/ been. (9 words)
_________________________________________________________________________
10. My elder sister/ likes/ because/ symbol/ thinks/ the/ whenever/ abroad/ she/ she/ goes/ to/ “Ao dai”/ it/
wear/ is/ of/ Vietnam . (18 words and phrases)
_________________________________________________________________________
---The end---

67
SỞ GIÁO DỤC VÀ ĐÀO TẠO HÀ NỘI
KỲ THI TUYỂN SINH VÀO LỚP 10 THPT CHUYÊN
Năm học 2013-2014
Môn thi: Tiếng Anh (Chuyên)
Ngày thi: 20 tháng 6 năm 2013
Thời gian làm bài: 120 phút
I. Listening: 8pts (Time for listening: 12 minutes X 2 = 24 minutes)
Part 1: Listen to 8 different situation. Circle the best answer.
1. On the train, you overhear a woman phoning her office. Why has she phoned?
A. To check the time of an appointment.
B. To apologize for being late.
C. To find out where her diary is.
2. You switch on the radio in the middle of a programme. What kind of programme is it?
A. a nature programme
B. a cookery programme
C. a news programme
3. You overhear a conversation between a watchmaker and a customer. What does the watchmaker say about
the watch?
A. It is impossible to repair it.
B. It is not worth repairing
C. He doesn’t have the parts to repair it.
4. You overhear a woman talking about her new neighbors. How does she feel?
A. Offended B. Shocked C. Suspicious
5. You hear a man talking about deep-sea diving. Why does she like the sport?
A. It suits his sociable nature
B. It contrast with his normal lifestyle.
C. It fulfils his need for a challenge life.
6. You turn on the radio and hear a scientist being interviewed about violins. What is the scientist doing?
A. Explaining how a violin works
B. Explaining how a violin is made
C. Explaining how a violin should be played.
7. You hear part of a radio programme about CD-ROMs. What is the speaker’s opinion of the CD-ROMs about
Australia which she tried?
A. Most of them are disappointing
B. You are better off with an ordinary guidebook
C. There is little different between them.
8. You turn on the radio and hear a woman giving advice to business people. What advice does she give about
dealing with customers?
A. Don’t let them force you to agree to something
B. Don’t be too sympathetic towards them.
C. Don’t allow them to stay on the phone too long.
Part II: You win hear part of a radio programme in which a woman called Sylvia Short is interviewed about
her job. For question 9-18, fill in the missing words to complete the passage.
Sylvia studied(9)_______at University.

68
After University, Sylvia worked as a(10) _______in Italy.
The company which employs Sylvia is called(11) _______.
Sylvia worked for the company for (12) _______before coming the manager’s assistant.
Part of Sylvia’s job is to organize the(13) _______in newspaper and magazines.
Sylvia often has to deal with strange questions from (14) _______.
Sylvia’s boss has a radio show on Fridays on the subject of(15) _______.
Sylvia has written about her (16) _______for a new book on Britain.
Sylvia says that in the future she would like to be a(17) _______on television.
Last year, Sylvia enjoyed attending a(18) _______in Australia.
II. PHONETICS (2.0 pts)
a. Circle the word whose underlined part is pronounced differently from the rest
1. A. campus B. relax C. locate D. fashion
2. A. visit B. because C. answer D. poison
3. A. caught B. bought C. drought D. laughter
4. A. explosion B. expectation C. expense D. explore
5. A. claimed B. warned C. occurred D. existed
b. Circle the verb whose main stress syllable is different from the rest.
6. A. influential B. opportunity C. temperament D. expectation
7. A. government B. employment C. refusal D. redundant
8. A. technology B. information C. communicate D. irregular
9. A. history B. audience C. existence D. cinema
10. A. signature B. business C. dialogue D. delicious
III. Vocabulary and Grammar (8,2 pts)
a. Circle the best option to complete the sentences.
11. He’s got plenty of_______experience as he’s worked in that field already.
A. second-hand B. first-hand C. primary D. tertiary
12. There was a long line of people outside the cinema last week. They queued_______to see a very famous
film.
A. in B. on C. at D. up
13. James_______a lot of money when his grandfather passed away.
A. came into B. came up C. came over D. came up with
14. You are under no obligation to help as assistance is purely_______
A. free B. voluntary C. charitable D. donated
15. All his plants for his own business fell_______
A. through B. down C. off D. away
16. How much time you spend_______the web a day?
A. to surf B. surfing C. surf D. surfs
17. His_______arrival_______all members of the family.
A. unexpected…delighted C. unexpected…was delighting
B. expecting…delighted D. unexpected....delight
18. The school was closed for a month because of serious_______of fever.
A. outcome B. outburst C. outset D. outbreak
19. We expected him at eight but he finally_______at midnight.
A. came to B. turned out C. turned up D. came off

69
20. I keep putting_______the decision because I can’t make up my mind.
A. up B. down C. off D. on
21. Megan solved her computer problem quite_______. She happened to mention it to a friend who had had
the same problem told her what to do.
A. occasionally B. clumsily C. accidentally D. attentively
22. Bill Gates is the best known and most successfully_______in computer software.
A. pioneer B. navigator C. generator D. volunteer
23. _______he is rich and famous, he lives in modest house in the village.
A. Because B. however C. Therefore D. Although
24. When I joined the army, I found it difficult to_______out the orders from my superiors.
A. call B. carry C. miss D. take
25. She applied for training as a pilot, but that turned her_______because her poor eyesight.
A. back B. up C. over D. down
b. Fill a suitable preposition in each gap below.
26. We need to spread_______in a long line and search the whole field thoroughly.
27. You mustn’t let the bigger kids push you_______!
28. It didn’t take me long to cotton_______to the fact that Gareth was rather unpunctual.
29. Nathan was lucky enough to be born_______a very wealthy family.
30. The cathedral features_______a documentary about important buildings.
31. I don’t have a watch, but_______- a guess I would say it was about elven o’clock.
c. Replaced the underlined words with the most suitable word from the box.
Cherished dreaded mourned regretted resented loathed offended
Reproached stressed deplored

0. Sally held very dear the memory of her childhood in the country → cherished
32. The Prime Minister said he strongly disapproved of the behavior of the demonstrators.
33. David felt extremely worried about visiting the dentist.
34. Peter was very sorry about leaving his old job.
35. Neil grieved for the death of his mother and his father for many weeks.
36. I am sorry if I hurt the feelings of your sister
37. Brenda really felt a strong dislike for her new boss
38. Our teacher laid emphasis on the importance of regular study.
39. Jim strongly criticized me for not doing my fair share of the work.
d. Complete the sentences by finding one word which fits in all three spaces.
40-41-42. Let’s go down to the river. It’s a really nice________for a picnic.
I’m afraid I’m going to be late. I’m having a________of bother with my car.
The evening in Blackpool was the only bright________in an otherwise disappointing holiday.
43-44-45. I’m so tired I’m finding it difficult to keep my________on my work.
If you can cast your________back to lesson two, you’ll remember we were talking about body posture.
My daughter is so ill, so I’ve got a lot on my________right now.
46-47-48. The caterers didn’t turn up and the guest of honour was late, so we had to make the________of a
rather terrible situation.
I know you’re not very keen on art, but just do your________in the exam.
I didn’t get the job in the end, but it’s probably for________

70
49-50-51. Is long hair back in________?
I actually don’t know much about that________of architecture.
If I was a rock star, I’d make sure I was treated with________wherever I went.
V. READING (5,8 pts)
a. Fill a missing word in a blank to complete the passage.
The library of Congress in Washington, D.C, which has the largest (52) ________of books in the world, is
fighting a battle against paper deterioration. The page of old books, often yellowed and torn, sometimes
crumble when they are touched. The main culprit in the battle is the acidic paper - that has been used for
making books since the 19th century. Air (53) _______and moisture have added to the problem. Strangely, the
books that are most in danger of (54) ________are not the oldest ones. The paper in books produced before the
18th century was made from cotton and line rags, which are naturally low in acid.
In the 19th century, with widespread literacy bringing a (55) ________for a cheaper and more plentiful
supply of paper, the industry began using chemically treated (56) ________pulp for making paper. It is the
chemical in this paper that is causing today’s problem. France, Canada, and Austria are all doing (57) ____ in
new methods of deacidification. A new technology has been (58) ________recently that allows for mass
deacidification of thousands of books at the same time. It (59) ________less than microfilming and still
preserves books in their original form. It is hoped there will soon be (60) ________facilities all over the world
to preserve library book collections.
b. Circle the best option.
Have you ever asked yourself what you are working for?. If you have ever had the time to consider
This taboo question, or put it to others in moments of weakness or confidentiality, you (61) ________well
have heard some or all of the (62) ________. It’s the money, of course, some say with a smile, as if explaining
something to a child. Or it’s the satisfactory of a task well-done, the .sense of achievement behind the
clinching of an important (63) ________. I work as a bus conductor once, and I can’t say I (64) ________the
same as I staggered along the narrow path trying to (65) ________out tickets without falling over to someone’
clap. It’s the company of other people perhaps, but if that is the (66) ________, what about fanners? Is it the
conversation in the farmyard that keeps them captivated by the job? Work is power and sense of status say
those (67) ________have either attained these elusive goals or feel aggrieved that nobody has yet recognized
their leadership qualities. Or we can blame it all on someone else, the family or the taxman, I suspect, and I say
this under my breath, that most of us work rather as mr.Micawber lived, hoping for something to (68)
________up. We’ll win the pools, and tell the boss what we really think. We’ll scrape together the (69)
________and open that little shop we have always dreamed of, or go round the word, or spend more time in
the garden. One day, we’ll get that (70) ________we deserve, but until then at least we have something to do.
And we are so busy doing it that we won’t have time to wonder why.
61. A. might B. can C. will D. should
62. A. below B. rest C. following D. later
63. A. deal B. position C. job D. engagement
64. A. enjoyed B. wished C. hoped D. felt
65. A. make B. turn C. issue D. give
66. A. one B. case C. question D. problem
67. A. which B. that C. who D. whom
68. A. move B. turn C. ease D. end
69. A. resources B. opportunities C. riches D. money
70. ambition B. talent C. ability D. promotion

71
c. Read the following passage and then circle the correct answer.
Reading to oneself is a modem activity which was almost unknown to the scholars of the classical and
medieval worlds, while during the fifteenth century the term “reading” undoubtedly meant reading aloud. Only
during the nineteenth century did silent reading become commonplace. One should be wary, however, of
assuming that silent reading came about simply because reading aloud was a distraction to others.
Examinations of factors related to the historical development of silent reading have revealed that it became the
usual mode of reading for most adults mainly because the tasks themselves changed in character. The last
century saw a steady gradual increase in literacy and thus in the number of readers. As the number of readers
increased, the number of potential listeners declined and thus there was some reduction in the need to read
aloud. As reading for the benefit of listeners grew less common, so came the flourishing of reading as a private
activity in such public places as libraries, railway carriages and offices, where reading aloud would cause
distraction to other readers.
Towards the end of the century, there was still considerable argument over whether books should be used for
information or treated respectfully and over whether the reading of materials such as newspapers was in some
way mentally weakening. Indeed, this argument remains with us still in education. However, whatever its
virtues, the old shared literacy culture had gone and was replaced by the printed mass media on the one hand
and by books and periodicals for a specialised readership on the other. By the end of the twentieth century,
students were being recommended to adopt attitudes to books and to use reading skills which were
inappropriate, if not impossible, for the oral reader. The social, cultural and technological changes in the
century had greatly altered what the term “reading” implied.
71. Why was Reading aloud common before the nineteenth century?
A. people couldn’t read for themselves.
B. there were few places available for private reading
C. silent reading had not been discovered
D. people relied on reading for entertainment.
72. The development of silent reading during the last century indicated________.
A. a change in the status of literate people C. an increase in the number of books
B. a change in the nature of reading D. an increase in the average age of readers
73. Educationalist are arguing about________
A. The importance of silent reading
B. The amount of information yielded by books and newspaper.
C. The effects of reading on health
D. The value of different types of reading materials.
74. The mass media and specialized periodicals showed that________
A. standards of literacy had declined
B. readers’ interests had diversified
C. printing techniques had improved
D. educationalists’ attitudes had changed.
75. What is the writer of this passage attempting to do?
A. explain how present-day reading habits developed
B. change people’s attitudes to reading
C. show how reading methods have improved
D. encourage the growth of reading
V. Writing (6,0 pts)

72
b. Finish each of the sentences in such a way that it means exactly the same as previous sentence.
76. “Don’t forget to give me a ring tomorrow, Peter” said John.
→ John reminded_______________________________________________________.
77. Let’s invite Teddy to the party on Saturday.
→ She suggested_______________________________________________________.
78. “Be careful. Don’t go too near the edge of the cliff,”. My mother said to the boys.
→ My mother warned ________________________________________________.
79. The plane had hardly left the airport when a big problem occurred.
→ No sooner__________________________________________________________.
80. Regarding payment, most major credit cards are acceptable.
→ As far as___________________________________________________________.
c. Rewrite each of the sentence below
81. “Well done, Kate. You have passed the exam”, said Lonnie (congratulated)
_____________________________________________________________________
82. You can’t get to the village in winter because of the snow. (access)
_____________________________________________________________________
83. I can’t find the answer without a calculator (out)
_____________________________________________________________________
84. John did the composition badly because he was tired. (mess)
_____________________________________________________________________
85. Your son doesn’t seem to be capable of concentrating on anything (seems)
_____________________________________________________________________
d. 86-100. It has been said “Not everything that is learned is contained in books”. Compare and contrast
knowledge gained from experience with knowledge gained from books. Which is more important?. Why?.
Write about 200-250 words improve your opinion in this space.

73
SỞ GIÁO DỤC VÀ ĐÀO TẠO HÀ NỘI
KỲ THI TUYỂN SINH VÀO LỚP 10 THPT CHUYÊN
Năm học 2012-2013
Môn thi: Tiếng Anh (Điều kiện)
Ngày thi: 22 tháng 6 năm 2012
Thời gian làm bài: 120 phút
I. Vocabulary and Grammar (5,5 pts)
a. Circle the best option to complete the sentence
1. While doing a test, you should________ silent.
A. make B. keep C. do D. have
2. I________visit my grandmother in the countryside.
A. sometime B. sometimes C. something D. sometimes
3. Why don’t you________with your study school?
A. go in B. go at C. go by D. go on
4. Lan asked me not________- anybody her secret.
A. tell B. telling C. to tell D. told
5. How long ago________here?
A. did you come B. you came C. were you coming D. would you come
6. Do you know who ?
A. did she talk to B. she talked to C. talked to she D. she talked
7. The Internet is wonderful invention________ modern life.
A. to B. at C. of D. on
8. Of the two geologists, one is experienced and________.
A. the other are not B. the. other is not
C. another is inexperienced D. other lacks experience
9. I am familiar with that product, I don’t know________times I’ve seen it advertised on TV.
A. how many B. how often C. how D. how much
10. You didn’t watch the football match on TV last night, ________?
A. do you B. don’t you C. did you D. didn’t you
11. I’m sure the________in this book is quite clear to you.
A. explanation B. explain C. explaining D. to explain
12. The doctor suggested her________two aspirin each day.
A. to take B. taking C. to be taken D. being taken
13. It is snowing. Would you like to________on Saturday or Sunday?
A. skiing B. go to ski C. go skiing D. go ski
14. Ao dai________the traditional dress in our country.
A. be considered B. is considering C. has considered D. is considered
15. The Government should do something to help________.
A. the poors B. the poor ones C. poor D. the poor
16. If energy________inexpensive and unlimited, many things in the world would be different.
A. is B. were C. would be D. will be
17. They________each other for ages.
A. haven’t seen B. don’t see C. didn’t see D. won’t see
18. Kate congratulated________getting good marks in the entrance exam.

74
A. me of B. me in C. me on D. me for
19. It seemed like a good plan but when we carried________, it didn’t work.
A. out B. on C. through D. over
20. I thought I would sell my house, but________second thoughts I changed my mind and________the whole
I think I was wise.
A. in-in B. on-on C. on-in D. in-on
b. Give correct form of the verbs in the bracket.
21-22. When I (wake up) ________last night, it (rain) ________.
23. I haven’t written any letters home since I (come) ________here.
24. If we go on (waste) ________water, there will be a shortage of fresh water in a few decades.
25. I remember (watch) ________this film last month.
26. Wait for me, I (come) ________in a few minutes.
27. They didn’t let Jason (take) ________a picture here.
28. The children were frightened because it (get) ________dark.
29. After three years living in London, I am used to (drive) ________on the left.
30. Pupils are made (work) ________hard at school.
31. Do you feel like (watch) ________TV?
32-33. It’s time you (turn) ________on the central heating. It (get) ________colder.
34-35. The teacher would rather all of you (stop) ________ (make) noise now.
c. Give the correct form of the capital letters
36. Nam didn’t go to school yesterday because of his________ (ILL)
37. The Coswold area is an excellent place for an________weekend for a longer trip. (ENJOY)
38. I don’t like him because he is________ (TALK)
39. If you are________-with your essay, I suggest that you should rewrite it. (SATISFY)
40. Preserving natural resources is of great________ (IMPORTANT).
41. The waste from this factory is really________ (POISON).
42. He seems to be quite________about the future. He looks very disappointed. (HOPE)
43. I got some________news this morning. (SURPRISE)
44. Fortunately, the water in the lake near my house is________ (POLLUTE)
45. There is no________-proof that UFOs exists (SCIENCE)
d. Fill in each gap below with a suitable preposition.
46. Who looks________your baby sister when your mother goes on business?- My father.
47. In my family, lighting accounts________25 percent of the electricity bill every month.
48. Remember to turn________the taps before you get out of the bathroom.
49. We are going to have a party________Christmas Eve.
50-51. I always go to school________foot, but yesterday I went________my father’s car.
52. What are you doing________present?
53. If we go________wasting water, there will be a shortage of fresh water in a few decades.
54. I came________my former teacher while doing shopping yesterday.
55. Jackson can help us________the decoration for Tet holiday.
II. Reading (2,0 pts)
A. Circle the best option.
It’s sometimes said that there is (56) ________new in the world of fashion. I met Annabelle yesterday. She
was a well-known(57) ________during the seventies. When her children were in their early teens, they

75
enjoyed looking (58) ________her old photo albums. They found it hard to believe that she (59) ________
such strange clothes. Did people really use to think (60) ________trousers looked good? And those with Ugly
platform shoes. (61) , Annabelle noticed to her amusement that seventies (62) ________were in fashion again.
“I was planning to throw all my old clothes (63) ________but my daughter went to a party last week”, she
said, “and guess (64) ________she was wearing - that’s right, some of my old clothes!”
56. A. anything B. thing C. everything D. nothing
57. A. writer B. model C. architect D. playwright
58. A. at B. up C. on D. after
59. A. was wearing B. used to wear C. has worn D. wears
60. A. be flared B. flare C. flared D. to flare
61. A. Therefore B. Consequently C. Moreover D. However
62. A. styles B. forms C. uniforms D. formats
63. A. out B. in C. away D. aside
64. A. how B. which C. what D. that
b. Read the following information about different pen pals and do the tasks followed
A. Mary, 24 years old, comes from Scotland and would like to find a pen pal who comes from East Europe.
She likes playing the piano and listening to jazz music. She is interested in history but does not like discussing
politics.
B. Kim Lee, 19 years old, comes from Seoul, South Korea. Kim loves traveling and hopes to visit Great
Britain in the future. He would like a pen pal who is interested in discussing the differences between life in
Europe and life in Asia. He loves listening to pop music and playing football in his free time,
C. Pietro, 42 years old, comes from Argentina. He is a businessman and would like to find a pen pal who is
also a businessperson and lives in North America. He is married with three children and likes using the Internet
in his free time.
D. Helga, 31 years old, comes from Germany and speaks French, English and Russian. She would like a pen
pal who is interested in exchanging ideas about language learning. She does not like using computers for
learning and believes that language learning can only happen in a classroom.
E. Jennifer, 18 years old, comes from New Orleans in the United States. She is interested in discussing the
political differences between East Europe and North America. She loves riding her horse, Jackie, and listening
to jazz music.
F. Alessandro, 25 years old, comes from Rome. He is interested in finding a pen pal who speaks different
languages and can exchange ideas on using the computer for learning purposes. He likes playing tennis and
football in his free time. Which pen pal is the best for these people? Choose ONLY ONE pen pal for each
person. Write the correct letter A-F in the blanks
65. Petr Vladovic, 18, comes from Serbia and would like a pen pal who is interested in discussing the current
political situation in the world. He likes jazz and playing tennis in his free time. The best pen pal for Petr
would be________
66. Tom Synder, 36, is a businessman from Toronto, Canada. He would like to find a pen pal from a different
continent to discuss business practices and differences between his country and others. The best pen pal for
Tom would be________
67. Olga, 32, comes from Moscow, Russia. She is a historian and would like to find a pen pal who is interested
in discussing the history of Russia before the communist regime. She is not interested in comparing different
political systems. The best pen pal for Olga would be________

76
68. Jack, 27, is from London. His favourite pastime is learning languages. He goes to evening German and
French classes and uses his computer to improve his German and French by visiting Internet sites.
The best pen pal for Jack would be________
69. Stuart, 22, is from Dublin. He loves travelling and wants to visit Asia in the near future and would like a
pen pal who likes playing football and who can tell him about the differences between life in Asia and Ireland.
The best pen pal for Stuart would be________
70. Elisabeth, 35, comes from Sydney, Australia. She likes learning languages, but does not like using modem
technology. She is interested in finding a pen pal who also enjoys learning languages in a traditional manner.
The best pen pal for Elisabeth would be________
b. Read this passage carefully and then circle the best answer to each question:
Computer programmer David Jones earns £35.000 a year designing new computer games, yet he cannot find
a bank prepared to let him have a cheque card. Instead, he has been told to wait another two years, until he is
18. The 16 year - old works for a small firm in Liverpool, where the problem of most young people of his age
is finding a job. David’s firm releases two new games for the expanding home computer market each month.
But David's biggest headache is what to do with his money. Despite his salary, earned by inventing new
programs within tight schedules, with bonus payments and profit-sharing, he cannot drive a car, take out a
montage, or obtain credit cards. He lives with his parents in their council house in Liverpool, where his father
is a bus driver. His company has to pay £ 15 0 a month in taxi fares to get him the five miles to work and back
every day because David cannot drive. David got his job with the Liverpool - based company four months ago,
a year after leaving school with six O-levels and working for a time in a computer shop. “I got the job because
the people who ran the firm knew I had already written some programmes”, he said. "I suppose £ 35,000
sounds a lot but actually that's being pessimistic. I hope it will come to more than that this year". He spends
some of his money on records and clothes, and gives his mother £20 a week. But, most of his spare time is
spent working. "Unfortunately, computing was not part of our studies at school", he said. "‘But I had been
studying it in books and magazines for four years in my spare time; I knew what I wanted to do and never
considered staying on at school. Most people in this business are fairly young, anyway”. David added: “I
would like to earn a million and I suppose early retirement is a possibility. You never know when the market
might disappear”.
71. David Jones is________.
A. a computer shop assistant B. a computer producer
C. a computer programmer D. an O-level student
72. David’s greatest problem is________.
A. earning more than £35,000 a year
B. inventing computer games
C. spending the money he earns
D. learning to drive
73. He was employed by the company because________.
A. he had worked efficiently in a computer shop
B. he had written some computer programs
C. he worked very hard
D. he had learnt to use computers at school
74. He left school after taking O-levels because________.
A. he did not enjoy school
B. he wanted to work with computers and staying at school did not help him

77
C. he was afraid of getting too old to start computing
D. he wanted to earn a lot of money
75. Why does David think he might retire early? -________.
A. He has to be young to write computer programs
B. He wants to stop working when he is a millionaire
C. He thinks computer games might not always sell so well
D. He thinks his firm might go bankrupt
III. Writing ( 2,5 pts)
a. Circle the best option A, B, C or D to complete the sentences.
76. The doctor advised him________.
A. for taking more exercise to lose weight
B. to him that he should do more exercise to lose weight
C. doing more exercises to lose weight
D. to take more exercise if he wanted to lose his weight
77. James________.
A. had to speak to his lawyer, he signed the contract.
B. spoke to his lawyer before being signed the contract.
C. will not sign the contract until he has spoken to his lawyer.
D. doesn’t sign the contract till he spoke to his lawyer.
78. All the switches________
A. must be turned off before we leave the workshop
B. is to be turned off before we leave the workshop
C. have be turned off before we leave the workshop
D. should be turned off before leaving the workshop
79. “I will do the shopping today” said Ba.
Ba said that________.
A. he would do the shopping that day
B. he will do the shopping today
C. he would do the shopping today
D. he will do the shopping that day
80. “Do you review your lessons in the evening?” Miss Anderson asked Tim.
Miss Anderson wanted to know________.
A. if Tim reviews his lessons in the evening
B. if Tim reviewed his lessons in the evening
C. Tim reviewed his lessons in the evening or not
D. Tim reviews his lessons in the evening or not
b. Circle the incorrect parts in the following sentences A, B, C or D and correct them.
81. We (A) are looking (B) forward to hear (C) from you. (D)
82. We studied (A) very hard (B) for the lesson, because (C) we passed the exam very easily. (D)
83. Nancy asked me (A) when (B) my school (C) vacation start. (D)
84. Jane wishes (A) you can help (B) her solve (C) this financial problem. (D)
85. There were (A) so much (B) people trying (C) to recognize each other (D) at the anniversary.
c. Arrange the words and phrases given in order to make meaningful sentences.
86. We/ the/ home/ at about/ left/ 9.15 am/ go/ to/ station/ to/. /

78
____________________________________________________________________
87. The train/ at/ Victoria/ just/ arrived/ before/ 10.45 am/ and then/ station// we/ a bus/ to/ the/ caught /London
Eye/. /
____________________________________________________________________
88. When/ at/ the/ London Eye/ we/ only/,/ we/ had to/ arrived /queue/ for/ about/ ten minutes/. /
____________________________________________________________________
89. I /scared/ was/ about/going/,/ on/ the cabin/ really/ but/I /went on/ still/. /
____________________________________________________________________
90. It/ really/,/ and/ was/ love/I /would/ to/ go on/ nice/ it /again/. /
____________________________________________________________________
91. The wheel/ very/ slowly/ and/ it/ took/, / about/ turned/ half an hour/ to go/ right/round/. /
____________________________________________________________________
92. When/ we/ the cabin/ we/ went on/,/ came off /a/ river trip/ the Embankment/ from/. /
____________________________________________________________________
93. We/ see/ Big Ben/boat/ and/from/ Houses of Parliament/ the/ could/the/ . /
____________________________________________________________________
94. The weather/ lovely and hot/ was/ we/ a picnic/ in/ so/ had/ the park /. /
____________________________________________________________________
95. We/ a bus/ back/ to/ Victoria station/ caught/ and/ arrived/ back/ in/ Crawley/ about/ 8.00 pm/. /
____________________________________________________________________
96.I/ felt/ it/ a long day/ walking/ but/ ,/ was/I/ really/it/ enjoyed/. /
____________________________________________________________________
d. Rewrite each of the following sentences in such a way that it means the same as the sentence printed
before it. Do not alter the given words in any way.
97. I don’t know the girl’s name. She helped me yesterday. (who)
____________________________________________________________________
98. He looks really like his father in many ways. (takes)
____________________________________________________________________
99. Nobody has used these rooms for a long time. (been)
____________________________________________________________________
100. “Could you please ring back in half an hour?” the receptionist said. (required)
____________________________________________________________________
The end

79
SỞ GIÁO DỤC VÀ ĐÀO TẠO HÀ NỘI
KỲ THI TUYỀN SINH VÀO LỚP 10 THPT CHUYÊN
Năm học 2012-2013
Môn thi: Tiếng Anh (Chuyên)
Ngày thi: 23 tháng 6 năm 2012
Thời gian làm bài: 120 phút
I. Phonetics( 2,5 pts)
a. Circle the word whose underlined part is pronounced differently from the rest
1. A. meant B. steak C. bread D. head
2. A. extensive B. pesticide C. institute D. congratulate
3. A. government B. tropical C. correspond D. economic
4. A. chapter B. yacht C. manage D. panel
5. A. ancient B. evidence C. practice D. celebrate
b. Circle the word whose main stressed syllable, is different from the rest
6. A. photography B. minority C. amateur D. heroic
7. A. comfortable B. politician C. uncontrollable D. practicality
8. A. managerial B. determination C. unbelievable D. inability
9. A. mysterious B. embroider C. volcano D. earthquake
10. A. infrastructure B. parade C. distinguish D. experience
II. Vocabulary and Grammar (7,0 pts)
a. Circle the best options A, B, C or D to complete the sentences.
11. The team won the championship four years_________.
A. running B. passing C. following D. rotating
12. Home is a base from which we can go into the world with_________.
A. confide B. confident C. confidence D. confidently
13. He can’t rent this second-hand car, _________buy a new one.
A. let alone B. unmentioned C. not to reckon D. unconsidered
14. The factory paid_________nearly a million pounds to their employees who were injured in the explosion.
A. on B. out C. back D. in
15. His father retired early_________his health.
A. on behalf of B. ahead of C. on account of D. in my view
16. She went_________a bad cold just before the new school year.
A. down with B. in for C. over D. through
17. Although they had only been invited for lunch, they_________until supper time.
A. stayed on B. stayed out C. stayed up D. stayed in
18. England_________2-2 with Germany in the first round.
A. tied B. pointed C. equaled D. scored
19. He thought there was a snake nearby when he heard the_________noise.
A. hissing B. crackling C. squealing D. humming
20. When you put some salt in hot water, it will_________.
A. melt B. dissolve C. soften D. disappear
21. Nani is a_________member of the Manchester United football club.
A. long-lasting B. long-term C. long-standing D. long-suffering
22. The_________of the headlights from the car behind him blinded Mr. Baker momentarily.

80
A. lightness B. brightness C. greatness D. glitter
b. Give correct forms of the words in the brackets.
23. _________is the one skill that can grant you the opportunity to get a good job. (Lead)
24. The talks were totally_________. We didn’t reach agreement on anything at all. (product)
25. The boy was very violent and his parents found him_________. (manage)
26. I want to study_________at university. (engineer)
27. The monument was erected in_________of the soldiers who fell for posterity. (remember)
28. He wrote so many_________sentences in his essay that I couldn’t understand it. (grammar)
29. You mustn’t leave your luggage_________for even a moment on the train. (attend)
30. The accommodation was_________and comfortable. (space)3
c. Use a verb in column A with an adverb particle in B to form a phrasal verb and fill in each sentence.
Each verb can be used only once. There are two extras in each column.
A B
Get take come break look Into out through of to down
let sit account run listen off for up round

31. Linna’s new book is due to_________next month. I wonder what the critics will think of it.
32. Scientists are mystified by the sudden rise in global temperature. They are unable to_________it.
33. I don’t want my colleagues to_________wind_________the fact that I’m leaving.
34. John works in that office. I quite often_________him in the streets round here.
35. Katehas a very pleasant manner. I’m sure the children will_________her at once.
36. Although the students_________the entire lecture, none of them is really paying attention to it.
37. The schools will_________next week. It’s almost holiday time.
38. Brian’s parents have high expectations of him and he is determined not to_________them_________.
III. Reading (5,5 pts)
a. Fill a suitable word in each blank to complete the passage
Natural disasters are often frightening and difficult for US to understand, because we have no (39)____ over
when and where they happen. What we can control is how prepared we are. Places that are more likely to have
(40) __________disasters, such as the earthquake-prone Pacific (41) __________of Fire, or coastal (42)
__________vulnerable to hurricanes, require accurate methods of predicting disasters and warning the public
quickly. Once people have been (43) __________, evacuation routes must be provided so that they can all (44)
__________quickly and safely, even if they travel on foot. People need to be educated on the risks in their
area, and what to do when a disaster strikes. After a disaster, even if no one has died, there is a lot of (45)
__________to people homes, farms and workplaces that must be repaired. This takes a lot of time and money
to fix, and a country damaged by a disaster usually needs a large amount of international (46) __________to
get better.
b. Circle the best option A, B, C or D to complete the passage.
Smart shoes that adjust their size throughout the day could soon be available. A prototype of such a shoe has
already been produced and a commercial (47) __________may be in production within a few years. The shoe
contains sensors that constantly (48) __________the amount of room left in it. If the foot has become too
large, a tiny valve opens and the shoe expands slightly. The entire control system is about 5square mm and is
(49) __________inside the shoe. This radical shoe meets a need because the volume of the average foot can
change by as much as 8% during the course of the day. The system is able to learn about the wearer's feet and
(50)a picture of the size of his or her feet throughout the day. It will allow the shoes to change in size by up to

81
8% so that they always fit exactly. They are obviously more comfortable and less likely to cause blisters. From
an athlete’s point of view, the shoes can help improve (51) _______a little, and that is why the first (52)
__________for the system is likely to be in a sports shoe. Eventually, this system will find a (53)
__________in other household items, from beds that automatically change to fit the person sleeping in them to
power tools that shape themselves to the user’s hand for better grip. There is no reason why the system couldn't
be adapted for use in hundreds of consumer (54) __________.
47. A. assortment B. version C. style D. variety
48. A. prove B. confirm C. inspect D. check
49. A. located B. sited C. established D. laid
50. A. build up B. pickup C. grow up D. set up
51. A. achievement B. performance C. success D. winning
52. A. purpose B. exercise C. use D. operation
53. A. function B. part C. way D. place
54. A. commodities B. possessions C. goods D. objects
c. Choose the paragraphs A-F to fit into the numbered gaps in the following magazine article.
A. It’s not that I’m suggesting that sailors should go back to enduring every hardship. It’s always been
important to me that my boats have a coal stove for warmth and dryness and cosy berths for sleeping. But why
go cruising at all if every sail sets, rolls and fastens itself?
B. Back on land, however, it is a sad fact that the very antiquity of classic boats means that they need a lot of
looking after. When I had a bad injury to my back, I realised that my IS-year love affair with her had to end.
Searching for a younger replacement produced no credible contenders, so I decided to build a new boat from
scratch.
C. In her timeless serenity, she is the living proof that it works; that there is no need to follow current fashions
to find satisfaction, and that sometimes it pays to listen to the lessons of history.
D. The next version was nearly right and by the time the final one appeared, the form was perfect. The
completed boat has now crossed the North Atlantic and has won four out of her first six racing starts.
E. Perhaps I am, though I doubt it. This boat has benefited from all the magic of old fashioned boat design, but
it would have been a much harder job without the advances of modem know-how.
F. For me a boat should always be a boat and not a cottage on the water. When I bought an earlier boat, Hirta,
in which I circumnavigated Britain for a TV race series, the previous owner observed that she had every
comfort, but no luxury. During my long relationship with her, Hirta taught me how wise he was.
THE BOAT OF MY DREAMS
The best boat design should combine old and new, says Tom Cunliffe. And he put it into practice in his own
craft, ‘The Westerman’.
This week, the Summer Boat Show in London is resplendent with fine yachts, bristling with new
technology. Nearly all are descendants of the hull-shape revolution that took place 25 years ago. By contrast,
my own lies quietly on a tidal creek off the south coast. She was designed last year but, seeing her, you might
imagine her to be 100 years old and think that her owner must be some kind of lost-soul romantic.
55.___________
It has to be said, however, that despite being an indispensable tool in current design methods and
boatbuilding practice, sophisticated technology frequently insulates crews from the harsh realities of maritime
life. These are often the very realities they hoped to rediscover by going to sea in the first place.
56. ___________

82
The occasional battle with flapping canvas is surely part of a seaman’s life. And for what purpose should we
abandon common sense and move our steering positions from the security of the aft end to some vulnerable
perch half-way to the. bow? The sad answer is that this creates a cabin like that of an ocean liner, with space
for a bed larger than the one at home
57. ___________
Her sails were heavy, and she had no pumped water, no electricity to speak of, no fridge, no central heating,
no winches, and absolutely no electronics, especially in the navigation department, yet she was the kindest,
easiest boat that I have ever sailed at sea.
58. ___________
The Westerman has never disappointed me. Although Nigel Irens, the designer, and Ed Burnett, his right-
hand man, are adept with computer-assisted design programs, Irens initially drew this boat on a paper napkin,
and only later transferred his ideas to the computer. After this had generated a set of lines, he carved a model,
just as boatyards did in the days of sail. Together we considered the primary embryonic vessel, and then fed
the design back into the electronic box for modification.
59. ___________
Her appearance is ageless, her motion at sea is a pleasure and her accommodation, much of it in reclaimed
pitch pine, emanates an atmosphere of deep peace. Maybe this is because she was drawn purely as a sailing
craft, without reference to any furniture we might put into her. That is the well- tried method of the sea.
60. ___________
Constructed in timber treated with a penetrating glue, she is totally impervious to water. Thus she has all the
benefits of a glass fiber boat yet looks like, feels like and sails like the real thing.
IV. Writing (5,0 pts)
a. Circle the incorrect parts in the following sentences A, B, C or D and correct them.
61. However cheap it is, (A) the poor quality (B) products can not always appeal to (C) customers(D).
62. The success(A) of the Intel ISEF project depends in (B) the professors’ instructions(C) as well as
the students’ work. (D)
63. Many foreign visitors(A) attracts(B) by Nha Trang, which (C) has a long (D) and beautiful beach.
64. Knowing that (A) it would be helpless(B) to continue working (C) for a nearly bankrupt company,
Luise decided to go away and find another type (D) of employment.
65. When (A) radio program became (B) popular, approximately (C) around 1925, many people stopped
attending (D) movies.
b. Finish each of the sentences in such a way that it means exactly the same as the sentence printed before it
66. We’d prefer you not to smoke.
→ We’d rather_________________________________________________________
67. Treating her like that was unfair.
→ She didn’t__________________________________________________________
68. It is quite pointless to complaint about him.
→ There’s____________________________________________________________.
69. Apart from David, everyone else at the meeting was a party member.
→ With______________________________________________________________
70. Contact the Student Service Office if you have any further problems.
→ Get_______________________________________________________________
c . Rewrite each of the following sentences in such a way that it means the same as the sentence printed
before it. Do not alter the given words in any way.

83
71-72.1 think that you’ve misunderstood the situation. (stick)
_____________________________________________________________________
73-74. Our little son is really looking forward to going to Disney World. (thought)
_____________________________________________________________________
75-76. Although she was beautiful and intelligent, she didn’t win the Miss World competition. (Despite)
_____________________________________________________________________
77-78. We must accept the fact that we have lost the golden opportunity. (terms)
_____________________________________________________________________
79-80. You didn’t tell me you were going to be away for a whole month. (neglected)
_____________________________________________________________________

---The end---

84
THPT CHUYÊN HÀ NỘI-AMS TERDAM
ĐỀ THI THỬ VÀO LỚP 10 THPT
Năm học 2011-2012
Môn thi: Tiếng Anh (Không Chuyên)
Thời gian làm bài: 120 phút
I. PHONETICS
a. Choose the words whose main stress syllable is different from the rest.
1. A. farmer B. farewell C. factory D. fairy
2. A. cattle B. country C. canal D. cover
3. A. money B. machine C. many D. mother
4. A. borrow B. agree C. await D. prepare
5. A. Government B. condition C. Parliament D. fortunate
b. Choose the words whose underlined part is pronounced differently from the rest
1. A. mysterious B. flying C. only D. mystery
2. A. landing B. tall C. talking D. falter
3. A. car B. park C. farm D. warm
4. A. curtain B. urgent C. hurry D. surface
5. A. row B. power C. slow D. show
II. VOCABULARY AND GRAMMAR
A. Circle the best one and write it in the blank to complete the sentences.
1. I like to see “Match of the Day” on Saturdays. That’s my_________
A. popular B. favorite C. preferable D. likeable
2. Her teaching experience gives her an_________when she works with children.
A. advantage B. advantaging C. advancement D. advanced
3. It’s an entertaining and_________documentary.
A. inform B. informed C. informative D. informal
4. The boy often responds to stress_________sick.
A. getting B. by getting C. with getting D. to get
5. York, _________we visited last year, is a nice old city.
A. that B. where C. who D. which
6. My shoes are dirty. I’d better take them_________before I come in.
A. up B. off C. away D. in
7. The hat only partially protected his face_________the sun.
A. from B. for C. with D. off
8. Her uncle suggested that she should_________a job in a bank.
A. get B. to get C. getting D. gets
9. The room was_________only by a single 40-watt light bulb.
A. light B. lighten C. lit D. lead
10. It was the highest election turnout in_________memory.
A. recently B. regularly C. regular D. recent
11. He’s only 17 and_________not eligible to vote.
A. therefore B. because of C. since D. now that
12. She replaced Jane Scott_________Managing Director.
A. for B. like C. to D. as

85
13. what can be done to keep business_________?
A. profit B. profitable C. profitably D. profitability.
14. He is the person _________friends trust him.
A. who B. his C. that D. whose
15. Everyone felt tired and hungry, _________they sat down and took a rest.
A. because B. so C. although D. so that
16. He has already been_________as a candidate.
A. called B. said C. remembered D. nominated
17. Stop behaving_________a three-year-old child.
A. like B. as C. of D. toward
18. It was the biggest_________of Vesuviouus for some years.
A. erupt B. eruption C. erupting D. erupts
19. Florida_________Georgia to the North.
A. is bordered B. borders C. frontiers D. boundaries
20. I must_________you that some off these animals are extremely dangerous.
A. warn B. suggest C. advice D. ask
b. Complete the passage with the correct tense form of the verb in brackets.
Mike Barette is 62 years old. He (1. Retire) _________2 years ago and since then he (2. Prepare) ______ to
go travelling. He (3.spend) _________a year travelling around Australia with his wife, Lucy. Mike and Lucy
are examples of a new kind of traveller; “grey voyagers”. Mike says “It’s something we (4. Want)___ to do for
many years, but we (5.not have) _________the opportunity. When we (6. Be) _________younger, there was
the family to consider, and of course when you have a career and a full-time job, you just can’t afford to take
12 months off to go travelling!”.
Last month, Mike (7. Buy) _________a large van, and since then he (8. Convert) _________it into a
motorized caravan with beds and a kitchen. It (9. Take) _________-another two months to finish it and then
we are off. “There are so many places we want to visit”, Explains Mike. We plan to travel in a big circle,
clockwise, around Australia. We (10. spend) _________at least a year travelling around Australia.
c. Underline and correct the mistakes in the following sentences.
1. Both the driver and the passengers was injured in the accident.
_________________________________________________________________
2. They neither have a fridge nor a stove in their new apartment.
_________________________________________________________________
3. I need both fine brown sugar as well as powdered sugar to bake a Hawaiian cake.
_________________________________________________________________
4. Either Mr, Brake or his secretary are meeting you at the airport.
_________________________________________________________________
5. Not only Mai plays the guitar well but she also sings beautifully.
_________________________________________________________________
6. The students not coming to the class yesterday explained their absence for the teacher.
_________________________________________________________________
7. My friend lives in a small village situating on the banks of the Red River.
_________________________________________________________________
8. The sunlight which comes through the window wakes up me early every morning.
_________________________________________________________________

86
9. I do not satisfy with the security conditions of your office.
_________________________________________________________________
10. Corn was one of the agricultural productions which were introduced to the European settlers the Indians.
_________________________________________________________________
D. Give the correct form of the word given in bracket.
1. His brother is very (1 .know)_________about the Asian Games.
2. It’s an opportunity for young people to (2.depth) _________their understanding of the world.
3. We should provide special services for (3. Able) _________athletes.
4. (4.Sun) _________energy is not only (5.limmit) _________but also clean and safe.
5. The photographs which were published in the newspaper were (6. Extra) _________.
6. It’s difficult to prove the (7.exist) _________of life in other planet.
7. The (8. Nature) _________are concerned about the (9. Protect) _________of environment.
8. Water (10. Conserve) _________is necessary for hydro power stations.
III. READING
a. Fill in each blank with a suitable word to complete the passage.
The story of jeans (1) _________almost 200 years ago. People in Genova, Italy, made pants. The cloth made
in Genova (2) _________called “jeanos”. The pants were called “jeans”. In 1950, a salesman in California
began selling pants (3) _________of canvas. His name was Levi Strauss. (4) _________they were so strong,
“levi’s pants” became popular with good miners, farmers and cowboys. Six years (5) ____, levi began making
his pants with a blue cotton cloth called denim. Sooner later, factory workers in the United States and Europe
began (6) _________his pants. Young people usually did not wear them.
In the 1950s, two people helped (7) _________jeans popular with teenagers: Elvis Presley, the King of Rock
and Roll, and James Dean, a movie star. Elivs (8) _________tight jeans. Most parents (9) ________ like Elvis
or his music, but teenagers loved him and started to dress (10) _________him. In “Rebel without a Cause”
James Dean wore jeans. He was a hero to many young people.
b. Read the passage carefully.
Quit Smoking Today
Some people find it easy to quit smoking. However, most people find it very difficult. If you are thinking
about giving up, here are three good reasons.
Protect your health
Smoking causes a number of serious disease. For example, it can cause emphysema, which is a disease that
slowly destroys the lungs. According to the United States Department of Health, most long-term smokers have
some degree of emphysema. Smoking can also cause lung cancer. In fact, according to a recent study,
compared with non-smokers, smokers are 10 times more likely to die from lung cancer.
And it is never too late. Doctors say that if you give up smoking now, your body will start to repair itself and
you might be able to avoid these disease.
Protect your family
Your smoking can harm other people. Breathing in other people’s smoke is called passive smoking. Apart
from simply being unpleasant, passive smoking is dangerous for the people around you. It is now well
established that there is a link between passive smoking and heart disease. According to studies, children
whose parents smoke are more likely to suffer from asthma. This is why smoking is now not allowed in many
public places. So, by quitting smoking, you are contributing to the health and well-being of the people around
you, including your own family.
Improve your looks.

87
Perhaps you should simply quit for the sake of your physical appearance. If you are a smoker, you probably
already know that smoking affects the look of your skin and teeth. Smoking stains your teeth yellow. It makes
your skin look grey because smoking reduces the blood flow to your skin, starving it of oxygen. Also,
according to a recent study published by the Office of Smoking and Health, wrinkles around smokers’ eyes
and mouth appear much earlier than they do around those of non-smokers of the same age. In short, smoking
makes you look older and less attractive. So speak to your doctors about quitting today, for the sake of your
health, your family and your looks!
Decide which the following statements are True (T), False (F) or no information (NI)
1. Giving up smoking is harder for some people than for others,
2. Although smoking harms your body, if you stop smoking in time, your body can recover.
3. Smoking is even more harmful to the people around a smoker than it is to the smoker.
4. The children of smokers are more likely to have some health problems than the children of non-smokers.
5. Smoking should not be allowed in public areas.
6. Most airlines do not allow smoking.
7. Smokers may thing they look cool, but they don’t.
8. If you want to have healthy skin, you should not smoke.
IV. Writing.
Rewrite the following sentences by using the words given in such a way that it means the same as the one
given.
1. “Are you leaving today or tomorrow?”, asked his mother.
→ His mother wanted to know________________________________________
2. “I’m addicted to the Internet and I’m chatting with my aunt now”, said the boy.
→ The boy said that________________________________________________
3. He started designing clothes more than twenty years ago.
→ He has________________________________________________________
4. They have just introduced a new style of jeans.
→ A new style____________________________________________________
5. Anne is pleased to meet her aunt again soon.
→ Anne is looking_________________________________________________
6. I am sorry I can’t speak English perfectly.
→ I wish_________________________________________________________
7. I can’t take you to the airport because I don’t have a car.
→ If_____________________________________________________________
8. We postponed the picnic because the weather was bad.
→ The weather was_________________________________________________
9. You must do this exercise carefully.
→ This___________________________________________________________
10. “Please don’t disturb me”, he said.
→ He told me______________________________________________________
11. It is not certain that Jones will get the job.
→ It is open________________________________________________________

---The end---

88
THPT CHUYÊN HÀ NỘI-AMSTERDAM
ĐỀ THI THỬ VÀO LỚP 10 THPT
Năm học 2011-2012
Môn thi: Tiếng Anh (Chuyên)
Thời gian làm bài: 120 phút
I. PHONETICS
a. Choose the word whose main stressed syllable is different from the rest
1. A. Vietnamese B. equipment C. understand D. volunteer
2. A. Aborigine B. necessary C. preference D. concentrate
3. A. postpone B. religion C. potential D. record (n)
4. A. politics B. economics C. rhetoric D. Catholic
5. A. employee B. television C. questionnaire D. picturesque
b. Choose the word whose underlined part is pronounced differently from the rest
1. A. desperate B. irritate C. grateful D. frustrate
2. A. beloved B. naked C. sacred D. learned
3. A. houses B. faces C. horses D. places
4. A. surplus B. surgery C. surprise D. surface
5. A. pizza B. lizard C. puzzle D. muzzle
II. VOCABULARY AND GRAMMAR
а. Circle the best one A, B, C, or D to complete the sentence.
1. Many music fans find him_________with the thought-provoking questions.
A. unforgettable B. unforeseen C. unfortunate D. unformed
2. Give Ann a call; she needs_________up.
A. encouraging B. moving C. laughing D. cheering
3. The weather condition in the South is favorable to agriculture, while_________the North is not quite so.
A. weather of B. weather in C. that in D. that
4. _________his brother’s generous help, he_________in such high social position.
A. With/wouldn’t have been B. with/ won’t be
C. But for/wouldn’t be D. Apart from/ wouldn’t have been
5. You should take her age_________consideration.
A. into B. for C. in D. to
6. She is trying to contact_________her close friend. She has lost contact_________her for five years.
A. with/with B. X/ with C. at/with D. to/to
7. The boy denied_________the window.
A. broken B. to break C. breaking D. break
8. He arrived at the hotel with only_________luggage.
A. a little B. a few C. little D. few
9. His poems were published under the_________of “Love and Reason”
A. headline B. title C. name D. subtitle
10. This shop is selling clothes at_________prices.
A. competed B. competing C. competition D. competitive
11. Children with_________diseases should not be allowed to go to school.
A. contact B. constant C. infectious D. influential
12. Ann is in a terrible state. Her house was_________last night.

89
A. robbed B. stolen C. burgled D. broken in
13. Dinner will be ready soon. Can you please_________the table?
A. settle B. lay C. make D. put
14. For the sake of his future career as an actor, he didn’t want to become_________as a detective.
A. thought B. named C. marked D. labeled
15. The sea is as_________as a mill-pond today.
A. blue B. deep C. calm D. warm
b. Fill in each blank with a suitable preparation or particle
1. Why did you leave_________the second question on your examination?
2. We got a cheque_________$500 in the post today.
3. How did you come_________that scratch on your cheek?
4. A good citizen is always loyal_________his government.
5. The airplane crashed into the sea about one kilometer_________the shore.
6. We’re hoping to get_________for a few days at Easter.
7. I’m sorry but Dr.Ho sees patients_________appointments only.
8. A gang of boys were setting_________fireworks in the street.
9. The chemical gives_________toxic fumes, so be extremely careful when using them
10. I do feel_________you, honestly!
c. Complete each of the sentences with the same phrasal verbs that has two different, meanings.
Be sure to use the correct form of verbs.
1. (a) Every 18-year-old young man will be_________before he goes to University.
(b) Please_________me_________when you are free.
2. (a) The computer_________all of sudden.
(b) David_________as a result of overwork.
3. (a) The new driver could not_________her car in a small car park.
(b) All should_________the committee’s good decisions.
4. (a) I will_________you on Sunday.
(b) The people are_________to donate money for the Earthquake victims,
5. (a) Business_________under the new management.
(b) Girl students usually_________a language faster than boy students.
d. Give the correct form of the word in bracket.
1. The old fishing village is very (picture) _________.
2. Mary gave the child a (hand) _________of sweets.
3. Did you really have to behave with such (severe) _________to the little girl?
4. ITỈ resign if you continue to (regard) _________everything I say.
5. The list is not intended to be (exhaust) _________.
6. After the children had gone back to school, the house was (compare) _________quiet.
7. (Accompany) _________children will not be allowed to cross busy roads.
8. You need to release the (tense) _________in these shoulder muscles.
9. The soil has been (poor) _________by erosion.
10. They were impressed by the power of her (argue) _________.
e. The following passage contains 10 mistakes. Underline and correct them in the space provided below.
The first has been done as an example.
0. have been → are

90
Every year earthquakes have been responsible for a large number of dead and a vast amount of destructions
in various parts of the world. Most of these damaged earthquakes occur neither in a narrow belt which
surrounds the Pacific Ocean or in a line which extends from Burma to the Alps in Europe. Some of the
destructions is direct caused by the quake themselves. An example of this was the collapse of buildings as the
result of vibration. Other damage results in landslides, tsunamis (large ocean waves) or major fires which are
which are initiating by the quake.
1_________ 6_________
2_________ 7_________
3_________ 8_________
4_________ 9_________
5_________ 10________
III. READING
Read the following passage and decide which answer (A, B, C, or D) best fits each gap.
When faced with some new and possible bewildering technology change, most people (51) _________in
one of two (1) _________. They either recoil from anything new, claiming that it is unnecessary, or too
(2) _________or that it somehow makes life less than (3) _________. Or they learn to (4) _________to the
new invention and eventually (5) _________how they could possibly have existed without it. (6) ________
computers as an example. For many of us, they still present a (7) _________to our freedom and give us a
frightening sense of a future in which all (8) _________will be taken by machines. This may be because they
seem mysterious and difficult to understand. Ask most people what you can (9) _________a home computer
for, and you usually get (10) _________answers about how They give you information’. In fact, even those of
us who are familiar with computer and use them in our daily work, have little idea of how they (11)
_________. But it does not take long to learn how to operate a business programme, even if things
occasionally go wrong for no apparent reason. Presumably, much the same happened when telephone and
television became (12) _________. What seems to alarm most people is the speed of (13) __ change, rather
than change itself. And the (14) _________that are made to new technology may well have a point to them,
since change is not always an improvement. As we discover during power cuts, there is a lot to be said for the
oil lamp, the coal fire, and forms of entertainment, such as books or board games, that don’t have to be (15)
_________into work.
1. A. Moments B. kinds C. Ways D. Types
2. A. Complicated B. Much C. Obscure D. Tiresome
3. A. Formerly B. Lively C. Personal D. Human
4. A. Adapt B. React C. Conform D. Use
5. A. Decide B. Wonder C. Suppose D. Admit
6. A. Discuss B. Propose C. Take D. Thus
7. A. Hazard B. Risk C. Control D. Threat
8. A. Measures B. Decisions C. instructions D. Chances
9. A. Run B. Apply C. Learn D. Use
10. A. Vague B. Such C. Up with D. Hundreds
11. A. Are B. Work C. Manage D. Consist
12. A. In existence B. Widespread C. Through D. Extensive
13. A. Technological B. Machinery C. Physical D. Future
14. A. objections B. Appliances C. Criticisms D. Fears
15. A. Connected B. Batteries C. Plugged D. Wired

91
b. Read the passage carefully
A Japanese construction company plans to create a huge independent city-state, akin to the legendary
Atlantics, in the middle of the Pacific Ocean. The city, dubbed “Marinnation”, would have about one million
inhabitants, two airports, and possibly even a space port. Marinnation, if built, would be a separate country but
could serve as a home for international organisations such as the United Nations and the World Bank.
Aside from the many political and social problems that would have to be solved, the engineering task
envisaged is monumental. The initial stage requires the building of a circular dam eighteen miles in diameter
attached to the sea bed in a relatively shallow place in international waters. Then, several hundred powerful
pumps, operating for more than a year, would suck out the sea water from within the dam. When empty and
dry, the area would have a city constructed on it. The actual land would be about 300 feet below the sea level.
According to designers, the hardest task from an engineering point of view would be to ensure that the dam is
leak proof and earthquake proof.
If all goes well, it is hoped that Marinnation could be ready for habitation at the end of the second decade of
the twenty-first century. Whether anyone would want to live in such an isolated and artificial community,
however, will remain an open question until that time.
Choose the best answer A, B, C or D.
1. According to the text, Marrination, when built, will be a(an) _________city.
A. Underground B. Underwater C. Marine D. legendary
2. The word ‘akin’ in paragraph 1 could be best replaced by_________.
A. Similar B. Next C. Likely D. close
3. Which of the following would NOT be a problem for the construction plan?
A. Social B. Political C. Financial D. engineering
4. Which of the following is NOT true according to the text?
A. Marinnation will be located in the middle of the Pacific Ocean.
B. Marinnation will be built in a deep place in the Pacific Ocean.
C. Marinnation could be served as home for international organisations.
D. Marinnation would have about one million inhabitants.
5. The word ‘monumental’ in Paragraph 2 means_________.
A. important and difficult B. like a large monument
C. serving as a monument D. important and historic
6. What is going to be built first?
A. a city B. a dam C. a monument D. a sea bed
7. The phrase ‘suck out’ in paragraph 2 is closest in meaning to which of the following?
A. draw out B. pull out C. dry up D. takeout
8. The most difficult task is to protect the dam against_________and earthquake.
A. Fire B. Water C. Sunlight D. weather
9. According to the text, people could come to live in the future city in the year_________.
A. 2020 B.2010 C. 2012 D. 2002
10. What does the author imply in the last sentence of the passage?
A. The construction of Marinnation could never be completed in time.
B. People will ask an open question about living in Marinnation.
C. People will ask how they can live in such an isolated and artificial city.
D. People might not want to live in an isolated and artificial community.
c. Fill in each numbered space with one appropriate

92
WILL TECHNOLOGY REPLACE SCHOOLS
Some people believe that soon schools will no longer be necessary. These people say that because of the
Internet and other new technology, there is no longer any need for school buildings, formal classes, or (1)
_________. Perhaps this will be true one day, but it is hard for me to imagine a world without (2) _______. In
fact, we need to look at how we can use new technology to make schools (3) _________not to eliminate them.
We should invent a new kind of school (4) _________is linked to libraries, museums, science centers,
laboratories, and even corporations. Experts could give (5) _________on video or over the Internet. TV
networks and local stations could develop programming about things students are actually (6) _________in
schools.
Is this just a dream? No. Already there are several towns where this is beginning to (7) _________.
Blacksburg, Virginia, is one of them. Here the entire city is linked to the Internet, and learning can take (8)
_________at home, at school, and in the office. Businesses provide programs for the schools and the
community. The schools provide computer labs for people without then own (9) _________home. Because
everyone can use Internet, older people participate as (10) _________as younger ones, and everyone can visit
distant libraries and museums as easily as nearby ones.
IV. WRITING
Rewrite the following sentences using the cue given in such a way that it means the same as the one given.
Do not alter this word in any way.
1. Tim insisted on being told the complete story.
→ Nothing but______________________________________________________
2. Philip’s inability to make decisions dates from his accident.
→ Ever____________________________________________________________
3. Mrs. Obama says she’s sorry she didn’t attend the meeting yesterday morning.
→ Mrs. Obama sends_________________________________________________
4. Without skillful surgery, he would not have survived the operation.
→ Thanks__________________________________________________________
5. We were just going to bed when the Earthquake happened.(point)
→ ________________________________________________________________
6. Tony bought Sally a necklace as a compensation for not having a holiday. (make)
→ ________________________________________________________________
7. My father is going to be really angry when he finds out that I have lost the car key. (wall)
→ ________________________________________________________________
8. If things go wrong, John, whatever you do, do not panic. (head)
→ ________________________________________________________________
9. At the moment I only have time to think about my examination (preoccupied)
→ ________________________________________________________________
10. The introduction of the new currency has greatly affected the economy, (impact)
→ ________________________________________________________________

---THE END---

93
SỞ GIÁO DỤC VÀ ĐÀO TẠO HÀ NỘI
KỲ THI TUYỀN SINH VÀO LỚP 10 CHUYÊN THPT
Năm học: 2011 - 2012
ĐỀ CHÍNH THỨC
Điểm (Bằng số/chữ) Chữ ký GK1 Chữ ký GK2 Môn thi: Tiếng Anh (Điều kiệu)
Ngày thi: 23 - 6 - 2011
Thời gian làm bài: 120 phút
Phách:

(Đề thi gồm 04 trang. Thí sinh làm bài vào tờ giấy thi này và không được sử dụng bất kỳ loại tài liệu nào kể cả
từ điển. Giám thị không giải thích gì thêm.)
I. VOCABULARY AND GRAMMAR (3,75 pts)
a. Circle the best option A, B, C or D to complete each of the following sentences.
1. He is my brother__________name is Bill.
A. He B. His C. Her D. She
2. It’s raining heavily, come__________my umbrella.
A. with B. under C. for D. to
3. Don’t worry, your luggage will be looked__________.
A. after B. over C. at D. upon
4. Do you know what__________? - I can’t remember.
A. is his name B. his name is C. was his name D. his name was
5. Your father’s sister is your__________.
A. niece B. cousin C. aunt D. sister
6. He is not__________in politics.
A. interest B. interests C. interesting Đ. interested
7. Let’s go somewhere else. There’s__________noise in this room.
A. too many B. too much C. enough D. too
8. That is the man__________I think hit me yesterday.
A. who B. whose C. which D. whom
9. Could you tell me who is responsible__________ passports?
A. to check B. checking C. about checking D. for checking
10. The person who__________in charge of a car is the conductor.
A. is B. are C. was D. were
11. The librarian asked us__________so much noise.
A. not to make B. no make C. not making D. don’t make
12. We are always proud__________our country.
A. about B. on C. of D. to
13. She put__________ speaking to him as long as possible.
A. off B. over C. away D. by
14. Would you like__________and__________me next summer?
A. come / visit visiting B. coming / visit C. to come / visit D. coming / visiting
15. We can’t go along here because__________.
A. the rood is been repaired B. the road is being repaired
C. the road is repairing D. the road is repaired

94
b. Complete the following sentences with the correct forms of the words in the brackets.
16. Do you know the man__________near the door? (stand)
17-18. My friend’s hobby is__________ (collect) stamps. He is a stamp__________. (collect)
19. Hard work always brings__________. (succeed)
20. The advantage of living in the countryside is that the air is__________. (pollute)
C. Give the correct tense form of the verbs in the brackets to complete the sentences.
21. Would you like __________ (go) for a walk with me?
22. When you__________ (visit) him, give him my best wishes.
23-24. Nam__________ (listen) to the music when the phone__________ (ring).
25. Look! The school boys__________ (play) chess in the classroom.
26. I am going to bed. I __________ (work)four hours and I am very tired.
27-28. The children were frightened because the lights__________ (go) out and they__________ (sit) in the
dark.
29-30. We are going__________ (have) our house __________ (repair) next month.
II. READING (3,125 pts)
a. Fill in each blank with a suitable word to complete the passage
...Tet or the Lunar New Year holiday is (31) __________most important celebration for Vietnamese people.
Tet is a festival (32)__________occurs in late January or early February. It is a time (33) ________ families to
clean and decorate then homes, and enjoy special food such as sticky rice cakes. Family (34)___ who live apart
try to be together at Tet.
...Easter is a joyful festival which is celebrated (35) __________many countries. It happens at around the
same time as Passover. (36) __________Easter Day, young children receive chocolate or sugar eggs - as long
as they are good. In many countries, (37) __________crowd the streets to watch colorful parades.
b. Read the passage and do the following task
HALLOWEEN IN AMERICA
Halloween was introduced to America by Irish immigrants around the 1840s. The holiday was also known
by the name Devil’s Night or Hell Night. People played mischievous tricks on their neighbors. For instance,
they might steal some of their eggs from the hen house.
In today’s times it has become more family-oriented with Halloween custom, games like “bobbing for
apples” or a contest for the most original costume.
To bob for apples: place apples in a large tub tilled with water. With their hands tied . behind the backs,
people try to catch one of these apples using their teeth. The person catching the most apples wins the game.
Decide which ones are True (T) or False (F).
__________38. Irish people introduced Halloween to America.
__________39. People knew Halloween by the name Hell Night.
__________40. People stole eggs from the hen houses of their neighbors.
__________41. Halloween is not considered to be more family-oriented nowadays.
__________42. People don’t wear anything special for Halloween.
__________43. People have an apple to play this game.
__________44. In the game “bobbing for apples”, apples are placed in water.
__________45. People try to catch apples with their hands.
c. Read this letter from someone who is doing a language course in France, Circle the best option A, B, C or
D.
Dear Mom and Dad,

95
This is just a quick letter to let you know that I’m fine. We’re all working quite hard because the exams are
only a few weeks away, but our teacher says he thinks we'll do well. (My landlady’s not so sure - she keeps
telling me I should stop watching so much TV and do a bit more reading.)
Anyway, we had a wonderful time at half term. I went off to Paris for a few days (46) __________some of
the others from the school, and we went on a guided (47) __________of the city. We saw most of the famous
(48) __________like the Louvre, where they have an amazing (49)of paintings, and the Eiffel Tower, Then we
went down to the south of France (50) __________train. It was so fast - the whole (51) __ was only about four
hours. We spent (52) __________time in Nice and in Cannes as well, and we all really enjoyed it. On our last
day we went to a small island called the Ile des Pelerins, and we saw the castle (53) __________the ‘Man in
the Iron Mask’. All in all, we had a great time and the weather was really good as well. The only thing I felt
(54) __________about was that we didn't have enough time to spare to go swimming - maybe next time.
Anyway, I must go now as I’ve got to do a lot of (55) __________for tomorrow. Will you write soon?
Love,
46. A. by B. to C. with D. together
47. A. trip B. visit C. tour D. excursion
48. A. shows B. sights C. views D. looks
49. A. set B. group C. gallery D. collection
50. A. in B. by C. with D. on
51. A. voyage B. travel C. drive D. journey
52. A. little B. a little C. few D. a few
53. A. are used to keeping B. are used to keep C. used to keep D. used to keeping
54. A. sorry B. apologetic C. displeased D. anxious
55. A. homework B. home work C. homeworks D. home works
II. WRITING (3,125 pts)
a. Put these words in the correct order to make meaningful sentences.
56. Hyde Park / a / place / for / good / in / is / fine / weather / camping /. /
___________________________________________________________________________
57. Jane / with / promised / keep / in / has / touch / is / us / to / while / in / she / Australia.
___________________________________________________________________________
58. What / do / of / English / you / learning / find / most / aspect / difficult / ? /
___________________________________________________________________________
59. If / down / in / there / will / you / forests / be / floods / cut / trees /,/ big / the / every year / the / . /
___________________________________________________________________________
60. The / convenient / Internet / a / fast / and / way / to get / update / information / is / which / is /, /
___________________________________________________________________________
61. Thanks / television / to /, / information / get / and / programs / can / latest / enjoy / the / inexpensive / way /
and / convenient / interesting / people / in / an /. /
___________________________________________________________________________
62. Keep / passport / asked / in case / you / your / are / to / it / at / handy / customs / show /. /
___________________________________________________________________________
63. In / going / the /, / we / boating / afternoon / in / river / and / the / had / picnic / went / on / the / a river /
before/ home / late / in / evening / the / bank /. /
___________________________________________________________________________
b. There is ONE mistake in each sentence below. Underline and correct it

96
Your correction
64. Stay on bed until you feel better to go to work. 64.__________
65. Would you mind to go to the local bookstore to get some more new books? 65. __________
66. My brother and me are members of this club. 66. __________
67. The movie was so bored that we fell asleep after an hour. 67. __________
68. Water-colors dry more faster than other paints. 68. __________
69. It usually makes us 20 minutes to go to school on foot every day. 69. __________
70. It was only when he got on the bus that he remembered not to lock the front door. 70. __________
71. Preserving natural resources mean reserving them for our culture. 71. __________
72.1 have never hear a more amusing story than this one. 72. __________
73. They suggested ban the sale of alcohol at football matches. 73. __________

c. Rewrite these sentences using the suggested words in the brackets. Do not change the words given in any
way.
74. My mother is going to give me a new bike. (A new bike)
_____________________________________________________________________________
75. “Remember to book a table for tomorrow afternoon, Jane” he said. (He reminded)
_____________________________________________________________________________
76. “Shall we go to the nightclub?” Mike said. (Mike suggested)
_____________________________________________________________________________
77. John is one of my closest friends. I have known him for three years. (whom)
_____________________________________________________________________________
78. Working so much will make you tired. (If)
_____________________________________________________________________________
79. How long have you learnt Japanese? (How long ago)
_____________________________________________________________________________
80. It has been reported that the chancellor is very satisfied with his visit to Japan. (been)
_____________________________________________________________________________
---THE END---

97
THPT CHUYÊN HÀ NỘI-AMSTERDAM
ĐỀ THI THỬ VÀO LỚP 10 THPT
Năm học 2010-2011
Môn thi: Tiếng Anh (Không Chuyên)
Thời gian làm bài: 120 phút
I. PHONETICS
a. Choose the word whose main stressed syllable is different from the rest
1. A. century B. frontier C. community D. ocean
2. A. identity B. result C. heritage D. unique
3. A.ethnic B. tradition C. belief D. expect
4. A. dynamite B. earthquake C. environment D. festival
5. A. computer B. memory C. serious D. special
b. Choose the word whose underlined part is pronounced differently from the rest
1. A. amount B. wounded C. bound D. round
2. A. known B. town C. own D. post
3. A. friendly B. simply C. imply D. specialty
4. A. champagne B. Christain C. children D. chicken
5. A. hour B. honest C. heart D. honor
II. VOCABULARY AND GRAMMAR
A. Circle the best one A, B, C, or D to complete the sentence.
1. I must congratulate you_________your excellent exam results.
A. for B. in C. on D. with
2. He has now been formally_________as a presidential candidate.
A. pointed B. considered C. received D. nominated
3. I wish you_________making that noise. It is bothering me.
A. stop B. would stop C. can stop D. will stop
4. What_________of learning English do you find most difficult?
A. aspect B. side C. detail D. position
5. Do you remember Mrs Lake, _________taught us English?
A. who B. whom C. that D. which
6. A tornado touched down in Kansas today, _________destruction to the telephone lines.
A. caused B. causing C. that caused D. it caused
7. He has never been to the United States, _________?
A. isn’t he B. doesn’t he C. hasn’t he D. has he
8. What makes you_________in the existence of UFOs?
A. believe B. to believe C. believing D. believed
9. We couldn’t go on a picnic as planned_________it was raining hard.
A. although B. but C. because D. so
10. You can look_________a word in a dictionary if you don’t know what it means.
A. for B. Up C. over D. after
11. She was responsible_________looking_________her younger brother.
A. for/after B. after/for C. of/up D. for/about
12. If you go on ahead, I’ll_________you later.
A. catch up with B. fit in with C. walk out on D. catch up with

98
13. Passengers are_________not to leave cases here.
A. made B. reported C. advised D. informed
14. Your behavior makes me_________
A. anger B. angrily C. angry D. in anger
15. If it hadn’t been for your laziness, you_________the exams.
A. could have passed B. could pass C. had passed D. must have passed
16. Have I_________to you how to use this machine?
A. answered B. explained C. told D. showed
17. We often watch a film on TV_________going to bed.
A. then B. during C. upon D. before
18. It’s time_________care of yourself.
A. you took B. you take C. to taking D. you taking
19. He found_________to answer all the questions within the time given
A. that impossible B. it impossible C. it impossibly D. that impossibly
20. Are there enough chairs for_________?
A. we all B. all we C. us all D. ourselves.
b. Complete each sentence with the correct tense and form of the verb in bracket.
1. You (know) _________the girl who just (leave) _________the shop?
2. I (buy) _________this hat in Tran Phu street.
- How much you (pay) _________for it?
3. What’s the matter? Why you (look) _________at me like that?
4. May I use your phone (call) _________a taxi ?
- There is no need (do) _________that. I (drive) _________you home.
5. Ann (want) _________to see you. You (not be) _________for ages.
c. Choose the underlined word or phrase that needs correcting.
1. I do not know where could he have gone so early in the morning.
A B C D
2. The trees were covered of snow when I took this picture last year.
A B C D
3. Would you mind to help me with my homework when you have time?
A B C D
4. I thought it was a very depressed movie.
A B C D
5. He stopped to write his letter because he had to leave for the hospital.
A B C D
6. Unless we work harder, we will finish on time.
A B C D
7. Children sometimes wish they can do things their older brothers and sisters do.
A B C D
8. Hurricane Andrew, that swept through southern Florida in 1992, killed 41 people and made
A B C
thousands homeless.
D
9. What happened in the house that day were a secret because nobody was allowed to enter.

99
A B C D
10. We reached the nearest village after walk for 5 hours.
A B C D
d. Choose the correct sentence with the same meaning as the original ones.
1. His story was so funny that it made us all laugh.
A. His story was too funny to laugh
B. His story couldn’t make us laugh
C. His story was very funny and we couldn’t help laughing.
D. We all laugh at him for his story.
2. I will remember to water the plants.
A. I won’t forget to water the plants.
B. I won’t forget watering the plants.
C. I remember watering the plants.
D. I promise watering the plants.
3. It rained heavily, so the football match was cancelled.
A. The match was cancelled because of the heavy rain
B. If it didn’t rain, the match wouldn’t be cancelled.
C. Despite the heavy rain, the match was cancelled.
D. If it hadn’t been for the heavy rain, the match wouldn’t be cancelled.
4. “No, I didn’t tell Jim our plan,”, said Tom.
A. Tom refused to tell Jim then plan.
B. Tom denied having told Jim then plan.
C. Tom denied to tell Jim their plan.
D. Tom didn’t agree to tell Jim their plan.
5. Jane had better change her job.
A. Jane had a better job after changing.
B. Jane had changed her job for good.
C. Jane should change her job.
D. Jane’s job had changed for the better.
III. Reading.
а. Choose the best answer to complete the following passage.
Choosing clothes can be difficult. Some people want to be (1)__________, but they don’t want to look
exactly (2)__________everybody else. Not all clothes are (3)__________for work or school, perhaps they’re
not (4)__________enough or simply not comfortable . It is easy to buy the (5)size, and find that your trousers
are too tight, especially if you’re a little bit (6)__________. Very (7)__________clothes make you feel slim. If
you buy light cotton clothes then they may not be (8)__________enough for winter. If your shoes are not
(9)__________, and if you aren’t (10)__________for the cold , you might look good , but feel terrible !
1. A. of fashion B. fashioned C. fashionable D. fashion
2. A. alike B. like C. similar D. same
3. A. fitted B. suitable C. comfort D. equal
4. A. formal B. strict C. uniform D. suited
5. A. false B. mistake C. wrong D. error
6. A. slim B. overweight C. thin D. enormous
7. A. loose B. lose C. loosened D. lost

100
8. A. warm B. cold C. hot D. cool
9. A. tight B. enclosed C. firm D. waterproof
10. A. worn B. clothed C. dressed D. fitted
b. Read the passage carefully and decide if it is T or F
Many people still believe that natural resources will never be used up. Actually, the world's energy resources
are limited. Nobody knows exactly how much fuel is left. However, we also should use them economically and
try to find out alternative sources of power. According to Professor Marvin Burnharrm of the New England
Institude of Technology, we have to start conserving coal, oil and gas before it is too late; and nuclear power is
the only alternative.
However, many people do not approve of using nuclear power because it's very dangerous. What would
happen if there was a serious nuclear accident? Radioactivity causes cancer and may badly affect the future
generations.
The most effective thing ia that we should use natural resources as economical as possible.
1. Natural resources will never run out
2. We know exactly how muck fuel is left
3. We shouldn’t waste fuel
4. According to Professor Marvin Burnham, there is no substitute for coal, gas and oil
5. Many people disagree to use nuclear as an alternative energy
IV. WRITING
Rewrite the following sentences using the words given so that it means the same as previous one.
1. I’m fond of music, but I can’t play any musical instruments.
→ Although_______________________________________________________________
2. “Are you going to visit your aunt tomorrow?”, Tom asked Mary.
→ Tom asked______________________________________________________________
3. The new stadium win be opened next month. It can hold 90.000 people (which)
→ _______________________________________________________________________
4. Mr. Richard is very interested in our plan. I spoke to him yesterday.
→ Mr. Richard_____________________________________________________________
5. I am so busy, so I don’t write to my friend regularly.
→ If______________________________________________________________________
6. She only feels good in warm weather. As it’s getting warmer, she feels better.
→ The warmer______________________________________________________________
7. If I were Peter, I’d look for another job.
→ I suggest_________________________________________________________________
8. He started the job 5 years ago.
→ He has___________________________________________________________________
9. We like to lie on the beach all day.
→ We enjoy_______________________________________________________________
10. Work hard or you will fail the exam.
→ If_____________________________________________________________________

---THE END---

101
SỞ GIÁO DỤC VÀ ĐÀO TẠO HÀ NỘI
KỲ THI TUYỂN SINH VÀO LỚP 10 THPT CHUYÊN
Năm học 2010-2011
Môn thi: Tiếng Anh (Chuyên)
Ngày thi: 24 tháng 6 năm 2010
Thời gian làm bài: 120 phút
I. PHONETICS (1 pt)
a. Choose the word whose underlined part is pronounced differently from the rest
1. A. Zoo B. Stood C. Soon D. Moon
2. A. double B. Enough C. Resource D. Rough
3. A. Marched B. Released C. Managed D. Increased
4. A. Planets B. States C. Astronauts D. Systems
5. A. Aware B. Wholesales C. Worldwide D. Wildlife
b. Choose the word whose main stressed syllable is different from the rest
6. A. Particular B. Preparation C. Participant D. Inventory
7. A. Amazing B. Talented C. Italy D. Discover
8. A. Remove B. Repeat C. Effort D. Prefer
9. A. Destruction B. Volunteer C. Economical D. Recommend
10. A. Independent B. Correspondent C. Electrical D. International
II. VOCABULARY AND GRAMMAR. (4.0 pts)
a. Circle the best option to complete the sentence.
11. “Can I help you?”-“______________”
A. I’m sorry you can’t B. Not for me. Thank you
C. No, it’s alright. I can manage D. I’d rather you couldn’t
12. “I’ve never seen that movie.” - “______________have I”
A. So B. Either C. Neither D. Too
13. Helen asked me__________the film called “Star Wars”.
A. have I seen B. Have you seen C. If I had seen D. if had I seen
14. The women said “This carpet was made__________, so it is expensive”.
A. by hand B. By the hand C. By hands D. By our hands
15. __________out of the shop, my friend Roger came in.
A. Coming B. when coming C. On coming D. As I came
16. Don’t do that work now because it takes__________-a lot of time.
A. up B. After C. On D. Over
17. You will spend at least one year working abroad__________you can find out how things ate overseas.
A. because B. So as to C. So that D. As long as
18. Her name was on the__________of my tongue, but I just couldn’t remember it.
A. edge B. Top C. Point D. Tip
19. Emotional speeches often__________strong feeling.
A. rise B. Raise C. Arouse D. Provide
20. A kangaroo carries its young in its__________.
A. pocket B. Pram C. Bag D. Pouch
21. After I__________- finished working, I switched off the machine.
A. have B. Had C. Having D. To have

102
22. Either Michael or I__________the keys to the car we have sold.
A. has taken B. Have taken C. Has been taken D. Have been taken
23. If you are entering a private property uninvited, you are__________.
A. invading B. Trespassing C. Interrupting D. Inciting
24. __________-built as a private home for Elvis Presley, Graceland is now open to the public.
A. When B. Although C. Being D. Despite
25. __________good teacher must have additional activities prepared for weak students.
A. The/X B. The/a C. A/x D. The/an
26. The sky was grey and cloudy. __________, we went to the beach.
A. Consequently B. Nevertheless C. Eventhough D. In spite of
27. If a boat is travelling downstream, it is going__________the current.
A. towards B. Against C. With D. Opposite
b. Choose the word that is closet meaning to the underlined word or phrase in each sentence.
28. The expensive skirt has become smaller after the first wash.
A. reduced B. Contracted C. Shrunk D. Decreased
29. Mr Tan spoke well of Ellen in front of the class for being brave.
A. dismissed B. Praised C. Honored D. Exalted
30. Nicholas brags about his father’s wealth all the time
A. lies B. Boasts C. Announces D. Hides
31. The soldiers had to fight the war to protect their country.
A. defect B. Defend C. Guard D. Fight
32. Please look closely at the specimen because the flaw can be very small and hard to detect.
A. scrutinize B. Glare C. Stare D. Investigate
c. Complete the following sentence with the correct form of the word in the right column.
33. The building is old and in need of_________. 33. renovate
34. He works as an_________for an insurance company based in London. 34. account
35. The magazine has an annual_________fee of $90. 35. subscribe
36. All staffs must wear_________clothing when operating this machine. 36. protect
37. The book can be_________for free from the Web. 37. download
38. The interviewers asked each_________the same basic questions. 38. interview
39. As a result of her expensive holiday, she now has a large________with the bank. 39. overdraw
40. He worked as a venture_________in New York. 40. capital

d. Complete each sentence using the suitable verb form in column A with a preposition in
column B to make expressions of phrasal verbs. (YOU can use more than once).
A B
Keep do squeeze head On off out for of
Plan take break make step In over up down away

41. Why did John_________so quickly when I arrived?


42. Although he plans to leave, we hope to_________him_________for a month or two while we find a
replacement.
43. He is_________from Ms Jones as head of the department next April.
44. Each part of the project has been_________in great detail.

103
45. The murder_________under questioning yesterday afternoon.
46. Tom sent me a note yesterday but I can’t_________what he had written.
47. Many small shops have been_________of the market by the major supermarkets.
48. We didn’t have a map, so we_________the hills hoping to find somewhere to stay.
49. Bill Morgan will_________the business as a chief executive.
50. We are_________with out old cooker and buying a new one.
III. Reading (2.4 pts)
a. Fill in the blank with a suitable word to complete the passage
In some ways, the United States has (51) _________spectacular progress. Fires no longer destroy18,000
buildings as they did in the Great Chicago Fire of 1871, or kill half of a town (52) _________2,400 people as
they did the (53) _________night in Peshtigo, Wis. But even with such successes, the United States still (54)
_________one of the worst fire death rates in the world. Safety experts say the problem is neither money nor
technology, (55) _________the indifference of a country that just will not take fires (56) ____ enough.
American fire departments spend far less on (57) _________fires than on fighting them. And American fire-
safety (58) _________are aimed almost entirely at children. Experts say the fatal error is an attitude that fires
are not really anyone’s fault.
B. Read the following magazine article in which six students (A, B, C, D, E, and F) say how they cope with
exams. Answer the question that follow by writing the letter A, B, C, D, E or F next to the questions
numbered (59-68). The student may be chosen more than once to answer the questions.
Six students explain how they cope with the stressful time before important exams.
Student A
Revising for exams can be really tiring, so I try to stay alert by drinking cups of coffee while I’m studying.
But the most important thing for me is to make sure I sleep well the night before an exam. Before I go to sleep,
I put on a CD of my favourite singer, or read a book or magazine-nothing to do with the exam.
Student B
For me the best way to stay on top of things during exams is not to worry too much and to take time off from
revision. I still meet up with my friends while exams are on. As well as that, I do a lot of exercise and I make
sure I have a good work-out the night before an exam. I just know that my brain will function better if I feel in
good shape.
Student C
I tend to panic in exams, so I have learnt that the best way to cope is to watch the time carefully through the
exam. I plan how much time I will need for each question, and keep an eye on the clock to see how I am doing.
I never tackle a question until I have thought about it, and made some rough notes on the answer.
Student D
In the past, I have found that I got lower marks than I could have got because I spent time worrying, not
studying. Before exams, I often talk things over with my parents and teachers. They often give very good
advice, as they have been through it all before, and it helps to get all my anxieties into the open. I think talking
to them is better for me than wasting a lot of valuable revision time worrying about everything.
Student E
During exams, I avoid all those negative people who are completely stressed out and wish they had done
more work. The night before an exam, I go out with my friends. We usually go for a walk, or go round to
someone’s house, order a pizza and watch a good video, preferably a comedy. It’s important to laugh a lot and
see the funny side of things when there is so much hard work to do the next day.
Student F

104
I know that I can only concentrate for a limited time, so I stop every hour or so for 10 minutes during my
revision sessions. I may go for a walk around the block to clear my head, or just sit and think about something
else to take my mind off the exams. I think it’s also important to avoid drinking lots of strong coffee, because
it tends to increase my anxiety, so I drink lots of fruit juice or mineral water.
Which of the students
59. _________- discusses his/her worries with more experienced people.
60. _________stops and thinks before answering a question in an exam.
61. _________likes to be with people who have a positive attitude.
62. _________ takes short break while he/she is revising?
63. _________thinks it is important to continue with a social life? .
64. _________relaxes with music.
65. _________had not always done as well as he/she could in exams?
66. _________finds that coffee makes him/her feel more anxious?
67. _________believes that lots of physical activity is good during exams?
68. _________thinks you should keep a sense of humour.
c. Read the article and choose the paragraphs from (A-H) that fit each gap (69-74). There is one extra
paragraph which does not fit in any of the gaps.
Night School
What would you do if you couldn’t attend school during the day?. If you are an adult who didn’t finish
secondary school, what possibilities are there for you? A growing number of people around the world are
looking for an option that really meets then needs.
0.___ E___
Night school is an educational option that exists in many countries of the world. Since formal schooling is
compulsory until the age of 16 or 18 in many places, young people who find themselves in need of work or
have other important daytime obligations can finish then studies. For many people, it’s the right choice.
69_________
Young people are not the only ones who face this choice, however. Night school also attracts adults who left
school early. Since they are already working adults, night school can give them the opportunity to go back to
school without leaving their jobs.
70. _________
Courses in night school are usually the same as those taught during the day. They are generally held in the
same building, too. Students have the same facilities and the same requirements as their fellow students during
the day. All in all, the experiences are the same. The only difference is that night school students go to school
later!
71. _________
Michael is a 17-year-old boy who works during the day on his family’s organic farm. It’s a new business
which requires a lot of careful attention to get started. Micheal’s father runs the farm now, but one day it will
be Micheal’s responsibility, which is why he needs to learn as much as he can about the business. Since he
didn't want to stop his schooling in order to work, he enrolled in night school. “It was a bit strange at first”, he
says “working all day then going to school in the evening, but I soon got used to it. In my classes there are a lot
of students who work during the day like I do, so we have many things in common. I’ve made really good
friends.
72. _________

105
Then we talked to Tonya, who is a teacher at night school. She used to teach during the day, but prefers
evening classes now. “The night students are very dedicated,” she told us. “It’s also really nice to have a mix
of adults and teens. They work hard, but they have a lot of fun, too.”
73. _________
Take David, for example. He’s a teenager who tried going to night school for a while so he could train as a
carpenter with his uncle during the day. “I was so tired after working all day that I didn’t have the energy to go
to school! And when could I do my homework? In my sleep? No way. I’d rather do one thing at a time. David
decided to work part-time on weekends and finish school during the day.
74. _________
For most of the students who do enroll, however, night school is the answer they’ve been looking for. It is a
challenge to balance work and school, and it’s definitely not for everyone. But those who go to night school
are really making the most of their time - day and night!
A. Many students also take summer classes, usually to make up for bad grades during the year. They have to
pass the summer course in order to move to the next level in the autumn when the new term begins.
B. More and more young people are taking on the responsibility of a job during the day, for different
reasons. Perhaps they have to help out their families, or maybe they want to learn a particular skill or trade.
But if they work during the day, how can they complete then studies? Is their only choice to leave school at
an early age?
C. Different people react in different ways to the experience, not all of then enjoy it. We visited one school
that runs classes in the evening and talked to students and teachers to find out more. Let’s meet them and see
what they have to say.
D. Not everyone, of course, thinks that night school is the answer. Some people don’t think there’s enough
time in one day for both school and work.
E. Fortunately, there is an alternative to going to school during the day - night school. Night schools offer
the same classes as regular secondary schools except during evening hours. This evening programme allows
students to continue working during the day and study at night.
F. Parents might also object to night school. Meg is a teenager who wanted to leave school to take up a
career in modeling. When Meg looked into going to night school, her parents warned her against it. “They
thought I would never finish school that way. My parents think working and going to night school is too
much for someone my age. I disagree, but I’m only 17, so I have to do what they say.
G. Next we spoke to Cheryl, a young adult in her 20 s who dropped out of school as a teenager. Later,
however, she realized that she didn’t have the skills and qualifications she needed for her career, so decided
to return to school. Since she works during the day, night school was the perfect option. “Going to night
school has been a great opportunity for a second start. I can keep my job and complete my studies,” Cheryl
explained.
H. What’s night school like? How is it the same - or different - from school during the day? Here we present
an up-close and personal look at night school.
IV. Writing (2,6 pts)
a. Finish each of the sentences in such a way its meaning the same as first ones
75. “Don’t leave the house until I get back, William”, his mother said.
→ William’s mother told______________________________________________
76. Mai thought the man in front of her was her former teacher.
→ Mai mistook______________________________________________________
77. His memory gradually failed as he grew old.

106
→ The_____________________________________________________________
78. Why did you throw away those newspaper?. I hadn’t finished with them.
→ If I______________________________________________________________
79. Then teacher is making them study hard in order to pass the exams.
→ They are_________________________________________________________
80. The best solution was thought of by Peter.
→ Peter came_______________________________________________________.
81. He does not appreciate his wife. (granted)
__________________________________________________________________
82. Contact the receptionist if you have any further problems. (get)
__________________________________________________________________
83. I found this lecture quite disappointing .(let)
__________________________________________________________________
84. It was hard not to start laughing when she started to sing (face)
__________________________________________________________________
85.My sister was offended when she was left out of the English club. (exception)
__________________________________________________________________
86-100. Write no more than 150 words inside this space about secondary school students’clothes.

107
SỞ GIÁO DỤC VÀ ĐÀO TẠO HÀ NỘI
KỲ THI TUYỂN SINH VÀO LỚP 10 THPT CHUYÊN
Năm học 2010-2011
Môn thi: Tiếng Anh
Ngày thi: 23 tháng 6 năm 2010
Thời gian làm bài: 120 phút
I. PHONETICS (1,25 pt)
a. Choose the word whose underlined part is pronounced differently from the rest
1. A. looked B. moved C. gained D. prepared
2. A. heal B. hockey C. exhaust D. hole
3. A. character B. church C. armchair D. kitchen
4. A. pleasant B. appeal C. reason D. teapot
5. A. typical B. family C. Bicycle D. cycling
b. Choose the word whose main stressed syllable is different from the rest.
6. A. celebration B. entertainment C. traditional D. preparation
7. A. Electric B. develop C. Energy D. another
8. A. available B. individual C. engineering D. transportation
9. A. important B. recreation C. improvement D. decision
10. A. convenient B. Previous C. Management D. purchase
II. VOCABULARY AND GRAMMAR. (4.125 pts)
a. Circle the best option to complete the sentence.
11. Tom can’t sign that document because he is_________age.
A. out of B. across C. under D. through
12. He said he_________in Moscow all his life.
A. lives B. was living C. has lived D. had lived
13. Hurry up, the train_________in a minute.
A. is leaving B. was leaving C. has left D. will be left
14. Their house is_________to the school.
A. near B. nearly C. close D. closely
15. The news_________so pleasant that he couldn’t help smiling.
A. is B. was C. are D. were
16. Fred is capable_________doing better work than he’s doing at the present.
A. to B. with C. of D. for
17. The typist is away so these documents can’t_________now.
A. type B. to type C. had typed D. is typing
18. My sister enjoys_________so much.
A. dance B. to dance C. dancing D. danced
19. My father is 3 years_________than my mother.
A. old B. older C. oldest D. elder
20. _________does she look like?. -She is young and pretty.
A. What B. Who C. How D. How old
21. I’ll call the taxi as soon as the things_________.
A. were packed B. have been packed C. will be packed D. packed
22. Do be carefull not to_________your coffee on this white rug, Bill.

108
A. drip B. spill C. filter D. leak
23. Henry’s work is well_________the average.
A. on B. above C. over D. under
24. “Go on, finish the desert. It needs_________up because it won’t stay fresh until tomorrow.”
A. eaten B. eat C. to eat D. eating
25. Great changes_________place in the city and a lot of factories_________
A. have taken/have been set up B. have been taken/have been set up
C. have taken/ have set up D. were taken/ were set up.
b. Complete the following passage with the correct form of the words
Exercise is one of the best ways of keeping depressed away. It improves your body, your mind and enables
you to perform better in the work place and at home.
Proper (26.breath) _________is essential if you want to get the most from exercise and you should also take
into (27.consider) _________your heart rate. It can be (28.harm) _________to do too much, which is why all
good fitness instructors emphasize the (29.Important) _________of “listening to your body”. When you first
start you should use good (30.judge) _________, because it’s easy to make the mistake of using the equipment
incorrectly on doing too much at one time. Start slowly and build up gradually. Exercise should not be seen as
a (31 .demand) _________task; it can be as easy as a quick walk . To increase your fitness (32.steady)
_________, exercise for 20 minutes a day, 4 to 6 times a week and you will notice a (33. Different)
_________in your body and mind in a few weeks.
c. Give the correct form of the verbs in brackets.
34. I never (see) _________any of Picasso’s painting before I visited the art museum.
35. Sorry, I’m late. You (wait) _________-long?
36. The government is worried because the number of people without jobs (increase) _________at present.
37. What Jane (do) _________at 5 o’clock yesterday.?
38. Linh has been in London for 2 years. Now he is used to (drive) _________on the left.
39. A lot of people (be) _________out of work if the factory closed down,
40. The paintings (exhibit) _________next month.
41. Peter (have)his driving license (take) _________away by the police yesterday.
42. She asked me where I (go) _________for my last summer holiday.
43. I’ve got a terrible headache.
“Have you?. Wait and I (get) _________an aspirin for you.
III. READING. (2,5 pts)
a. Fill in each blank with a suitable word to complete the passage.
There is usually one important subject missing (44) _________most school timetables. Very few students
are taught how to organize their learning and how to make the best use of their time. Let’s (45)
_________some simple examples. Do you know how to (46) _________up words in a dictionary, and do you
understand all the information the dictionary contains? Can you make notes quickly and can you understand
them?
For some reason, many schools give learners no help with these matters. Teachers ask students to memorize
pages from books, but don’t explain how to do it. (47) _________by heart can be useful but it is important to
have a genuine (48) _________of a subject. You can waste a lot of time memorizing books, (49)
_________understanding anything about the subject.
b. Read an article about the library in the past. Choose the answer (A, B, C or D) which you think fits best
according to the text

109
Going to the library in the past
When I go into our local library, I often watch children looking at the shelves filled with a variety of brightly
coloured books. They pick a book, glance through a few of the pages and then almost immediately reject it
before beginning to look at another book. I smile to myself for when I was a child in the 1910s, we were never
allowed anywhere near the books. They were kept in some remote comer of the building to which only the
librarian had access.
How, you may ask, did we manage to choose' the book which we wanted to borrow? Well, immediately to
the right of the enhance was a room which served two purposes: it was a reading room for the older members
of the community and it contained catalogues in alphabetical order of the titles and authors of the books kept in
the library. Using these, all one had to do was to write out
a list of the books required. However, since most of us knew very few authors by name, and even fewer book
titles, the whole process of borrowing a book was based upon guesses. There was no possibility of looking
through the first few pages to help us form an opinion, no looking at illustrations to discover if a book might
arouse our interest.
Even now I recall almost with pain some of the selections my friends and I made. We learned with dismay
that titles often gave little guidance as to what the book was about. If we could have returned the book the next
day, our irritation would have been considerably reduced, but this was not possible. The librarian did not allow
us to bring back any book until we had kept it for at least a week.
Having written out your list, you presented it to the librarian. If you thought your troubles were finished, you
were sadly mistaken. Your hands were inspected to make sure they were clean. More than once, one or other
of us was sent out of the library and told to return when we had washed our hands.
Once the librarian was satisfied that we were clean enough, she would disappear into another room and
return with the first book on our list which was available. When a book was returned, she would make a
thorough inspection of the pages to make sure that it had not been damaged in any way. At least that way we
never got blamed for something we had not done.
Although the librarian appeared very strict and frightening to a small boy, I owe her and the man who gave
the library to the town an immense debt of gratitude. They led me into the land of story, romance, and
adventure which in years to come brightened many a dull day.
50. According to the first paragraph, how is life different for children today from life for children in
the 1910s?
A. They always get what they want.
B. They have a wider choice of books.
C. They are allowed to behave badly.
D. They spend more time reading books.
51. It was difficult for children to know which books to choose because_________
A. there was no list of authors.
B. the illustrations did not interest them.
C. there were only a few book titles.
D. they could not see the books in advance.
52. How did children find out which books were available in the library?
A. They had to write out a list.
B. They had to ask the librarian.
C. They had to look through the catalogues.
D. They had to order them.

110
53. What does the writer say about book titles?
A. Nobody knew any of them.
B. They told us what books were about.
C. There were not very many of them.
D. They were not very helpful.
54. What rule was there about returning a book to the library?
A. You had to keep it until the following day.
B. You had to return it within a week.
C. You had to keep it for a week or more.
D. You had to return it by the following day.
55. What did the librarian do as soon as she had received a child’s list?
A. She looked at the children’s hands.
B. She sent all the children away to wash their hands.
C. She washed the children’s hands.
D. She asked the children if their hands were dirty.
56. Why does the author think that it was a good idea for the librarian to check the books carefully when they
were returned?
A. She would be able to find out why you had damaged the book.
B. You would not be accused of damaging a book that was already damaged.
C. She would be able to find out if somebody else had damaged the book.
D. You would not be blamed if you had damaged a book.
57. Why is the writer grateful to the librarian?
A. She helped open up the world of literature to him
B. She told him wonderful stories
C. She helped to cheer him up when he was older.
D. She made him feel like an adult.
IV. WRITING (2,125 PTS)
a. Circle the incorrect parts in the following sentences A, B, C, D
58. New laws should be writing to accommodate changes in our society.
A B C D
59. Some of the plants require very little care, but this one needs much more sunlight than the others ones.
A B C D
60. As a child grows on, its physical health is affected by many elements in the air, water and food.
A B C D
61. Of the two Hemingway novels I’ve read, I like “A farewell to Arms” the best, not only because of
A B C
its structure, but also because of its fascinating story.
D
62. If space tourism is to become a reality, much work would need to be carried out to estimate the
A B C
potential size of a space tourism market.
D
63. The commission’s report mentions the contributions made by both big corporations as well as
A B C

111
small businesses to the growth of the U.S economy.
D
b. Finish each sentence so that its meaning has the same with previous one.
64. Your hah needs cutting.
→ You need___________________________________________________________
65. It’s a pity. I didn’t take my teacher’s advice.
→ I wish______________________________________________________________
66. I’m getting angry. The fan is not working. The light is not also working.
→ Neither_____________________________________________________________
67. The road is very bad. I can’t drive fast.
→ The_______________________________so_______________________________
68. “Would you like to have dinner with me?” Laura said to Tom.
→ Laura invited________________________________________________________
69. Turn off all switches before leaving the workshop.
→ All the switches______________________________________________________
70. He couldn’t stop quickly enough to avoid the accident. (time)
_____________________________________________________________________
71. He spents two hours deciding which needs to buy. (mind)
_____________________________________________________________________
72. We spoke in whispers as we didn’t want to wake the baby up. (so as)
_____________________________________________________________________
73. Claire was worried about her son’s coming home late so often (made)
_____________________________________________________________________
74. I’m afraid to say that we haven’t got any sugar left. (run)
_____________________________________________________________________

---THE END---

112
THPT CHUYÊN HÀ NỘI-AMSTERDAM
ĐỀ THI THỬ VÀO LỚP 10 THPT CHUYÊN
Năm học: 2009-2010
Môn thi: Tiếng Anh
Thời gian làm bài: 120 phút
I. PHONETICS
a. Choose the word whose main stressed syllable is different from the rest.
1. A. property B. attitude C. summarize D. astronomer
2. A. remain B. become C. current D. create
3. A. broaden B. specific C. theatre D. fiction
4. A. scriptwriter B. composer C. dressmaker D. filmmaker
5. A television B. documentary C. whatever D. competition
b. Choose the word whose underlined part is pronounced differently from the rest.
1. A. area B. breathe C. increase D. meat
2. A. children B. chemical C. switch D. watch
3. A. needed B. stamped C. walked D. laughed
4. A. slash B. stack C. stamp D. swamp
5. A. surgeon B. agent C. engine D. regard
II. VOCABULARY AND GRAMMAR
a. Circle the best option.
1. During the first World War, an________5 million people lost their lives.
A. assumed B. estimated C. envisaged D. approximated
2. The aim of the Council’s latest campaign is to________the use of the new recycling facilities.
A. project B. prosecute C. promote D. provide
3. Mozart was a very________musician and had written several concertos by the age of 10.
A. gifted B. endowed C. expert D. qualified
4. John is a cousin of________.
A. him B. his C. he D. himself
5. The car had a________tire, so we had to change the wheel.
A. broken B. cracked C. bent D. flat
6. I disapprove ________people smoking in public area.
A. of B. with C. at D. on
7. The question of late payment of bills was________again at the board meeting.
A. risen B. raised C. brought D. taken
8. John, what is that noise, is there________on the sheet?
A. something B. nothing C. anything D. none thing
9. I’ve got a very high opinion________- your brother.
A. On B. to C. for D. of
10. I like him very much; he is________
A. quite an intelligent boy C. a quite intelligent boy
B. quite intelligent boy D. a boy quite intelligent
11. Their flat is decorated in a________
A. Tasteful B. sweet C. delicious D. tasty
12. Driving a car with faulty brakes is________quite a risk.

113
A. Putting B. setting C. taking D. being
13. ________the wet weather, the football match went ahead.
A. Although B. Owing to C. However D. In spite of
14. The Government hopes to________-its plan for introducing cable TV.
A. Carry out B. turn out C. carry on D. keep on
15. We regret to inform you that the materials you ordered are________
A. Out of work B. out of stock C. out of reach D. out of practice
b. Choose the words closest in meaning to the underlined words in the following passage.
My neighbor, Mr Roger, is a much (1) despised garbage collector. Some people (2) taught him by covering
their noses when he walks past them. Although he sometimes comes home dirty and smelly, we still (3) respect
him. If he were a weakling, he would have (4) thrown in the towel because of all the (5) humiliation he
receives. However, he quietly does what it takes to support his family. This really shows his strength of
character.
1. A. looked up B. looked down C. discouraged D. challenged
2. A. cheer for B. joke about C. mock at D. bully
3. A. ridicule B. recognize C. notice D. admire
4. A. fought back B. given up C. given in D. taken in
5. A. embarrassment B. humdrum C. disgrace D. shame
c. Give the correct form of the word given in bracket.
1. There are few (explore)________places left on the earth. Man has been nearly everywhere.
2. His boss told him off because he had behaved (respond) ________
3. Unless something is done about unemployment, the (look) ________for the future is not good.
4. Telling lies is (moral) ________
5. We must make (provide) ________for our old age by saving money.
d. Fill in each blank with a suitable verb to form the phrasal verb with “down”, “away” or “over”. Be sure
to use the correct form.
1. I can’t decide now. I will________it________with my staff and let you know later.
2. It is sometimes wise to wait till prices________before buying the thing you want.
3. The hall was filled up very early and the late-comers were________
4. Refusing to listen to the speaker, the crowd________him________
5. She was offered a part-time job in a fast-food restaurant, but the________it________
6. After the meal, the maid________the plates and dishes.
7. Could you be kind enough to________this letter to see if there is any mistake.?
8. You may sell the used books or________them________
9. There was a street fight and the shouting did not________till quite some time later.
10. While John was transferred, his assistant________from him immediately.
e. Underline and correct the mistakes.
1. The front-page articles in the daily newspaper has the most important news.
__________________________________________________________________________
2. A student at University should attend class regularly and hand in then assignments on time.
__________________________________________________________________________
3. I came here three and a half month ago.
__________________________________________________________________________
4. I think I have made a good progress in English

114
__________________________________________________________________________
5. It is a fact that I almost drowned makes me very careful about water safety whenever I go swimming.
__________________________________________________________________________
6. Studying psychology last year made me realize that what kind of career did I want to have.
__________________________________________________________________________
7. When I entered the room, I found my younger brother stand on the kitchen table.
__________________________________________________________________________
8. Jones that used to speak Spanish has organized a tour for senior citizens.
__________________________________________________________________________
9 .Neither of the girls have turned in the term papers to the teacher yet.
__________________________________________________________________________
10. when I was a child, I was always afraid of the beggars whom they went from house to house in my
neighborhood.
__________________________________________________________________________
III. Reading
a. Choose the best answer to complete the following passage.
I don’t want to alarm you. There is still enough sand left in the world to satisfy most holidaymakers but in
many parts of the world beaches are literally being (1)________away and have to be regularly (2) ____.
First, much of the sand for beaches (3) ________from cliffs which crumble away as they are pounded by the
waves. To (4) ________them, sea walls are often erected. With cliffs no (5) ________crumbling, the beaches
are robbed of the material which would (6) ________feed them.
Beaches are also (7) ________with sand and gravel by rivers which bring it down from the mountains and
hills. In some places rivers are being dammed and (8) ________built to retain water. They trap more of the
sediment so the rivers (9) ________less sand and gravel to the sea. This is happening in California, for
example, and in Scotland. In Egypt the (10) of the Aswan Dam has (11) ________the Nile silt, so much less
silt is being fed towards coastal (12) ________. That has meant the delta is now eroding instead of (13)
________as before. Thirdly, to improve access to the beach many holiday resorts build a promenade along the
sea front. Like some of the fortifications of cliff (14) ________, this usually has a flat vertical surface off
which the waves (15) ________. This helps wash the sand away down the beach and most of it is lost.
1. a. thrown b. rubbed c. washed d. cleaned
2. a. removed b. replaced c. rebuilt d. redrawn
3. a. collects b. forms c. falls d. comes
4. a. protect b. prepare c. surround d. cover
5. a. sooner b. longer c. further d. later
6 a. normally b. often c. sometimes d. occasionally
7. a. presented b. given c. filled d. supplied
8. a. reservoirs b. canals c. wells d. locks
9. a. fetch b. take c. pull d. push
10. a. designing b. engineering c. building d. forming
11. a. kept b. trapped c. sealed d. solidified
12. a. beaches b. resorts c. areas d. parts
13. a. growing b. shrinking c. swelling d. reducing
14. a. tops b. faces c. features d. hangings
15. a. bounce b. jump c. splash d. ripple

115
b. Read the passage carefully.
My lawyer, Mr Turner, is the only man I know who has seen the ghost. He is a quiet even-tempered man
whose life is spent in dealing with facts. He is the last person in the world to give way to fantasy. He has a wife
and two children of whom he is proud, takes a modest holiday abroad every year and spends his Sundays
gardening. He is knowledgeable about art and architecture, though he doesn’t pretend to be an expert by any
means. It is, therefore, all the more surprising that he should be so insistent about the ghost. It happened, so he
says, like this:
He was travelling from London to the north of England by train. It was a misty November evening and the
train was half empty. In fact, for the first part of the journey Mr. Turner had the carriage to himself and sat
dozing over a newspaper. However, at the first stop a passenger jumped in, slamming the door behind him. He
seemed out of his breath as if he had been running. He was a striking looking young man with dark, bushy hair
and bright intelligent eyes. He was dressed rather oddly in a long waistcoat with silver buttons, tight trousers
and embroidered waistcoat. Mr. Turner didn’t pay much attention to this because people wear all sorts of
extravagant clothes these days and he had long grown accustomed to them.
Presently, the two men got into conversation, as people do on long journeys. Mr. Turner was interested to
discover that the young man was very knowledgeable about art - in particular portraits. His name, he said, was
Joseph Hart, and he was on his way to visit an exhibition. It seemed that he worked in a famous London Art
Gallery - a picture restorer, perhaps, thought Mr. Turner, he seemed to know a great deal about varnishes and
paints, and even more about the subjects of certain portraits. When Mr. Turner asked his opinion of the portrait
of a famous judge by an artist he admired, his companion laughed and said: “He’s only a reproduction - a good
one I agree but you can’t talk to a reproduction”. He spoke as though the person in the portrait were still living.
After a while the carriage got hot and steamy and Mr. Turner dropped off. He woke up just as the train was
drawing up at a junction with a grinding of brakes. His companion had disappeared.
A few days later, having returned to London, Mr.Turner found himself near the Art Gallery. Moved by some
impulse, he went in and inquired for Joseph Hart. The attendant directed him to a room devoted to early
nineteenth century portraits of well-known men. There was no one in the room and Mr. Turner looked around
him. Without knowing quite how he had got there, he found himself. standing in front of a full-length portrait
of a young dark man in tight trousers and an embroidered waistcoat. The eyes smiled at him with a hint of
amusement. The name-plate at the foot of the picture read: Joseph Hart, Gentleman, 1800-1835.
1. What kind of person was Mr. Turner?
A. Imaginative B. Fantastic C. Sensible D. Insensitive
2. Although he was a lawyer, Mr. Turner_________.
A. pretended to know a lot about art. B. knew something about art
C. pretended to take interest in art. D. intended to learn more about art.
3. When the passenger entered Mr. Turner’s department, _________.
A. he was panting B. he was running
C. the train was just training D. the carriage was half-empty.
4. The passenger’s clothes didn’t seem strange to Mr. Turner because_________.
A. he was used to wearing strange clothes. B. he liked people who wore strange clothes
C. everyone he knew wore strange clothes. D. he had seen a lot of people in strange clothes
5. Mr. Turner thought the young man might_________.
A. be an Art Dealer B. be an Art Expert
C. renew old pictures D. paint reproductions of old pictures
6. Why wouldn’t the young man give an opinion on the portrait of the judge?

116
A. The judge wasn’t alive. B. The judge was still alive.
C. The picture was a copy. D. He hadn’t seen it.
7. When did Mr. Turner first realize that the passenger had gone?
A. When the train started. B. After the train had stopped.
C. Just before the train stopped. D. When the train was leaving the. station.
8. Why did Mr. Turner go into the Art Gallery?
A. He was walking past there. B. He had never been there before.
C. He has planned to do so D. He suddenly decided to.
9. In the part of the Gallery that Mr. Turner was directed to, _________.
A. there were a lot of pictures of unknown people
B. there were a lot of nineteenth century people
C. no one else was looking at the pictures
D. he only saw one portrait.
10. When Mr. Turner looked the portrait of Joseph Hart, _________.
A. he smiled at it B. he thought it smiled at him
C. he didn’t recognize it D. he was amused
IV. WRITING
Rewrite the following sentences using the word in brackets.
1. Nobody helped me at all (finger)
____________________________________________________________________
2. I don’t mind discussing it with you. (willing)
____________________________________________________________________
3. She found it hard to concentrate on her work because of the noise. (difficulty)
____________________________________________________________________
4. They are going in the direction of Ha Long Bay. (making)
____________________________________________________________________
5. He was partly responsible for his own failure. (contributed)
____________________________________________________________________
6. It seems that whatever I try to do, something bad always happens. (goes)
____________________________________________________________________
7. My brother kindly lent me his car for the weekend, (favor)
____________________________________________________________________
8. She has to do much. (Much…)
____________________________________________________________________
9. The weather forecast was inaccurate, so we didn’t take the right precaution. (Had.....)
____________________________________________________________________
10. An argument between the management and the staff resulted in the postponement of the meeting. (put)

117
SỞ GIÁO DỤC VÀ ĐÀO TẠO HÀ NỘI
KỲ THI TUYẾN SINH VÀO LỚP 10 THPT CHUYÊN
Năm học 2009-2010
Môn thi: Tiếng Anh
Ngày thi: 25 tháng 6 năm 2009
Thời gian làm bài: 120 phút
I. PHONETICS (l.pt)
a. Choose the word whose underlined part is pronounced differently from the rest
1. A. begun B. funny C. upwards D. autumn
2. A. teacher B. Deaf C. reach D. team
3. A. cough B. enough C. although D. laugh
4. A. watching B. machine C. matches D. kitchen
5. A. asked B. helped C. prayed D. dressed
b. Choose the word whose main stressed syllable is different from the rest.
6. A. profitable B. Ordinary C. definitely D. delicious
7. A. disappointed B. mysterious C. electronic D. economic
8. A. moderation B. reputation C. competitor D. innovation
9. A. popularity B. convenient C. controller D. embarrassing
10. A. technological B. personality C. representative D. environmental
II. VOCABULARY AND GRAMMAR. (4.0 pts)
a. Circle the best option to complete the sentence.
11. Tell me_________
A. what do you want B. you want what C. What you want D. That what you want
12. If I_________- be somebody else, I_________to be a film star.
A. could/would B. Can/would like C. Could/would like D. Can/ like
13. When you buy something, you are usually given a_________.
A. recipe B. Receipt C. Bill D. Prescription
14. Don’t forget to give Mr Brown my message, _________?
A. do you B. Are you C. Will you D. Should you
15. Bob got fired. It’s difficult for him to find_________job.
A. the others B. The another C. Another D. Other
16. Look at these two pieces of material I have just bought. Which do you like_________?
A. most B. More than C. Best D. Better
17. The factory paid_________nearly a million pounds to their employees who were injured in the explosion.
A. off B. Out C. Back D. In
18. This_________is best served with boiled rice and a crisp green salad.
A. plate B. Meal C. Food D. Dish
19. A good friend will always_________you when you’re in trouble.
A. stand for B. Stand up against C. Stand by D. Stand up
20. She came in, picked up her things, and left before I could_________a sound.
A. pronounce B. Tell C. Express D. Utter
21. Remember to go_________your test paper to check for grammar and spelling mistakes.
A. on B. Over C. Off D. Into
22. Unemployment is a serious problem in the area. There are_________jobs for people there.

118
A. a little B. A few C. Few D. Little
23. He has always wanted to see his name in_________
A. paper B. News C. TV D. Press
24. When the clock_________twelve, we raised our glasses and drank to celebrate the New year.
A. struck B. Beat C. Hit D. Shot
25. Lenny, _________the TV programme. I want to listen to the news.
A. turned off B. Turns off C. Turns on D. Turn on
26. You will know the result_________two weeks.
A. on B. About C. In D. During
27. She spoke quietly to him_________nobody else could hear a word.
A. because B. Although C. Despite D. So that
28. I got very nervous during the exam. When the examiner asked my name, my mind wen completely __
A. empty B. Blank C. White D. Void.
29. _________the salary meets my expectations. I will accept the job offered.
A. Provided that B. Due to C. Even if D. Unless
30. His father retired early_________being unhealthy.
A. on behalf of B. Ahead of C. On account of D. In my view
b. Choose the word or phrase that is closest meaning to the underlined ones.
31. First year students are required to undertake simple experiments.
A. submerge B. incover C. Attempt D. Encounter
32. The reason for a fall in the death rates is the improvement of public health services and medical care.
A. degradation B. policy C. progress D. step-back.
33. He testified that he had seen the man leaving the building around the time of the murder.
A. said B. gave evidence C. Disapproved D. Denied
34. “Look at that gymnast over there!. Her joints must be really easily bent without breaking”.
Anomy commented.
A. elastic B. Pliable C. Flexible D. Slippery
35. I’ve heard the education ministry will be bringing in some new methods of teaching english as a second
language soon.
A. presenting B. Commencing C. Starting D. Introducing
c. Complete each sentence with the suitable verb form in column A with a preposition in column B. No need to
use all.
A B
Make land give come reduce increase meet get in for up down to at with on

36. Her behavior has___her___trouble with her teacher.


37. We________after school on Tuesdays and go to someone’s house to play computer games.
38. Carol has___up___a great new idea to make money.
39. The other children were so cruel that they___him___tears.
40. The runner was slow at first but she___up___it in the last few metres and won.
d. Complete the sentence with the correct form of the word in brackets.
41. “Don’t worry about that ink stain on your blouse. It’s hardly (notice)__________”, Mary told.
42. “Oh, I give up. I just can’t seem to get through to this number!”.
- “Why don’t you try asking the (operate)__________to help you?

119
43. The book is long, uninteresting and not very well-written. I find it(read)__________
44. She is one of the greatest (perform)__________-to appear in this theatre.
45. Intensive cultivation has (poverty)__________the soil
46. He achieved (notorious)__________for failing a drugs test after winning an Olympic final.
47. “No, I don’t have any pets in our class”. I don’t believe in (favourite)__________I just treat all my students
the same”, one teacher was over-head telling another.
48. Dr Kevin wants to be one of the most famous (environment)__________-in the world.
49. ‘I don’t really like that man over there. He’s rather (argue)__________. I nearly got into a fight with him
once, too”. Robert said.
50. The (extend)__________of his knowledge surprised the board of examiners.
III. Reading (2,0 pts)
a. Give a suitable word in each blank
Kiyo Tanaka skillfully steers her wheelchair between the flower beds as she gardens outdoors. Gardening is
one of her (51)________year-round activities. Believe it or not, thirteen-year-old Kiyo also successfully (52)
________plants indoors without soil. She fills plastic (53) ________with a special solution for nourishing
plants. She uses a (54) ________of screen to support each plant in container, allowing the roots to dangle into
the solution so that half of the root system is (55) ________. Every few weeks, Kiyo pours our the old
solution, flushes the container with clear water and then pours in flesh solution, which she buys at a gardening
center. With this kind of indoor gardening, Kiyo doesn’t have to worry (56) ________-outdoor garden pests,
poor soil, or bad weather. She keeps the plants on shelves where they can get good (57) ________-and where
she can reach them (58) ________. Her indoor plants are a living testimonial to the good care she gives them.
b. The people all below want to visit a museum. There are 8 description of museum. Decide which museum
would be the most suitable place for the following people. Write the answer (A- H) next to number.
59__________Cristina wants to visit a museum with her daughter to see how people used to live. They
want to have Inch there and buy some presents to take home. They will go by car.
60__________Joanne wants to visit a museum and see people making things. She has no car and would like to
have lunch there.
61__________Carl’s hobby is painting pictures, so he would like to see the work of other painters who live in
the area. He wants to have a snack at the museum. He travels by public transport.
62__________Duncan wants to find out where local people worked in the past and what they did in their spare
time. He wants to buy a book about the exhibition. He travels by bus.
63__________The Cannavaro family want to walk around a site which offers opportunities for the
whole family to take part in activities. They want some where pleasant to eat their own sandwiches. They have
a car.
MUSEUM IN THE AREA
A. Stackworth Museum tells the history of the famous Stackworth family, and gives information about
other well-known local people. These include poets, artists and writers. There is an excellent cafe and a car
park.
B. Charberth Museum is near the main bus station and has a rich collection of objects, 19th - century
paintings and photographs showings life in the town over the centuries- the jobs people did and how they
entertained themselves. An accompanying book showing the works on display is available from the Museum
Shop as well as some attractive gifts. There is no café.

120
C. Fairley Museum is arranged like an old - fashioned village. You can see people working at their trades
to produce tools, pots and even boats using traditional skills. There is a small picnic area in the car park but
most people eat in the excellent café. The museum is on a bus route.
D. Westerleigh Museum is near the bus station, and contains exhibitions showing the town’s development.
In a separate room there are works by some well- known artists as well as changing exhibitions of work by
local artists. Sandwiches, cakes and hot drinks are on sale in the cafe.
E. The rooms in Scotwood Museum are famished as they were 100 years ago. The staff spend the day as
people did then and are happy to explain what it was like. There are activity sheets for children and a shop
with books, souvenirs and cards, as well as a good cafe and car park.
F. Freshwater was once an important fishing port. Freshwater Museum, inside the old harbour office,
shows how the town developed and later became a tourist centre. There is an activity room for young
children with DVDs, a large picnic area, car park, and good bus service.
G. Set in beautiful countryside, the Woodlands Museum is arranged like a village of 100 years ago. To learn
more about this period, visitors are encouraged to spend time doing practical things such as making pots and
cooking. There is an adventure playground with a picnic area under the trees and parking.
H. Middleworth Museum is full of objects from the past, which tell the story of different people who
worked in the area, from factory workers to the men who built the canal and the railway. There is a Family
Folder of things to do. The museum has a café and is near the bus and railway station.
c. Read the passage and then answer the questions.
No smoking!
Smoking causes lung cancers. Lung cancer is the number one cancer among men. Ninety percent of the
people who get lung cancer die. Smokers have six times as much cancer of the mouth, tongue, and throat as
non-smokers. Smokers have twice as much heart disease. Smoking gives people breathing problems. It harms
the stomach. Smoking causes one million early deaths in the world every year.
Throughout the world, women live longer than men. Why? Because men smoke more. Nonsmoking men
live just as long as women. Smokers pollute the air. They breathe out 100 times as much pollution as
nonsmokers. They breathe it out on their children and on their wives or husbands. Children whose parents
smoke have more breathing and lung problems than other children. Among nonsmoking women married to
smokers, there are twice as many deaths from lung cancer as there are among women married to non-smokers.
Men have the right to kill themselves by smoking if they want to. They do not have the right to kill their wives
and children by smoking
Smokers are absent from work more. Factories and businesses lose millions of dollars every year because of
smokers. Smokers have higher medical bills. All of these facts from research show that smoking is bad. So
why do people smoke. Young people start smoking because everyone else is doing it. They feel grown up and
independent. It helps them relax. Then it becomes a habit.
It is very difficult to stop smoking, but the chance of getting lung cancer decreases only one year after you
stop. After ten years, the chance is almost the same as for nonsmokers. It is possible to quit smoking. Thirty
million people in the United States have quit because they believe the research.
64. How many people die when they get lung cancer?
65. What does smoking cause harm to the body?
66. What happens to children when their parents smoke?
67. Why do young people smoke?
68. What are the chances of getting lung cancer ten years after people stop smoking?
69. Why do smokers pollute the air?

121
70. Why do the factories and business lose millions of dollars every year?
IV. Writing. (3 pts)
a. Finish each of the sentence
71. Mr Jonse remembered the date, and so did his wife
→ Mr Jones didn’t__________________________________________________
72. He didn’t try to conceal his dislike to me.
→ He made no_____________________________________________________
73. The best solution was thought of by Sally.
→ Sally came______________________________________________________
74. “We moved here in 1998”, said John.
→ John said_______________________________________________________
75. The magazine is published every week.
→ The magazine comes______________________________________________
76. John’s mother doesn’t agree if he goes to work. (disapproves)
_________________________________________________________________
77. You are so lucky to have inherited all that money. (come)
_________________________________________________________________
78. Paula always got angry when she made a mistake. (temper)
_________________________________________________________________
79. Every on who spoke to the victims is a suspect. (under)
_________________________________________________________________
80. Don’t be deceived by his long words. (in)
_________________________________________________________________
81-100. Write no more than 150 words to say about the energy waste problem around you and suggest the
solutions to save energy.
---THE END---

122
SỞ GIÁO DỤC VÀ ĐÀO TẠO HÀ NỘI
TRƯỜNG THPT CHUYÊN HÀ NỘI AMSTERDAM
ĐỀ THI OLYMPIC (2008-2009)
MÔN TIẾNG ANH LỚP 10 (HỆ KHÔNG CHUYÊN)
Ngày thi: 27 tháng 3 năm 2009
Thời gian làm bài: 60 phút
I. PHONETICS
a. Choose the word whose underlined part is pronounced differently from the rest.(Spts)
1. A. Noon B. food C. Foot D. moon
2. A. climber B. bomb C. subtle D. subtitle
3. A. admiral B. admire C. admirable D. addict
4. A. Creature B. creative C. creamy D. crease
5. A. future B. culture C. fortunate D. mature
b. Choose the word whose main stressed syllable is different from the rest (5pts)
6. A. Facilities B. confirm C. comedy D. refrigerator
7. A. photography B. preference C. wilderness D. outdoor
8. A. Visual B. insert C. technical D. miracle
9. A. responsibility B. submarine C. comprehend D. recommend
10. A. diversity B. amphibian C. Heritage D. surprise
II. VOCABULARY AND GRAMMAR
a. Circle the best option to complete the sentence. (15 pts)
11. An announcement about the 8 o’clock flight to New York________
A. has been already made B. Has already made
C. Already made D. Has already been made
12. She is extremely competent and________
A. industrial B. Industrious C. Industry D. Industrialized
13. People should change their inappropriate lifestyle________destruction to natural reverse.
A. that causes B. Caused C. Causing D. A & C are correct
14. Tears contain an antiseptic that helps protect our eyes________infection
A. from bacterial B. In bacterial C. Bacterial D. With bacterial
15. Artists have painted nature________centuries.
A. for B. In C. Since D. From
16. ________English, which has no gender system, German has masculine, feminine and neuter.
A. Besides B. Except C. Apart D. Unlike
17. Palma will have to________her antiques, because she needs the money.
A. part with B. Take out C. Move on D. Clear up
18. “I found these books. Do you need them?”. - oh, yes. Those are the ones I________earlier.
A. was looking for them B. Looking for C. Am looking for D. Was looking for
19. The whole family was in________with her about what they should do.
A. agreeing B. Agreeable C. Agreement D. Agreeably
20________more help, I could call my neighbor.
A. needed B. Should I need C. I have needed D. I should need
21. I didn’t get home until after mid-night last night. Otherwise, I________your call.
A. returned B. Had returned C. Would return D. Would have returned

123
22. The total weight of all the ants in the world is much greater than________
A. to all human beings B. All human being
C. that of all human being D. Is of all human beings.
23. Without music, the children wouldn’t________so much fun.
A. behaving B. Be C. Have had D. Have been
24. Charles Darwins’s first scientific book, published in 1842________a substantiated theory on the
origin of coral reefs and atolls.
A. to present B. Presented C. Presenting D. Presents
25. He was told he could join the Scouts when he________old enough.
A. would be B. Should be C. As D. Will be
II. Fill in each blank with the suitable word picked out from the box, (10 pts)

Cut-off failures pests prolonged throughout barely widespread


flooding farmland developing

Famine is a (26)_________food shortage that causes (27) _________hunger and death. (28) _________
history, famine has struck at least one area of the world every few years. Most of the (29) _______ nations of
Africa, Asia and Latin America have (30) _________-enough food for their people. Millions in these countries
go hungry. When food production or imports drop for any reason, famine may strike and thousands or millions
of people may die.
Many famines have more than one cause. For example, the great Bengal famine of 1943 in Eastern India
was caused by both natural and historical events. World War n resulted in a general food shortage and let to the
(31) _________of rice imports from Burma, which had been occupied by the Japanesse. Then a cyclone
destroyed much (32) _________Famine struck, and more than 1.5 million people died.
Nearly all famines resulted from crop (33) _________ - The Chief causes of them included drought, too
much rainfall and (34) _________, plant disease and (35) _________. Many other factors may also help create
a famine.
III. Each sentence has a mistake. Find and correct it (10 pts)
36. Diamonds are graded according to weigh, color and cut
37. Many seeds remain dormantly in the ground until conditions involving heat and water are correct for
germination.
38. Soya beans contain not only vitamin and also important minerals.
39. The brain is made up of billions of neutrons that differ with each other greatly in size and shape.
40. Electric timing used for the first time in the 1912 Olympics in Sweden.
41. Some fish senses distortions of electrical field through special receptors.
42. The brain loses fifty thousands neutrons a day and yet maintains its basic patterns and memories
43. Penguins have special glands who remove salt from the water they drink and the food they eat.
44. Coral reefs are limestone formations composed of tiny sea organisms and the remains.
45. It is only in the last 200 years as people have begun climbing mountains.
IV. Use the correct form of the verb in bracket. And you have to add particles or preposition to the verbs to
make them meaningful. (10 pts)
46. When the clock stopped, he______it______, found what was wrong and put the whole thing together
again. (Take)
47. I have a part-time job in a shop, which______about US $500 a month (bring)

124
48. we should______-our differences and discuss the things we have in common. (put)
49. Sorry, we have______orangeade. Would you like some water? (run)
50.She became an orphan after her mother was______by cancer at the age of 40. (strike)
51. The fire brigade should always be______to a house fire. (call)
52. Look! The resident in our neighborhood are______fireworks to celebrate Christmas. (let)
53. I’m really______-my holiday in Italy next month.(Look)
54. I can’t advise you on that. You’d better______it______with your mother. (Take)
55. Armed terrorists are reported to have______- the Embassy. (take)
c. Reading
I. Fill in blank with a suitable word (8 pts)
When you are (56)_________interviewed for a job, remember that it’s normal for many people to be
nervous, particularly in a stress producing situation. There are plenty of jobs where nervous is frown (57)
_________. You should know (58) _________sure that interviewers want people will (59) ________ into their
company and be (60) _________-for the job. (61) _________, applicants should be confident (62)passing this
mental (63) _________successfully.
II. Read the passage and choose the best answer (7 pts)
It is common believed in the United States that school is where people go to get an education. Nevertheless,
it has been said that today children interrupt their education to go to school. The difference between
schooling and education implied by this remark is important.
Education is much more open-ended and all-inclusive than schooling. Education knows no bounds. It can
take place anywhere, whether in the shower or on the job, whether in the kitchen or on the tractor. It includes
both the formal, learning that takes place in school and the whole universe of informal learning. The agent
(doer) of education can vary from respected grandparents to the people arguing about politics on the radio,
from a child to a famous scientist. Whereas schooling has a certain predictability, education quite often
produces surprises. A chance conversation with a stranger may lead a person to discover how little is known
of other religions. People receive education from infancy on. Education, then, is a very broad, inclusive term; it
is a lifelong process, a process that starts long before the start of school, and one that should be a necessary
part of one’s entire life.
Schooling, on the other hand, is a specific, formalized process, whose general pattern varies little from one
setting to the next. Throughout a country, children arrive at school at about the same time, take the assigned
seats, are taught by an adult, use similar textbooks, do homework, take exams, and so on. The pieces of reality
that are to be learned, whether they are the alphabet or an understanding of the workings of governments, have
been limited by the subjects being taught.
64. What does the author probably mean by using the expression “children interrupt their education to go to
school” (lines 2-3) ?
(A) Going to several different schools is educationally beneficial.
(B) School vacations interrupt the continuity of the school year.
(C) Summer school makes the school year too long.
(D) All of life is an education.
65. The word “bounds” in line 4 is closest in meaning to
(A) rules (B) experience (C) limits (D) exceptions
66. The word “chance” in line 11 is closest in meaning to
(A) unplanned (B) unusual (C) lengthy (D) lively
67. The word “an integral” in line 15 is closest in meaning to

125
(A) an equitable (B) a profitable (C) a pleasant (D) an essential
68. The word “they” in line 20 refers to
(A) slices of reality (B) similar textbooks (C) boundaries (D) seats
69. The passage supports which of the following conclusions?
(A) Without formal education, people would remain ignorant.
(B) Education systems need to be radically reformed.
(C) Going to school is only part of how people become educated.
(D) Education involves many years of professional training.
70. The passage is organized by
(A) listing and discussing several educational problems
(B) contrasting the meanings of two related words
(C) narrating a story about excellent teachers
(D) giving examples of different kinds of schools
D. Writing
I. Rewrite each sentence so that it meaning is the same with previous one. (5 pts)
71. Everyone thought that he had stolen the car.
→ The car_____________________________________________________________
72. We weren’t surprised by his success.
→ It came_____________________________________________________________
73. They were seasoned travelers, which we had not expected them to bed
→ Contrary____________________________________________________________
74. He did not pass his driving test until he was nearly 30.
→ It__________________________________________________________________
75. Without his help, we would all have denied.
→ Had it______________________________________________________________
II. Rewrite full sentences, using the suggested words. (5 pts)
76. Scientists/ be/ shock/ find out/ more/ one million species/ lose/ 2050.
______________________________________________________________________
77. There/ be/ far/ few/ people/ the meeting/ I/ expect.
______________________________________________________________________
78. It/ be/ not worth/ try/ explain/ everything/ him.
______________________________________________________________________
79. Without/ his father/interfere/ everything/ would/ go/ smoothly/ yesterday.
______________________________________________________________________
80. He/ be/ most/ likely/ person/ succeed/ solve/ the problem.
______________________________________________________________________
—The end—

126
SỞ GIÁO DỤC VÀ ĐÀO TẠO HÀ NỘI
KỲ THI TUYỂN SINH VÀO LỚP 10 CHUYÊN
TRƯỜNG THPT CHU VĂN AN & HÀ NỘI AMSTERDAM
Năm học: 2007 - 2008
Môn thi: Tiếng Anh
Ngày thi: 21-06-2007
Thời gian làm bài: 150 phút
I. PHONETICS (1 pt)
a. Which word has the underlined part pronounced differently from the others?
1. A. baggage B. courage C. damage D. stage
2. A. depended B. needed C. wanted D. striped
3. A. kitchen B. Christmas C. challenge D. exchange
4. A. advertise B. addition C. advantage D. adventure
5. A. thunder B. ethnic C. health D. gather
b. Circle the word whose main stress syllable is different from the rest.
6. A. prevention B. agreement C. satisfy D. experience
7. A. camera B. volcano C. establish D. industrial
8. A. elephant B. violent C. agree D. mineral
9. A. exist B. advance C. pagoda D. purpose
10. A. linguistic B. experience C. dormitory D. communicate
II. VOCABULARY AND GRAMMAR (4pts)
a. Circle the best one A, B, C or D and write it in the blank to complete the sentence.
11. One problem for any teacher is that each student has his own________needs.
A. separate B. divided C. individual D. distinctive
12. Good theories are important of course, but we must have________evidence to support them.
A. empirical B. true C. realistic D. first-class
13. If we hurry, we might get there________to catch the early train.
A. right B. in time C. on time D. before time
14. Joining a newsgroup allows computers users to make________with other who share a similar
interest.
A. fun B. touch C. approach D. contact
15. To get a passport, you must send in your birth________and two photos.
A. certificate B. license C. paper D. card
16. His face looks________, but I can’t remember his name.
A. similar B. alike C. memorial D. familiar
17. In special______, a person who is found guilty of murder may receive no punishment at all from court.
A. places B. times C. surroundings D. circumstances
18. She tries to set________ an hour a week for practice.
A. about B. down C. aside D. in
19. ________stay the night if it’s too difficult to get home.
A. At all costs B. By all means C. In all D. On the whole
20. There’s no________to book tickets in advance. There are always plenty of seats.
A. problem B. matter C. need D. demand
21. Robert and his wife________to my house for tea yesterday evening.

127
A. came round B. came about C. came down D. came away
22. Our house, which________by the river, has a beautiful and modern design.
A. is situated B. situates C. situate D. has situated
23. Each of the guests________a bunch of flower.
A. are given B. is given C. were given D. give
24. I regret________you the story. I really didn’t know it would make you disappointed.
A. tell B. telling C. to tell D. told
25. I was made________had when I was at high school.
A. studying B. studied C. to study D study
26. To everyone’s surprise, Mr. Brown________at the Trade Union meeting.
A. turned in B. turned over C. turned up D. turned round
b. Fill in each blank with the correct form of the word in capital letters.
27. Every________in this country has the right to vote. CITY
28. This city has over half a million________. INHABIT
29. Our company helps________people to find new jobs. EMPLOY
30. According to the________, the French lesson starts at ten. TIME
31. I can’t sit on this chair. It’s really________. COMFORT
32. Harry asked for a________and the cashier gave him one. RECEIVE
33. The conditions in the man’s prison were________. HUMAN
34. They frequently________the traffic as they march through the street. MOBILE
35. In his family, he is a________child. TROUBLE
36. ________are doing the best to make people aware of the danger of all pollution. ENVIRONMENT
c. Fill in the gaps with prepositions or particles.
37. Is it possible to insure my bike________theft?
38. She was very critical________her friends’ new furniture.
39. I’m afraid I took your coat________mistake.
40. He reluctantly consented________his daughter’s marriage.
41. Government officials are looking________reports that the river is being polluted by a chemical factory.
42. Jack, you look tired. Are you________the weather?
43. The doctor cured me________my illness.
44. ________luck, we should be in Japan by 4 p.m.
d. Fill in each blank with the correct tense of the verb in the brackets.
Nick disliked school. Being the class truant, he (45) ________ (not realize) the importance of education.
Then, one day while he (46) ________ (play) truant, he met a beggar. Curious, the beggar (47) ________ (ask)
him why he was not in school. Knowing the reason, the beggar (49) ________ (warn) Nick that if continued as
he was, he (49) ________ (eventually end) up his life. Plagued by fears, Nick decided to change for the better.
His sudden change (50) ________ (surprise) all his teachers. His improvement became obvious to all.
III. READING (2 pts)
a. Fill in each numbered blank with one suitable word.
If you decide to take a space trip, you should have to get ready a few months (51) __________the flight.
You must be in excellent (52)__________condition. You should run a lot, swim everyday, and do aerobics
and push-ups. You must get a letter from the (53) __________that shows you are in perfect health. Once you
get on the trip, you will be in a different (54) __________. You will see pictures (55) __________the Earth.
You may also find your country and other interesting places. You will be able to see the oceans, the big rivers,

128
and the tall mountains. You will be able to see (56) __________many times because you will (57) _______ the
earth 16 times a day! You will also be able to see stars that you couldn’t see from the earth. When you are in
orbit, you will be able to get out of your seat and walk in the (58) __________. You will be able to walk on the
walls or on the ceiling (59) __________in a circus. You will not weigh anything! You will (60)
__________totally free and enjoy the wonderful feeling you have never had before. If you were on board now,
you would experience those marvelous things. So do you think you will be able to take a space trip? Start to
dream now and your dream may come true someday.
b. Read the text and circle the best answer A, B, C or D to complete it.
Amelia Earhart, America’s most famous woman pilot, was born into a (61) __________family in 1897. At
the age of 23, she began taking flying lessons in California. In spite of having a number of crashes, which were
fairly common in the (62) __________days of aviation, she decided to make flying her career. In 1932 she
became the first woman (63) __________a solo flight across the Atlantic. She set out on May 20th, and not
only did she succeed in arriving safely, but she (64) __________to do it in record time as well. She was
awarded honours of all kinds. As her fame grew, she travelled around the country giving lectures and (65)
_______- women’s rights. In 1937, she decided to do something that (66) __________
woman had tried before to fly all the way round the world. She reached Australia, and set (67) __________ on
the last part of her journey across the Pacific on July 2nd. A few hours later, she sent a radio message saying
she was running short of (68) _________. Then there was silence. President Roosevelt immediately had the
area searched, but the 9 ships and 66 planes were unable to find any sign of her. Since her death, there have
been many (69) ________about her: some say that she was captured by the Japanese, others claim that she
lived on a South Pacific island with a fisherman. However, although many people have tried to work out
exactly what happened, no (70) __________of her or the plane has ever been found.
61. A. health B. healthful C. wealth D. wealthy
62. A. late B. sunny C. bright D. early
63. A. to make B. was made C. make D. made
64. A. managed B. succeeded C. wanted D. failed
65. A. prohibiting B. promoting C. providing D. promising
66. A. some B. not C. no D. any
67. A. on B. off C. in D. of
68. A. water B. food C. fuel D. money
69. A. claims B. lies C. stories D. objections
70. A. trace B. story C. sign D. sight
IV. WRITING (3 pts)
a. Circle the incorrect part (A, B, C or D) in each of the following sentences and then correct it
71-72. The plane delayed for more than one hour because of the bad weather.
A B C D
73-74. Will you please pass me the salt shaker and the bowl of the spaghetti?
A B C D
75-76. Last night we ate fresh fish with com and rice in dinner.
A B C D
77-78. I have never seen as many expensive cars than they were in that shopping center.
A B C D
79-80. The new brochures describing all our services were delivered to us late yesterday and were shipped
A B C D

129
out early tomorrow morning.
b. Rewrite these following sentences, keep the same meaning.
81 -82. Nobody expected her to lose, but she did. (against)
_____________________________________________________________________________
83-84. You must concentrate on your work more. (apply)
_____________________________________________________________________________
85-86. She will probably be elected. (stands)
_____________________________________________________________________________
87-88. The boss was annoyed because his secretary came to work late (objected)
_____________________________________________________________________________
89-90. The demand for tickets was so great that people queued day and night. (such)
_____________________________________________________________________________
c. (91-100) Write not more than 150 words inside this space to tell your friends how to protect the school
environment.
THE END

130
SỞ GIÁO DỤC VÀ ĐÀO TẠO HÀ NỘI
KỲ THI TUYỂN SINH VÀO LỚP 10 CHUYÊN
TRƯỜNG THPT CHU VĂN AN & HÀ NỘI AMSTERDAM
Năm học: 2006 - 2007
Môn thi: Tiếng Anh
Ngày thi: 21-06-2006
Thời gian làm bài: 150 phút
I. Phonetics. (1,0 point)
a. Which word has the underlined part pronounced differently from the others?
1. A. borrow B. prow C. growth D. low
2. A. nature B. natural C. native D. nation
3. A. near . B. fear C. rear D. pear
4. A. loud B. mouth C. southern D. boundary
5. A. prohibit B. exhibition C. dishonest D. dishonor
b. Which word has the different position of stress from the others?
6. A. education B. compulsory C. intelligent D. technology
7. A. decision B. redundant C. assistant D. management
8. A. theory B. delicious C. generate D. confidence
9. À. faithfully B. sincerely C. completely D. extremely
10. A. vegetable B. interesting C. volleyball D. detective
II. Vocabulary and Grammar. (3,0 points)
a. Circle the best one (A, B, C or D) and write it in the blank to complete the sentence.
11. We locked the door to__________our dog from getting away.
A. prevent B. keep C. block D. stop
12. If we go by the side-roads we can__________most of the traffic.
A. be away B. avoid C. keep away D. keep distant
13. The child was too tired to walk any further so I had to__________him.
A. take B. bring C. bear D. carry
14. As the year passed, Joe’s memories of his terrible experience__________away, and he began to lead a
normal life again.
A. faded B. backed C. passed D. got
15. The new code of practice doesn’t__________from the old one although there are some important changes.
A. different B. differ C. difference D. differentiate
16. This cheese has gone__________. We’ll have to throw it away.
A. contaminated B. decayed C. muddy D. stale
17. The__________his doctor is to get a certificate to say that he can still drive a car.
A. time only my father visits B. only time my father visits
C. time my father only visits D. time my father visit only
18. They were such good friends. I am surprised they__________and no longer speak to each other.
A. broke out B. shut out C. fell out D. put out
19. Scientists__________a carefully controlled experiment on the mystery virus.
A. carried over B. measured out C. carried out D. put up
20. His writing is so bad that I can’t__________what he writes.
A. make out B. make off C. make up D. make for

131
21. Her father died when she was small so she was__________by her aunt and uncle.
A. brought up B. grown up C. looked over D. taken after
22. Don’t__________! I haven’t finish explaining yet.
A. hang up B. hold on C. hang on D. call up
23. The two schoolboys were so angry with each other that they had a__________fight.
A. sit down B. sit up C. stand up D. knock-down
24. There is no time to do anything but__________important points.
A. touch B. run C. go D. pick
b. Fill in each gap with a suitable preposition or particle.
25. Helen had great admiration__________her history teacher.
26. I was__________the impression that you liked Indian food.
27. Your performance this term contrasts very favorably__________last term’s.
28. You are perfectly capable__________making your own bed, I would have thought.
29. We met at the hotel completely__________a coincidence.
30 “31. You should try to cooperate__________that organization__________ every way.
32. What is his attitude ’the plan?
c. Complete the text with words formed from the words in capitals.
Dear Sir or Madam,
I traveled last week on a Transworld Airbus from London Gatwick to Copenhagen. This was the
(33)____OUT____ journey of a holiday in Denmark, a (34)_______PACK____ tour arranged through a
company called “Sunset”. My (35)______FLY____was due to leave at 8.20 a.m. on Tuesday 25th November,
but did not in fact leave until 20.30, a delay of more than eight hours. The reason given was that vital
(36)_____MAINTAIN____ work had to be carried out. Although all passengers was given a free meal, no
other offer of (37)_____ASSIST____was given. Such long delay is totally (38)_____ACCEPT__ and I feel
justified in circumstances in requesting some forms of financial 39)_____COMPENSATE____. I have written
to the tour (40)____OPERATE____who denied responsibility and advised me to write to you. I look forward
to hearing from you.
Yours faithfully,
John Brown
III. Write. (3,6 points)
a. Put these words in order to complete the following sentences.
41-42. If / down / in / there / will / you / forests / be / floods / cut / trees /, / big / the / every year /the/.
_________________________________________________________________________________
43 -44. With / the / civilization /, / ever / fewer / advance / people / live / in / now / forests /, / but / are / more /
valuable / forests /than.
_________________________________________________________________________________
45-46. I / think / trash / to / of / we / the / should / to / environment / burn / garbage / protect / reduce /
produce / we / the / amount.
_________________________________________________________________________________
47-48. The / problem / view / computer / important / universe / of / about / the / is / it / has/ and / will / to/
man’s / of/ himself / what / and / do / his / all / place / most / the / done.
_________________________________________________________________________________
49-50. Suddenly /, / a huge / thrown / away / up /, / gas / and steam / was / dust / the / cloud / fifteen / into /
the / of / sky /, / ash / miles / and / fell / on / farms / rocks / and villages / to / twenty / miles / from / volcano.

132
_________________________________________________________________________________
b. Each sentence has a mistake. Underline and correct
51-52. Each of the students in the accounting class has to type then own research paper this semester.
53 -54. The recent developments in the field of robotics has been extremely beneficial to those who are
physically handicapped.
55-56. Reports on the rise in hotel costs come from authority sources in the industry.
57-58. Because the 70 years old building was structurally unsafe and too expensive to repair, it had
to be demolished.
59-60. The wooden fence surrounded the factory is beginning to deteriorate from rain.
c. Rewrite the following sentences using the words in bold, in such a way that it means the same as the one
given. Do not alter these words in any way.
61 -62. You can stay with us for a few day. (put)
__________________________________________________________________________
63 -64. They suspended Jackson for the next two matches. (banned)
__________________________________________________________________________
65 -66. New drugs are being discovered which are helping the fight against this disease. (discovery)
__________________________________________________________________________
67 -68. Mary breathed a sigh of relief when she realized that her money hadn’t been stolen. (relieved)
__________________________________________________________________________
69 -70. Jane bought the expensive coat without thinking about it first. (spur)
__________________________________________________________________________
71 -72. Everyone played a part in the football team’s success. (contributed)
__________________________________________________________________________
73 -74. If a group is so small that it cannot maintain a self-contained unit within the society, it may be
assimilated. (occur)
__________________________________________________________________________
75 -76. It is highly likely that language conflict will lead to political conflict if one linguistic group blocks the
social mobility of another. (likelihood)
__________________________________________________________________________
IV. Read. (2.4 points)
a. Fill in each numbered blank with one suitable word.
The word jeans comes from a kind of material that was made in Europe. The material (77)________ jeans,
was named after sailors from Genoa in Italy, because they (78)________clothes made from it. In the 18th
century jean cloth was made completely (79)________cotton and workers at that time loved (80) ___ it
because the material was very strong and it did not wear (81)______easily. In the 1960s, many university and
college students wore jeans. (82)________made different styles of jeans to (83)________the 1960s’ fashions:
embroidered jeans, painted jeans and so on. In the 1970s more and more people started wearing jeans because
they became (84)________. In the 1980s jeans finally became high fashion (85)___, when famous designers
started making theft own styles of jeans, with their own labels (86) ________them. Sales of jeans
(87)________ up and up. But in the 1990s the worldwide (88)________situation got worse, and the sale of
jeans stopped growing.
b. Read the text and decide which answer (A, B, C or D) best fits each space.
Interpreting the feelings of other people is not always easy, as we all know and we (89)________as much on
what they seem to be telling us, as on the (90)________words they say. Facial (91)and tone of voice are

133
obvious ways o showing our (92)_______to something, and it may well be that we unconsciously
(93)________views that we are trying to hide. The art of being tactful lies in (94)________these signals,
realizing what the other person is trying to say, and acting so that they are not embarrassed in any way. For
example, we may understand that they are (95)________reluctant to answer our questions, and so we stop
pressing them. Body movements in general may also (96)____feelings and interviewers often (97)_____
particularly attention to the way a candidate for a job walks into the room and sits down. However, it is not
difficult to present the right kind of appearance, while what many employers want to know relates to the
candidate’s character (98)________and psychological tests, and the further problem of whether such tests
actually produce (99)________ results. For many people, being asked to take part in such a test would be an
objectionable intrusion into their private lives. Quite (100) ________from this problem, can such tests predict
whether a person is likely to be a conscientious employee or a valued colleague?
89. A. estimate B. rely C. reckon D. trust
90. A. others B. real C. identical D. actual
91. A. looks B. expression C. image D. manner
92. A. view B. feeling C. notion D. reaction
93. A. express B. declare C. exhibit D. utter
94. A. taking down B. putting across C. picking up D. going over
95. A. at least B. above all C. anyhow D. in fact
96. A. display B. indicate C. simply D. infer
97. A. have B. show C. make D. pay
98. A. quirks B. mannerisms C. traits D. points
99. A. faithful B. regular C. reliable D. predictable
100. A. different B. apart C. away D. except

The end

134
SỞ GIÁO DỤC VÀ ĐÀO TẠO HÀ NỘI
KỲ THI TUYÊN SINH VÀO LỚP 10 CHUYÊN
TRƯỜNG THPT CHU VĂN AN & HÀ NỘI AMSTERDAM
Năm học: 2005 - 2006
Môn thi: Tiếng Anh
Ngày thi: 22-06-2005
Thời gian làm bài: 150 phút
I. Choose the correct word or phrase to complete the sentence.(2,0 pts)
1. We have bought extra food_________our friends stay to dinner.
A. In case B. If C. provided D. as long as
2. Peter asked what time the post office_________.
A. Close B. to close C. closing D. closed
3. He took great pride_________his success.
A. Of B. about C. in D. for
4. There’s nothing we can do to change their decision. We’ll just have to_________the best of the situation.
A. Have B. make C. give D. take
5. Would you mind keeping a (an) _________-on our house for us while we’are away?
A. Hand B. look C. eye D. view
6. It’s the best book I’ve ever read.
A. Which B. who C. that D. what
7. The boy was made_________-his homework.
A. To do B. do C. doing D. to doing
8. Could I have_________sugar in my tea, please?
A. Any B. some C. little D. few
9. Mary_________eat a lot of sugar when she was young.
A. Was used to B. usually C. did use D. used to
10. He arrived at the airport with only_________luggage.
A. A little B. a few C. any D. a lot of
11. I suggested_________the job because she badly needed work.
A. Her B. to her C. her for D. for her
12. You’re better see a doctor, _________you?
A. Hadn’t B. didn’t C. wouldn’t D. don’t
13. Because Mary’s parents worked abroad, they sent her to a_________-school.
A. College B. secondary C. training D. boarding
14. In order to_________with his studies, he worked through the summer.
A. Take on B. catch on C. take up D. catch up
15. So they’re your favorite group. Have you seen them?
A. Alive B. live C. living D. lively
16. The sky began to darken as the wind_________
A. Setup B. put in C. got up D. turned out
II. For question 1-10, read the text below. Use the word given in brackets to form a word that fits in each
numbered space. (1,5 pts)
How many times have you come back from what was meant to be a relaxing holiday and said, “If only I had
stayed at home”. Why are holidays often more stressful than staying at home?. It is not actually very (1.

135
Surprise) _________that foreign travel is (2.tire) _________and it is not just because of the distances involved.
A (3. Success) _________trip needs planning and very careful (4. Prepare) _________-this is hard work.
Having set off, you will probably have to spend hours in stuffy airports because of (5. End) _________delays.
(6. Fly) ________, as everyone knows, is itself a stressful experience for most people. Finally, you arrive in an
(7. Familiar) _________environment with perhaps no (8. Know) _________ whatsoever of the local language.
It is almost like becoming a child again. One feels so (9. Help) _______
and stupid. Imagine not being able to explain what you want to eat to a (10. Wait) _________or where you are
staying to a taxi driver.
III. Put each verb in brackets into a suitable verb form (2,5 pts)
I was on time for my dentist’s appointment, but the dentist was still busy with another patient, so I
(1 .sit)_________in the waiting room and (2. Read)_________-some of the old magazines lying there. While
I(3) (wonder) whether to leave come back another day, I(4) (notice) a magazine article about teeth. It(5)
(begin)_________: ‘How long is it since you last(6)(go)_________to the dentist? (7) (you go) regularly every
six months? Or (8) (you put off) _________your visit for the last six years?’ Next to the article was a cartoon
of a man in a dentist’s chair. The dentist(9) (say) _________: ‘ I’m afraid this(10) (hurt) _________.’ I(11)
(suddenly realize) _________that my tooth(12) (stop) _________aching. But just as I(13) (open)
_________the door to leave, the dentist’s door (14) (open) _________. ‘ Next please,’ he (15) (call)
_________, as the previous patient -(16) (push) past me. ‘Actually, I’m not here to see you, I (17) (wait)
_________for my friend,’ I (18) (shout) _________, leaving rapidly as I could.(19) (you ever do)
_________this kind of thing? Surely I can’t be the only person who(20) (hate) _________the dentists!
IV. Complete the dialogue. (1 pts)
Town Information Office, Brighton, Sussex
Tourist: Good morning. Could you suggest what we could visit here?. Are there any old buildings, for
example?.
Officer: Well, there’s the Royal Pavilion. It’s very famous.
Tourist: ______________
Officer: Take the number 14 bus from here; it goes straight there.
Tourist: ______________
Officer: At quarter to and quarter past the hour.
Tourist: ______________
Officer: Oh, it only takes about ten minutes so it’s not so far.
Tourist: ______________
Officer: Everyday, including weekends, opening hours are 10 A.M to 4 P.M
Tourist: ______________
Officer: No, not very. It costs 200 pounds for adults and 50p for children.
Tourist: ______________
Officer: Well, why don’t you take a family ticket?. That’s much cheaper?
Tourist: ______________
Officer: You are very welcome. Thank you.
V. Finish each of the following sentence (1 pts)
1. The book was so interesting that I couldn’t put it down.
→ It was__________________________________________________________
2. I tried to tell him to give up smoking. I didn’t succeed, however.
→ Although_______________________________________________________

136
3. I thought this journey would last longer than it did.
→ This journey didn’t_______________________________________________
4. The doctor can’t see you earlier than Wednesday, I’m afraid.
→ Wednesday _____________________________________________________
5. I understood Hamlet only after seeing it on the stage.
→ Only after seeing_________________________________________________
6. How old do you think this house is?
→ When do you think_______________________________________________
7. She didn’t work hard enough so she lost her job.
→ The reason she___________________________________________________
8. She asked a policeman the distance to the nearest post office.
→ “How___________________________________________________________
VI. Read the text below and think of the word which best fits each space. (2pts)
A family portrait is a valuable picture - that is fun to (1)______at now, it’s great for relatives far (2)__, and
it will bring back memories in the years to (3)_________Family change quickly as children (4)______,
therefore don’t wait, whatever your position in the family - photograph your family group now, and plan to
make this (5)__________regular event. Your family album isn’t really complete (6)_________-this record of
all of you together. Getting the painting of the family together (7)_________always easy, and so you will need
to plan ahead to be sure (8)_________has time to pose. A relaxed, friendly feeling is thing that (9)_________“
the picture, and you can’t expect people to relax (10) _________-they’re in a hurry to do (11)_________else.
Make your plans when you’re all together and with a cheerful, friendly (12)________- say, during a meal, and
set a time convenient (13)_________- everyone. A family portrait takes some technical planning, too. Make
(14)_________your mind in advance (15)_________room you want to use; (16)_________your camera
position and check the lighting. If you want to be in the picture, make sure you know exactly (17)_________-
the self-timer on your camera operates. With most (18)_________, you’ll have from eight and twelve seconds
to get into the picture (19) _________you press the shutter button.

THE END

137
SỞ GIÁO DỤC VÀ ĐÀO TẠO HÀ NỘI
KỲ THI TUYỂN SINH VÀO LỚP 10 CHUYÊN
TRƯỜNG THPT CHU VĂN AN & HÀ NỘI AMSTERDAM
Năm học: 2001 - 2002
Môn thi: Tiếng Anh
Thời gian làm bài: 150 phút
I. There are 10 mistakes in this letter. Please find and correct. (1,25pt)
Dear Jane,
I was so sorry to hear about your mother’s illness, and will be glad that you are going to Scotland to see how
she has been. It will be nice for her to see you as she hasn’t seen you for a long time. Of course, I will look
after Tim when you have gone there. We thoroughly enjoy having him last year and my dog missed him when
he was leaving and looking for him everywhere. I’m sure he will delight to see him again. Would you bring
him on Tuesday afternoon?. Or, if that didn’t suit, any time on Wednesday. Don’t bother bringing dog food; I
have had plenty. I hope you will have time to have tea with me when you bring Tim and that by then you had
better news of your mother.
I’m looking forward to hearing from you soon,
Love, Sue.
1. 2. 3. 4. 5.
6. 7. 8. 9. 10.
II. These sentences below may be true or wrong. Please tick X if it’s Right; If it’s incorrect, please underline
and tick X in column Wrong and correct it in column Correction. (l,25pts)
SENTENCE RIGHT WRONG CORRECTION
1. There are hundreds of people waiting at the platform when
the train arrived.
2. Before we left, Penny went to pay the hotel room.
3. Can you give me a paper to write Maggie’s telephone on?
4. Would you like Mark to take a picture of you?
5. Satoko is going to the hairdresser’s this afternoon to cut his
hair.
6. I talked and laughed with the passenger sitting next to me
when the train set off.
7. Because Uzi is having problem in class, his parents are
going to school next week to talk to the teacher.
8. Richard brought his guitar on the picnic, so we sang few
songs and had a really good time.
9. Can I introduce you to my friend Jane. She is coming from
London.
10.You speak English very well, but you are too careless with
your written work.

III. Put the correct form of the word in brackets (1,25 pts)
When did the first toys come into (EXIST)______and what led to their development.? Did they represent

138
an attempt by adults to make children happy, or did they arise from the various playful (ACTIVE) ______ of
children themselves? As everyone knows, the young (FREQUENT) ______copy the (BEHAVE) _____
of then elders, and, in their play, they often adopt objects used by adults for entirely different purposes. These
objects (COURAGE) ______the child’s (IMAGINE) ______and lead to games in which everyday articles
often play unusual and (EXPECT) ______roles.
It is rather surprising that for an (EXPLAIN) ______of the origin of toys, we cannot turn to folk stories.
However, no traditional tale (RELATE) ______to the origin of toys exists, and so our (KNOW) ______is
restricted to archaeological study and limited evidence from documents.
IV. Fill in each blank with a suitable preposition (1,25 pts)
1. Don’t write your pencil ________pencil.
2. She got nine marks________ten for her English.
3. He fell________his bicycle.
4. There are many bridges________the Thames.
5. I bought this hat________ten shillings.
6. The ball went________the window.
7. He walked________the door, but fell________the chair on the way out of the room.
8. When you have finished________that book, put it back________its place.
V. Rewrite the sentence so that its meaning is the same with previous one. (1,25 pts)
1. The workers were tired but they kept working.
→ In spite of________________________________________________________.
2. This is the sad news that you have brought.
→ What____________________________________________________________
3. People say he can speak many languages.
→ He is____________________________________________________________
4. Hand your tests to me when you have finished.
→ He asked his students_______________________________________________
5. I don’t have enough money so I can’t buy that bike.
→ If_______________________________________________________________.
6. A train leaves at eight o’clock every morning.
→ There is__________________________________________________________
7. Because she was not sure of the way, she asked the policeman.
→ Not_____________________________________________________________
8. This is the last time I go to his house.
→ I certainly________________________________________________________
9. ”I want you to finish the report by the end of this week.”, the director told her.
→ The director wanted________________________________________________
10. Someone said that she had done her work well.
→ Her work________________________________________________________
VI. Use the given word to make the sentences. (1,25 pts)
Dear Mrs Baker,
1. I/write/ thank/ you/ enjoyable two weeks/I / spend/ your conversation class/ last year.
__________________________________________________________________
2. I/ already/ be/ England/ six weeks/ when/ I/ join/your/ class.
__________________________________________________________________

139
3. But/ this/ be/ my first opportunity/ speak English properly.
__________________________________________________________________
4. My country/ students/ not be really taught/ how/ speak/ listen/
__________________________________________________________________
5. Instead/ teachers/ concentrate/ grammar/ translation/which/ make/ difficult/ students/ communicate.
__________________________________________________________________
6. Now/I/ look/ job/ which/I/ can use English.
__________________________________________________________________
7. I/ hope/ something suitable/ find/ next week/ two.
__________________________________________________________________
8. If/I/ be/ successful/I/stay/Britain/until Christmas.
__________________________________________________________________
9. Thank you/ once again/give me/confidence/ speak English/ without/ worry/ my mistakes.
__________________________________________________________________
10.I / look forward/ see/ you/ again.
__________________________________________________________________
VII. Fill in the blank with suitable words. (2,5 pts)
Katherine Mansfield was born in Wellington, New Zealand, (1) ________when she was (2) ________
school age, her parents (3) ________-to London to. (4) ________English school. She had a (3) ______for
music and for writing and began writing literary (6) ________before she was 18. After finishing school, she
(7)to New Zealand and started writing short stories (8) ________a newspaper in Melbourne (9) _______ to
publish. This was (10) ________1907.
In 1911 her (11) ________-book of short stories was published and (12) ________the attention of (13) ___
cities and readers. During the years of 1912-1914 Katherine Mansfield wrote stories (14) ________two
London magazines. In most of these stories the (15) ________used New Zealand material. At the end of 1917
Katherine Mansfield fell ill (16) ________tuberculosis (17) ________ continued to work. At the beginning of
20s her best (18) ________-of stories (19)very often in her short stories we can see the cruelty of the rich and
(20) ________poor.
-THE END-

140
TRƯỜNG THPT CHUYÊN ĐỀ KIỂM TRA KIẾN THỨC
NGUYỄN HUỆ VÀO LỚP 10 THPT CHUYÊN
Môn thi: TIẾNG ANH (ĐIỀU KIỆN)
ĐỀ THI CHÍNH THỨC Ngày thi: 13 tháng 3 năm 2016
Thời gian làm bài: 120 phút
(Đề thi gồm 06 trang)

- Thí sinh điền đáp án vào khung kẻ trống trong đề.


- Thí sinh không được sử dụng bất kỳ loại tài liệu nào kể cả từ điển và điện thoại.
- Giám thị không giải thích gì thêm.
ĐIỂM GIÁM KHẢO 1 GIÁM KHẢO 2
Họ & tên: (Thống nhất) (Điểm/họ tên/chữ ký) (Điểm/họ tên/chữ ký)
SBD:
Phòng thi số:
Học sinh trường THCS:

PART ONE: PHONETICS – GRAMMAR & VOCABULARY (55 pts)


I. Questions 1-5: Choose the word whose underlined part is pronounced differently from that of the
others by circling A, B, C or D and write your answers into the box below. (5.0 pts)
1. A. there B. appear C. wear D. prepare
2. A. climate B. comprise C. notice D. divide
3. A. kicked B. fixed C. pleased D. missed
4. A. ring B. benefit C. wander D. wonderful
5. A. slavery B. last C. guest D. towards
II. Questions 6-25: Circle the best option (A, B, C or D) to complete each of the following sentences and
write your answers into the box below. (20 pts)
6. That famous book _______ of five chapters.
A. comprises B. makes C. has D. consists
7. He used to _______ his living by delivering vegetables to city hotels.
A. earn B. gain C. get D. make
8. The shop promised to keep the goods for me if I _______ a deposit.
A. made B. paid C. gave D. left
9. We’ll play tennis and _______ we’ll have lunch.
A. so B. so that C. then D. after
10. Nobody was injured in the accident, _______?
A. was he B. wasn’t he C. were they D. weren’t they
11. I wouldn’t like to have _______ a snake or a spider as a pet.
A. neither B. both C. nor D. either
12. _______ being ill, she still went to class yesterday.
A. Even though B. Despite C. Because D. Since
13. On the _______ to the town, there is a beautiful wood.

141
A. way B. direction C. street D. entrance
14. We should try to study hard to _______ our knowledge.
A. better B. farther C. lessen D. strengthen
15. I didn’t _______ her as she was wearing a blond wig.
A. perceive B. know C. realize D. recognize
16. Women’s Aid is a UK charity _______ aim is to end domestic violence against women and children.
A. who B. whose C. which D. whom
17. Passover is also an _______ spring festival.
A. ancient B. anxious C. annual D. official
18. The two friends always back _______ up in everything they do.
A. each other B. one another C. themselves D. ourselves
19. Dinosaurs are believed to _______ millions of years ago.
A. have gone out B. have died out C. have run off D. died out
20. –“I’m taking my first exam next week.” –“______.”
A. Cheers B. Good luck C. Well done D. Congratulations
21. – “Something is worrying me?” – “_______”
A. Well, what are you doing? B. Well, tell me who’s that?
C. Well, tell me what’s worrying you? D. Well, I know.
22. Wait _______ you get home before you unpack your parcel.
A. during B. until C. when D. after
23. The hotel has been built on the _______ of the lake.
A. border B. boundary C. edge D. front
24. _______ does it take you to do the washing?
A. How fast B. What time C. How often D. How long
25. Due to bad weather, the picnic has been _______ until next Sunday.
A. put off B. taken off C. put down D. switched off
III. Questions 26-35: Use the correct form of the words in brackets to complete sentences and write your
answers into the box below. (10 pts)
26. Recently, health foods have increased in _______. (POPULAR)
27. She presented her ideas clearly and _______. (LOGIC)
28. Nowadays _______ knowledge of English can be a problem in business. (SUFFICIENCY)
29. Sorry about the mistakes, I _______ the instruction you gave me. (UNDERSTAND)
30. He is interested in the _______ of old building. (PRESERVE)
31. Housework has _______ been regarded as women’s work. (TRADITION)
32. The whole country is trying to get rid of _______. (POOR)
33. We will live happier and _______ life if we keep our environment clean. (HEALTH)
34. It is _______ of you to cheat in the exam. (HONEST)
35. _______ is now a serious problem in many countries. (FOREST)

IV. Questions 36-45: Put the verbs in brackets into the correct form or tense and write your answers into the
box below. (10 pts)
- He (36. wear) _______ the same old coat since he (37. move) _______ here.
- A lot of people (38. kill) _______ by AIDS recently, and I wish nobody (39. die) _______ any more.
- After (40. stop) _______ by the police, the man admitted (41. steal) _______ the car but denied driving at
100 miles an hour.

142
- We (42. have) _______ our house (43. repair) _______ next month.
- My sister is going to graduate from Hanoi University in July. By then, I (44. complete) _______ my entrance
examination.
- His doctor advised (45. cut) _______ down on the amount of fat food.
V. Questions 46-55: Fill each gap with a suitable preposition to complete each of the sentences. (10 pts)
46. Wearing uniforms helps poor students feel equal _______ others.
47. He made a good impression _______ his first day at work.
48. Egypt is famous _______ ancient Pyramids.
49. Where is Anna? She should be here _______ now.
50. There are usually a lot of parties _______ New Year’s Eve.
51. They revealed that he had supplied terrorist organization _____ weapons.
52. The certificates can be exchanged _______ goods in any of our stores.
53. We plan to impose limitations _______ the use of cars in the city.
54. Their stated aim was to free women _______ domestic slavery.
55. I think they’ve gone to the airport to see their boss _______.
PART TWO: READING (25 pts)
I. Questions 56-65: Read the passage carefully, then fill in each blank with ONE suitable word and write
your answers into the box below. (10 pts)
People appear to learn in different ways. Some people expect to make mistakes in their studies and are
capable (56) _______ benefiting from their mistakes. They don’t mind (57) _______ corrected by their teacher
and indeed often ask to be corrected.
Others, (58) _______, dislike making mistakes. They try to avoid doing anything (59) _______ they might
do badly. They would (60) _______ perfect something in small steps and be sure they have got it right than
attempt to do task (61) _______ on a subject they don’t feel they have finished exploring yet.
Both (62) _______ of learning seem to be equally valid, but a combination of the two may be the (63)__
solution. In order to learn effectively, students have to remember to take risk sometimes. But they also have to
feel comfortable and secure with (64) _____ they’re doing so as not to become demotivated. All students
should at (65) _______ think about questioning the way that they approach learning.
II. Questions 66-75: Read the text below and decide which answer A, B, C or D best fits each numbered
blank and write your answers into the box below. (10 pts)
Environmental Concerns
Earth is the only (66) _______ we know of in the universe that can support human life. (67) _______ human
activities are making the planet less fit to live on. As the western world (68) _______ on consuming two-thirds
of the world’s resources while half of the world's population do so (69) _______ to stay alive, we are rapidly
destroying the very resource we have by which all people can survive and prosper. Everywhere fertile soil is
(70) _______ built on or washed into the sea. Renewable resources are exploited so much that they will never
be able to recover completely. We discharge pollutants into the atmosphere without any thought of the
consequences. As a (71) _______ the planet’s ability to support people is being reduced at the very time when
rising human numbers and consumption are (72) _______ increasingly heavy demands on it. The Earth's
natural resources are there for us to use. We need food, water, air, energy, medicines, warmth, shelter and
minerals to (73) _______ us fed, comfortable, healthy and active. If we are sensible in how we use the
resources they will (74) _______ indefinitely. But if we use them wastefully and excessively they will soon
run (75) _______ and everyone will suffer.
66. A. situation B. place C. position D. site

143
67. A. Although B. Still C. Yet D. Despite
68. A. continues B. repeats C. carries D. follows
69. A. already B. just C. for D. entirely
70. A. sooner B. neither C. either D. rather
71. A. development B. result C. reaction D. product
72. A. doing B. having C. taking D. making
73. A. hold B. maintain C. stay D. keep
74. A. last B. stand C. go D. remain
75. A. out B. off C. over D. down
III. Questions 76-80: Read the following passage and choose the correct answer (A, B, C or D). Write your
answers into the box below. (5pts)
Each nation has many good people who take care of others. For example, some high school and college
students in the United States often spend many hours as volunteers in hospitals, orphanages or homes for the
aged. They read books to the people in these places, or they just visit them, play games with them or listen to
their problems.
Other young volunteers work in the homes of people who are sick or old. They clean up their houses, do
their shopping or mow their lawns. For boys who no longer have fathers, there is an organization called Big
Brothers. College students and other men take these boys to baseball games or on fishing trips and help them
to get to know things that boys usually learn from their fathers.
Some high school students take part in helping disadvantaged or handicapped children. They give care and
comfort to them and help them to overcome their difficulties. Young college and university students participate
in helping the people who have suffered badly in wars or natural disasters. During summer vacations, they
volunteer to work in remote mountainous areas to provide education for children.
Each city has a number of clubs where boys and girls can go to play games or learn craft. Some of these
clubs show movies or organize short trips to interest. Most of these clubs use a lot of high school and college
students as volunteers because they are young enough to remember the problems of younger boys and girls.
Volunteers believe that some of the happier people in the world are those who help to bring happiness to
others.
76. Volunteers usually help those who are sick or old in their homes by _______.
A. moving the lawns, doing shopping and cleaning up their houses
B. telling them stories, and singing and dancing for them
C. cooking, sewing or washing their clothes
D. taking them to the baseball games
77. Big Brother is _______.
A. the name of a club B. a home for children who have no brothers
C. the name of a team D. an organization for boys who no longer have fathers
78. Most of the boys’ and girls’ club use many high school and college students as volunteers because_____.
A. they have a lot of free time
B. they can understand the problems of younger boys and girls
C. they know how to the work
D. they are good at playing games and learning crafts
79. Volunteers believe that _______.
A. in order to make others happy, they have got to be happy
B. the happiest people in the world are those who make themselves happy

144
C. the happiest people in the world are those who are young and healthy
D. bringing happiness to others makes them the happiest people in the world
80. What is the best title for the passage?
A. Taking care of others
B. Volunteers: the happiest people in the world
C. Voluntary work in the United States
D. Helping old and sick people in the United States
PART THREE: WRITING (20 pts)
I. Questions 81-85: Choose the correct sentence (A, B, C or D) that is made from the words given. Write
your answers into the box below. (5pts)
81. very / few / airports / United States / modern / as that / Atlanta.
A. Very few airports in the United States are as modern as that of Atlanta.
B. Very a few airports in the United States are more modern as that Atlanta.
C. Very few of airports in the United States are moderner as that in Atlanta.
D. Very a few of the airports in the United States are as modern as that of Atlanta.
82. both / term paper / final exam / often require / college class.
A. Both a term paper as well as a final exam have often required for a college class.
B. Both a term paper and a final exam are often required for a college class.
C. Both term paper and final exam as well is often required for a college class.
D. Both a term paper and also a final exam has often required for a college class.
83. people / competitive / more likely / suffer / effects / stress / health.
A. People are competitive and more likely suffering by the effects of stress for their health.
B. People, competitive and more likely, suffer from the effects of stress on their health.
C. People are competitive and more likely to be suffered from the effects of stress for their health.
D. People who are competitive are more likely to suffer from the effects of stress on their health.
84. color / star / depend / heat / energy / produce.
A. The color of a star depends on the heat and the energy it produces.
B. The color from a star is depended on the heat and the energy to produce.
C. The color of a star that depends on the heat and the energy produces.
D. The color star depends on the heat and the energy it produces.
85. those / us / smoke / have lungs / x-ray / regularly.
A. Those of us who smoke should have our lungs x-rayed regularly.
B. Those of us smoke and have the lungs x-ray regularly.
C. Those of us who smoke and have the lungs x-rayed regularly.
D. Those of us smoke and so have our lungs x-ray regularly.
II. Questions 86-90: Choose the underlined part (A, B, C or D) that needs correcting and CORRECT IT. (5
pts)
86. (A) Asking about the date of the election, the prime minister (B) commented that (C) no decision (D) had
yet been made.
87. Why (A) do you think (B) you are allowed (C) to criticize me (D) as that?
88. Buying clothes (A) are often (B) a very time-consuming practice because those clothes that a person likes
(C) are rarely the ones that fit (D) him or her.
89. The (A) fish and chips that my mother (B) is cooking in the kitchen (C) taste (D) delicious.
90. Many theories on (A) conserving the purity of water (B) has been proposed but no one (C) has been as

145
(D) widely accepted as this one.
III. Questions 91-95: Rewrite each of the following sentences in such a way that it means the same as the
sentence printed before it. Use the bold words given in brackets. Do not alter the given words in any way. (5
pts)
91. It took us three hours to find a room for the night. (SPENT)
→ We ______________________________________________________________________________
92. Would you rather I stayed with you during the holidays? (PREFER)
→ Would you ________________________________________________________________________
93. The English test was not easy enough for me to do well. (SO)
→ The English test ___________________________________________________________________
94. The police said Jim had stolen the money. (ACCUSED)
→ The police ________________________________________________________________________
95. Because of his illness he could not work effectively. (IMPOSSIBLE)
→ His illness ________________________________________________________________________
IV. Questions 96-100: Rewrite each of the following sentences in such a way that it means the same as the
sentence printed before it. Use the bold words given in brackets. Do not alter the given words in any way. (5
pts)
96. New Year comes near. People are busy.
→ The nearer _______________________________________________________________________
97. Both Peter and John don’t like tea.
→ Neither __________________________________________________________________________
98. Mary will only come to the party if Peter is invited.
→ Unless ___________________________________________________________________________
99. In spite of the bad weather, we went swimming.
→ Although _________________________________________________________________________
100. He is very sorry he didn’t come to the meeting yesterday.
→ He apologies ______________________________________________________________________

146
TRƯỜNG THPT CHUYÊN NGUYỄN HUỆ ĐỀ KIỂM TRA KIẾN THỨC
ĐỀ THI CHÍNH THỨC VÀO LỚP 10 THPT CHUYÊN
Môn thi: TIẾNG ANH (ĐIỀU KIỆN)
Ngày thi: ___ tháng 4 năm 2016
Thời gian làm bài: 120 phút
(Đề thi gồm 06 trang)

- Thí sinh điền đáp án vào khung kẻ trống trong đề.


- Thí sinh không được sử dụng bất kỳ loại tài liệu nào kể cả từ điển và điện thoại.
- Giám thị không giải thích gì thêm.
Họ & tên: ĐIỂM GIÁM KHẢO 1 GIÁM KHẢO 2
SBD: (Thống nhất) (Điểm/họ tên/chữ ký) (Điểm/họ tên/chữ ký)
Phòng thi số:
Học sinh trường THCS:

PART ONE: PHONETICS - GRAMMAR & VOCABULARY (55 pts)


I. Questions 1-5: Choose the word whose underlined part is pronounced differently from that of the others
by circling A, B, C or D and write your answers into the box below. (5.0 pts)
1. A. drive B. drink C. sky D. ride
2. A. laughed B. decided C. started D. invented
3. A. crash B. handbag C. dangerous D. sandwich
4. A. born B. robber C. politic D. orange
5. A. hour B. honorable C. honest D. history
II. Questions 6-25: Circle the best option (A, B, C or D) to complete each of the following sentences and
write your answers into the box below. (20 pts)
6. All the boys are good at cooking, but ______ is as good as the girls.
A. either B. neither C. every D. none
7. Instead of ______ about the good news, Peter seemed to be indifferent.
A. exciting B. being excited C. to excite D. to be excited
8. She nearly lost her own life ______ attempting to save the child from drowning.
A. at B. with C. in D. for
9. She was happy ______ a few of her friends sent her birthday cards.
A. although B. because C. while D. so
10. Daisy wrote ______ report on the Vietnam war.
A. ten - page B. a ten - page C. ten pages D. ten - pages
11. He does not know how to ______ the central heating so the room is cold.
A. stop B. shut C. operate D. hold
12. The ______ man kept on looking at his watch while he was waiting for the bus to arrive.
A. impatient B. smart C. lonely D. quiet
13. Some journalists were waiting for the ______ singer.
A. sensible B. rich C. powerful D. famous
14. Smoking is an ______ habit.
A. illegal B. ordinary C. unhealthy D. admirable

147
15. What a ______!.
A. new small nice house B. nice small new house
C. new nice house, that is small D. new nice and small house
16. Don’t forget to give Mr. Brown my message, _______?
A. do you B. are you C. shall you D. will you
17. He spends hours ______ the Internet every day.
A. surfing B. visiting C. clicking D. downloading
18. Horse-riding is ______ expensive ______ cycling.
A. more - than B. most - than C. as - than D. more - as
19. My teacher can write a beautiful poem in ______.
A. little than half an hour B. a little than half an hour
C. less than half an hour D. least than half an hour
20. Can you explain the difference ______ these two words?
A. from B. of C. to D. between
21. I regret ______ you the story. I really didn’t know it would make you disappointed.
A. tell B. telling C. to tell D. told
22. I was made ______ hard when I was at high school.
A. studying B. studied C. to study D. study
23. We got on the plan and waited about ten minutes before it ______.
A. take off B. land C. took off D. landed
24. The ______ in some countries get allowance from the government.
A. unemployed B. unemployment C. unemploying D. unemploy
25. By the time we ______ there, I’m afraid they ______.
A. will get – will have left B. get – have left
C. will get – have left D. get – will have left
III. Questions 26-35: Use the correct form of the words in brackets to complete sentences and write your
answers into the box below. (10 pts)
26. She has made a _______ decision to send her son the countryside with her parents. (SURPRISE)
27. He didn’t give any _______ for his absence. (EXPLAIN)
28. The jacket didn’t fit me very _______. (GOOD)
29. I wish you every _______ in your new job. (SUCCEED)
30. You look more _______ in your new dress. (BEAUTY)
31. We found the new fashion very _______. (COLOUR)
32. Few people can _______ without water for more than a week. (EXISTENCE)
33. Tourism has _______ agriculture as the nation’s main industry. (REPLACEMENT)
34._______, the best thing to do is to call them before you go. (PROBABLE)
35. People like love stories with happy _______. (END)
IV. Questions 36-45: Put the verbs in brackets into the correct form or tense and write your answers into the
box below. (10 pts)
- It’s time we (36-turn) _______ on the central heating. It (37- get) _______ colder every day.
- I hate (38 - read) _______ about sport, but I don’t mind (39 - learn) _______ about music. What about (40 -
have) _______ a music page
- The computer is broken. It will have to (41-repair) _______.
- If she (42 - not have to) _______ walk home in the rain, she (43-not catch) _______ a terrible cold last

148
Friday.
- The first Olympic Games (44 - take) _______ place at Olympic in Ancient Greece, nearly three thousand
years ago. They (45-hold) _______ every hour years.
V. Questions 46-55: Fill each gap with a suitable preposition to complete each of the sentences. (10 pts)
46. Did the children account ________ 45% of the total population affected the tsunami.
47. You remind me of your uncle. We used to work ________ each other.
48. Take the bus, and get ________ in front of the library
49. We’re always proud ________ your success.
50. Shall we go for a picnic tomorrow? It depends ________ the weather.
51.________ heat from the sun, there would be no life on earth.
52. My mother is very keen ________ growing flowers.
53. We must wait ________ Friday for the final exam results.
54. What time did you arrive ________ the party?
55. It was careless ________ him not to lock the gate last night.
PART TWO: READING (25 pts)
I. Questions 56-65: Choose the words in the box to fill in each blank in the following passage. (10 pts)
but like grow like from enough heat it in into
Have you ever seen a greenhouse? Most greenhouses look (56) _______ a small glass house. Greenhouses
are used to (57) _______ plants, especially in the winter. Greenhouses work by trapping (58) _______ from
the sun. The glass panels of the greenhouse let in light but keep heat (59) _______ escaping. This causes the
greenhouse to heat up, much like the inside of a car parked (60)_______ sunlight, and keeps the plants warm
(61)_______ to live in the winter.
The Earth’s atmosphere is all around us. It is the air that we breathe. Greenhouse gases in the atmosphere
behave much (62)_______ the glass panels in a greenhouse. Sunlight enters the Earth’s atmosphere, passing
through the blanket of greenhouse gases. As (63)_______ reaches the Earth’s surface, land, water, and
biosphere absorb the sunlight’s energy. Once absorbed, this energy is sent back (64)_______ the atmosphere.
Some of the energy passes back into space, (65)_______ much of it remains trapped in the atmosphere by the
greenhouse gases, causing our world to heat up.
II. Questions 66-75: Read the text below and decide which answer A, B, C or D best fits each numbered
blank and write your answers into the box below. (10 pts)
English has for more than a century and (66)_______ half (67)_______ called a world language. The
number of people who speak it (68)_______ their mother tongue has been estimated at between three hundred
million and four hundred million. It is recognized as an (69)_______ language in countries (70)_______ 1.5
billion people live. In China, the importance attached to (71)_______ English is such that a televised teaching
course drew audiences of up to one million. But this spread of English thorough the world is relatively recent.
In the (72)_______sixteenth century English was spoken by just under five million people. The (73)_______of
English in North America was the (74)_______ step in its worldwide expansion. The United States is a huge
commercial market and his has tended to promote the English language in many other nations. About eighty
percent of the data stored on the world’s computer (75)_______ believed to be in English and nowadays
insufficient knowledge of English can be a problem in business.
66. A. a B. more C. about D. none
67. A. be B. been C. being D. was
68. A. as B. like C. with D. such as
69. A. office B. offices C. official D. officious

149
70. A. where B. when C. what D. which
71. A. learn B. learning C. be learn D. being learn
72. A. late B. latest C. lately D. later
73. A. arrive B. arrived C. arrival D. arriving
74. A. lock B. chain C. rope D. key
75. A. has B. were C. are D. is
III. Questions 76-80: Read the following passage and choose the correct answer (A, B, C or D). Write your
answers into the box below. (5pts)
Many people now think that teachers give pupils too much homework. They say that it is unnecessary for
children to work at home in their free time. Moreover, they argue that most teachers do not properly plan the
homework tasks they give to pupils. The result is that pupils have to repeat tasks, which they have already
done at school.
Recently many parents complained about the difficult homework which teachers gave to their children. The
Greek parents said that most of the homework was a waste of time, and they wanted to stop it. Spain and
Turkey are two countries, which stopped homework recently. In Denmark, West Germany and several other
countries in Europe, teachers cannot set homework at weekends. In Holland, teachers allow pupils to stay at
school to do their homework. The children are free to help one another. Similar arrangements also exist in
some British schools.
Most people agree that homework is unfair. A pupil who can do his homework in a quiet and comfortable
room is in a much better position than a pupil who does his homework in a small, noisy room with the
television on. Some parents help their children with their homework. Other parents take no interest at all in
their children’s homework.
It is important, however, that teachers talk to parents about homework. A teacher suggests suitable tasks for
parents to do with their children. Parents are often better at teaching their own children.
76. According to the writer, many parents would like their children ___________.
A. to do more difficult homework
B. to do homework both at school and at home
C. to have test homework
D. to do homework at school only
77. According to many parents, ___________.
A. a lot of homework has not been planned properly
B. children are too lazy to do
C. teachers do not set enough homework.
D. children shouldn’t be given marks for homework
78. Greek parents thought ___________.
A. more time should be allowed for homework
B. their children’s homework was useful
C. their children’s homework was useless
D. their children’s homework was too easy
79. In some countries, especially in Europe ___________.
A. teachers cannot have their children do homework at weekends
B. few people think homework to children only at weekends
C. most people agree that it is fair to have children do homework at home
D. teachers are allowed to give children homework only at weekends

150
80. Pick out the statement that is not true ___________.
A. Teachers should advise the parents about how to work together with their children at home
B. Only a small number of people think homework is fair.
C. All parents show great interest in their children’s homework
D. Children can do their homework at school and help each other in some schools in Britain.
PART THREE: WRITING (20 pts)
I. Questions 81-85:Choose A, B, C or D that best fits the meaning of the given sentence. (5pts)
81. It’s really hard to believe ____________.
A. he would say such a terrible thing
B. such a terrible thing he would say
C. to say he would such a terrible thing
D. would he say such a terrible thing
82. Why don’t you _______________ us?
A. go to the house party with
B. go together the house party with
C. go the house party with
D. together the house party with
83. The new teacher loves poems and makes us ___________.
A. learn them heartily
B. heartily learn them
C. learn them by heart
D. learning by heart them
84. I’ll give you another hour ___________.
A. making your mind up
B. making up your mind
C. make up your mind
D. to make up your mind
85. They are rich enough to own ___________.
A. not a helicopter only, but a yacht as well
B. not only a helicopter, but a yacht as well
C. only a helicopter, but a yacht as well
D. as well as a helicopter, not only a yacht
II. Questions 86-90: Choose the underlined part (A, B, C or D) that needs correcting. (5 pts)
86. Roberto always likes (A) go (B) running (C) in the (D) summer.
87. How (A) is the weather like (B) in winter (C) in your (D) country?
88. On (A) Sunday I just stay at (B) home because (C) I’m usually very tiring (D).
89. The plane delayed (A)for more than (B) one hour because of (C) the bad weather (D).
90. They are known (A) that colds can be avoided (B)by eating the right (C)kind of food and taking exercise
regularly.
III. Questions 91-95: Rewrite each of the following sentences in such a way that it means the same as the
sentence printed before it. Use the bold words given in brackets. Do not alter the given words in any way. (5
pts)

151
91. The demand for tickets was so great that people queued day and night. (SUCH)
→ It was_______________________________________________________________
92. I haven’t seen her for two years (LAST)
→ The_________________________________________________________________
93. Now that his mother was being there, they said nothing about it. (ACCOUNT)
→ On__________________________________________________________________
94. Thanks to his aunt’s legacy of $10.000, he was able to buy the house he wanted. (NOT)
→ Had his_______________________________________________________________
95. The house seemed to have been unoccupied for several months. (LOOKED)
→ It____________________________________________________________________
IV. Questions 96-100: Rewrite each of the following sentences in such a way that it means the same as the
sentence printed before it. Use the bold words given in brackets. Do not alter the given words in any way. (5
pts)
96. Although Judy was severely disabled, she participated in many sports.
→ Despite________________________________________________________________
97. I’m sorry I was rude to you yesterday.
→ I apologize_____________________________________________________________
98. Nobody can deny that she has a beautiful voice.
→ It____________________________________________________________________
99. She liked Paris very little, and Rome less.
→ She thought Rome_______________________________________________________
100. Turn off all switches before leaving the workshop.
→All the switches__________________________________________________________

152
TRƯỜNG THPT CHUYÊN NGUYỄN HUỆ ĐỀ KIỂM TRA KIẾN THỨC 3
ĐỀ THI CHÍNH THỨC VÀO LỚP 10 THPT CHUYÊN
Môn thi: TIẾNG ANH (CHUYÊN)
Ngày thi: 10 tháng 4 năm 2016
Thời gian làm bài: 120 phút
(Đề thi gồm 06 trang)
- Thí sinh điền đáp án vào khung kẻ trống trong đề.
- Thí sinh không được sử dụng bất kỳ loại tài liệu nào kể cả từ điển và điện thoại.
- Giám thị không giải thích gì thêm.
Họ & tên: ĐIỂM GIÁM KHẢO 1 GIÁM KHẢO 2
SBD: (Thống nhất) (Điểm/họ tên/chữ ký) (Điểm/họ tên/chữ ký)
Phòng thi số:
Học sinh trường THCS:

PART I: LISTENING (15 points)


Listen to a conversation between Mrs. Phillips and Robert about library work, answer the questions.
Questions 1-6: Answer the following questions. Write NO MORE THAN THREE WORDS AND/OR A
NUMBER for each answer. Write your answers into the box below.
1. What is Mrs. Phillip‟s job at the library?
2. What is Robert‟s job at the library?
3. What time does the library open?
4. What time does Robert can go home after he finishes his duty in the library?
5. Where are fiction and non-fiction books collected?
6. Where can children find their suitable books?
Questions 7-11: Decide whether these statements are True (T) or False (F). Write your answers in the box
below.
7. The books in the brown cart are for republishing.
8. The books in the black cart need to be repaired.
9. The white cart books are to be sold as used books to raise money for the laboratory.
10. “Fashion show” is the most popular activity of the library.
11. It costs children nothing to see movies on Saturday noon.
Questions 12-15: Complete the table. Write NO MORE THAN TWO WORD AND/OR A NUMBER for each answer.
Write your answers into the box below.
Activity Location Day and Time
(12 ) ____________________ Children‟s Room Thursday at 11:00
Family Movies (13 ) ____________________ ( 14) ____________________
(15 ) ____________________ Meeting Room Friday at 6:30
PART II: PRONUNCIATION, VOCABULARY & GRAMMAR (35 points)
Questions 16-17: Choose the word (A, B, C or D) whose underlined part is pronounced differently from that
of the others. Write your answers into the box below.
16. A. hasty B. nasty C. tasty D. wastage

153
17. A. rough B. thought C. thorough D. through
Questions 18-20: Choose the word (A, B, C or D) that differs from the others in the position of the primary
stress. Write your answers into the box below.
18. A. argumatative B. psychological C. contributory D. hypersensitive
19. A. compensate B. competence C. communist D. commitment
20. A. mysterious B. preferable C. modernity D. historical
Questions 21-30: Choose the best option (A, B, C or D) to complete each of the following sentences.
Write your answers into the box below.
21. Although he was completely _______ as a furniture maker, he produced the most beacutiful chairs.
A. unable B. untrained C. incapable D. uneducated
22. There is no point in telephoning him. He’s certain _______ by now.
A. to leave B. to have left C. left D. having left
23. _______, dolphins have no sense of smell.
A. As known as far B. As far as is known
C. It is known as far D. Known as far as it is
24. When I finish writing this composition, I’m going to _______ and go to bed.
A. make time B. hit the day C. hit the big time D. call it a day
25. It could have been a lot worse _______ there.
A. when he was not B. whether or not he had been
C. for he had not been D. had he not been
26. _______ kind of organization throughout the world uses computers to conduct business.
A. Virtually every B. Virtually all C. Nearly most D. Nearly all
27. After a lengthy debate, the spokesman announced the board had _______ a unanimous conclusion.
A. committed B. solved C. reached D. compromised
28. Not only _______ in the project, but he also wanted to become the leader.
A. did Jack involve B. had Jack been involved
C. was Jack involved D. was involved Jack
29. The factory is closed today. The workers are _______ strike because their demand _______ increase
_______ pay has not been met.
A. on – for – in B. in – for – in C. for – on – in D. on – in – for
30. _______ snow that covers _______ top of _______ mountain is _______ beautiful sight.
A. The – the – the – the B. The – the – the – a
C. Ø – the – the – the D. Ø – Ø – the – the
Questions 31-35: Find one word that can fill in the blank of the three given sentences. Write your answers
into the box below.
31._______
a. An _______ a day keeps a doctor away.
b. I am the _______ of our parents’ eye.
c. The ________ doesn’t fall far from the tree.
32._______
a. They are looking at the _______ of fish in the lake.
b. My younger brother is studying at a private _______.
c. A _______ bus is very popular nowadays.
33._______

154
a. Could you add this up for me? I’m not good at all with _______.
b. The cocktail party was attended by several well-known public _______.
c. Models have to watch their _______ all the time so they eat very carefully.
34._______
a. Anna _______ in bed looking at the ceiling and wondering whether or not to get up.
b. I'm going to _______ a dustsheet down on the floor and start painting the walls immediately.
c. Will you _______ the table while I get the meal ready?
35._______
a. Why don’t more women _______ for public office?
b. Does your car _______ on unleaded petrol or diesel?
c. We are going to _______ a series of advertisements in the local paper.
Questions 36-45: Use the correct form of the bracketed word to complete the passage. (0) has been done as
an example. Write your answers into the box below.
What kind of __ education ___(0. EDUCATE), experience and background should we be giving our
children? In a _______ (36. COMPETE) western economy, they need to be well-informed and _______ (37.
KNOW). In a complex ever-changing modern world, it is _______ (38. ARGUE) that the task of preparing the
next generation cannot be carried out within the _______ (39. TRADITION) classroom framework, because
acquiring a proper education in this way is simply _______ (40. POSSIBLE). This is where computer-aided
learning steps in, providing children with the skills that they will need in tomorrow’s world, in a one-to-one
learner-center environment.
There are those who claim, however, that _______ (41. EXCEED) exposure to computers may prove to be
harmful rather than _______ (42. BENEFIT) to children and that is more _______ (43. PRODUCE) for
education to take place in the classroom. They say that, although computers seem to hold a _______ (44.
RESIST) appeal for some people, particularly young children, it is _______ (45. PREFER) for children to have
a person, rather than a machine, for a teacher.
Questions 46-50: Replace each underlined verb with the correct form of a synonymous phrasal verb in the box. There
are two extra ones that you do not need to use. Write your answers into the box below.
come into turn in draw up let on
make out take after turn down
46. Before we do anything else, we ought to prepare a plan of action.
47. It was getting late so I decided to go to bed.
48. I believe that Diana has recently inherited a lot of money.
49. I was rejected for the army on the health ground.
50. I resemble my mother. She was small with blond hair and had a terrible memory too.
PART III:READING COMPREHENSION (25 points)
Questions 51-55: You are going to read an extract from a magazine article about psychology. Five
paragraphs have been removed from the extract. Choose from the paragraphs A-F the one which fits each
gap (51-55). There is one extra paragraph which you do not need to use. Write your answers into the box
below.
Unless you’re a loner, you're probably a member of many different groups. Some are long lasting, like your
family, while others, such as a football crowd, are temporary. Every group functions in different ways but there
are still some common psychological features to any group.
(51) __________

155
Being part of a group changes the way you behave. The presence of others has a generally arousing effect on
the nervous system. This is natural-you don’t know what they will do. They may move about, speak to you, or
even attack you, and your brain has to pay attention to all these messages. Having others around is just
distracting. It divides your attention, so you can't focus on the task in hand.
(52) __________
But when it comes to complex tasks, the prospect of not being evaluated may free you from anxiety, so
encouraging you to perform better. To test this, psychologists asked volunteers to carry out a complex
computer task in separate rooms. Some were told performance would be evaluated individually - causing
performance anxiety - while others were told the results would be averaged with the rest of the group. As
expected, the second group did better than the first.
(53) __________
The problem becomes even worse when individuals are disguised with war paint or uniforms. Analysis
shows that the more people there are in a mob, the greater the antisocial behaviour. Being ‘submerged’ within
a group does have many negative connotations but it's not always a bad thing. Instead of thinking about a
rowdy mob, think of relaxing, dancing and enjoying yourself at a party where you are also just a part of the
crowd.
(54) __________
But surprisingly, research has shown that the use of this technique by groups isn't that effective in either the
number or quality of ideas generated. You get better results if you set people to work individually on a
problem. We also tend to assume that decisions made by groups are better than those made by individuals,
which is why we form committees.
(55) __________
But any group can get a decision badly wrong because their thought processes can go awry. This is a process
psychologists call 'groupthink'. A close-knit group of advisers isolated from argument and criticism can grow
to believe they can do no wrong. The group then becomes more important than the individuals who are part of
it. Further, a leadership style which concentrates on the group rather than the individual can contribute to this.
For a group and its behaviour are shaped by who is in charge and the roles the other members play. A good
front man or woman is persuasive, not directive, communicates and speaks clearly, listens well and appeals to
group members’ emotions and feelings as well as thoughts and ideas. Many, but not all, leaders show
dominance in the group. But the dominant person in the group doesn't always make the best leader, and it isn't
necessarily true that having the smartest people always makes for the most successful group.
A. This lack of individual accountability may lead to people letting go of their inhibitions - a process known as
‘deindividuation’. Sometimes this is dangerous, releasing violent and impulsive behaviours that individuals
would never dream of indulging in alone.
B.They have a need for power, characterised by talking a lot, wanting to be listened to and to make decisions.
In a group it can be easy to spot the dominant people. They make eye contact, point and even touch people but
don’t encourage return gestures. Such behaviour studies have been key in shaping business organisation.
C. For example, in general, humans are social animals, that's why we get together in groups in the first place.
Important elements of our individual identity come from being part of a group. Most people enjoy being in a
group - it's a way of forming emotionally satisfying relationships.
D. But it has to be a carefully selected team, not a randomly generated group, if creative decisions are to be
made. In fact, theory on team building has shown that it is better if people work in small teams of
complementary pairs. Big teams don't get anything done, even though people like them.

156
E.So, how does all this stimulation affect achievement? It has been argued that people do better on simple
well-rehearsed activities when they're with others than when they are alone. Also, if their individual efforts
within the group are not being monitored, there’s a tendency to relax and merge into the crowd.
F.Another positive feature of groups is that they generate ideas and opinions, and use these to make decisions.
That’s why the modern trend in teaching is for students to work in small groups to prepare presentations and
why brainstorming is so popular in the work context.
Questions 56-65: Read the following passage and decide which answer A, B, C or D best fits each numbered
blank. Write your answers into the box below.
A LOVE OF TRAVELLING
For Nigel Portman, a love of travelling began with what’s (56) _______ a „gap year‟. In common with
many other British teenagers, he chose to take a year out before (57) _______ to study for his degree. After
doing various jobs to (58) _______ some money, he left home to gain some experience of life in different
cultures, visiting America and Asia. The more adventurous the young person, the (59) _______ the challenge
they are likely to set themselves for the gap year, and for some, like Nigel, it can (60) _______ in a thirst for
adventure.
Now that his university course has (61) _______ to an end, Nigel is just about to leave on a three-year trip
that will take him right around the world. What’s more, he plans to make the whole journey using only means
of transport which are (62) _______ by natural energy. In other words, he’ll be (63) _______
mostly on bicycles and his own legs; and when there’s an ocean to cross, he won’t be taking a (64) _______
cut by climbing aboard a plane, he’ll be joining the crew of a sailing ship (65) _______ .
As well as doing some mountain climbing and other outdoor pursuits along the way, Nigel hopes to passon
to the people he meets the environmental message that lies behind the whole idea.
56. A. called B. named C. referred D. Known
57. A. settling down B. getting up C. taking over D. holding back
58. A. achieve B. raise C. advance D. win
59. A. stronger B. wider C. greater D. deeper
60. A. result B. lead C. cause D. create
61. A. come B. turned C. reached D. brought
62. A. pulled B. charged C. forced D. powered
63. A. attempting B. using C. relying D. trying
64. A. quick B. short C. brief D. swift
65. A. anyway B. alike C. instead D. Otherwise
Questions 66-75: Read the passage carefully, then fill in each blank with ONE suitable word. Write your
answers into the box below.
Students frequently complain (66) _______ studying for hours on (67) _______ and then not doing well in
their final exams. Many factors can (68) _______ in poor performances: (69) _______ illness or a personal
problem. Getting high grades can also put a terrible strain on students (70) _______ don’t want to let their
parents down.
Can students‟ knowledge be judged by a (71) _______ exam? Certainly not! If we want to be fair, students
ought to be (72) _______ on a regular basic. So does that mean more exams? Yes, but (73) _______ many
written ones. In many countries, student do not have to take written exams (74) _______they don’t want to,
they are (75) _______ to take oral ones instead. This seems to be fairer way of assessing understanding, not
just testing it.
PART IV: WRITING (25 poitns)

157
Questions 76-80: Finish each of the following sentences in such a way that it means the same as the
sentence printed before it.
76. We couldn’t relax until all the guests had gone home.
→ Only _____________________________________________________________________________
77. House prices have risen sharply this year.
→ There has ________________________________________________________________________
78. She furiously threw the book across the room.
→ Such _____________________________________________________________________________
79. Experts think that all dogs evolved from wolves.
→ All dogs are _______________________________________________________________________
80. The northwest of Britain has more rain each year than the southeast.
→ The annual _______________________________________________________________________
Questions 81-85: Rewrite each of the following sentences in such a way that it means the same as the
sentence printed before it. Use the bold words given in brackets. Do not alter the given words in any way.
81. The two theories appear to be completely different. (COMMON)
_____________________________________________________________________________________
82. His rude behaviour is too much for me. (PUT)
_____________________________________________________________________________________
83. I find his handwriting very hard to read. (DIFFICULTY)
_____________________________________________________________________________________
84. He doesn’t have money to go on holiday. He doesn’t have time, either. (NEITHER)
_____________________________________________________________________________________
85. You’d feel better if you had a quiet holiday. (DO)
_____________________________________________________________________________________
Questions 86-100: (In about 200 words) Write an essay about the advantages and disadvantages of studying
in groups.
______________________________________________________________________________________
______________________________________________________________________________________
______________________________________________________________________________________
______________________________________________________________________________________
______________________________________________________________________________________
______________________________________________________________________________________
______________________________________________________________________________________

158
TRƯỜNG THPT CHUYÊN NGUYỄN HUỆ ĐỀ KIỂM TRA KIẾN THỨC
ĐỀ THI CHÍNH THỨC VÀO LỚP 10 THPT CHUYÊN
Môn thi: TIẾNG ANH (Điều kiện)
Ngày thi: 08 tháng 05 năm 2016
Thời gian làm bài: 120 phút
(Đề thi gồm: 07 trang)
- Thí sinh điền đáp án vào khung kẻ trống trong đề.
- Thí sinh không được sử dụng bất kỳ loại tài liệu nào kể cả từ điển và điện thoại.
- Giám thị không giải thích gì thêm.
Họ & tên: ĐIỂM GIÁM KHẢO 1 GIÁM KHẢO 2
SBD: (Thống nhất) (Điểm/họ tên/chữ ký) (Điểm/họ tên/chữ ký)
Phòng thi số:
Học sinh trường THCS:

PART I. PHONETICS, GRAMMARAND VOCABULARY (5.5 pts)


I. Circle the word whose underlined part is pronounced differently from others (0.5pt)
1. A. dead B. leave C. creature D. teacher
2. A. succeed B. accept C. account D. accident
3. A. energy B. baggy C. deny D. primary
4. A. birthday B. within C. mouth D. thing
5. A. cause B. clause C. audience D. laugh
II. Circle the best option (A, B, C or D) to complete each of the following sentences. (2.0 pts)
6. They all laughed because the film was very________.
A. amused B. amuse C. amusement D. amusing
7. His father, ________is a university professor, is very generous.
A. who B. that C. which D. whom
8. John bought a new dictionary, but he didn’t tell me how much________
A. did it cost B. it costs C. has it cost D. it cost
9. Here’s some iced tea. - This is too sweet________to guests.
A. to serve B. for serving C. to serve it D. serve
10. In addition________the news, television provides us with variety of programmes that can satisfy taste.
A. to/ every B. to/ each C. for/ every D. for/ all
11. I don’t like to ask people for help as a rule but I wonder if you could________me a favour.
A. make B. pick C. do D. find
12. When you________the intersection of Fifth Road and Broadway, turn right.
A. come B. get C. reach D. arrive
13. I haven’t seen Tom________he got married.
A. before B. until C. since D. although
14. Your grandfather is rather tired so do not________your visit.
A. prolong B. lengthen C. delay D. shorten
15. The doctor told him to keep________sweets and chocolate to lose weight.
A. at B. back C. up D. off
16. They’ll never________to get here by six - the roads are quite busy today.

159
A. manage B. arrive C. succeed D. able
17. The meeting had been cancelled________the chairman’s absence.
A. because B. because of C. despite D. although
18. The teacher told us________an essay and________any mistake.
A. to write/ not to do B. to write/ not to make
C. write/ not to do D. to write/ to make
19. If he________the music so loud, the neighbours wouldn’t have called the police.
A. has played B. didn’t play C. was playing D. hadn’t played
20. She wondered________her only son was then.
A. where B. if C. whether D. what
21. ________schooling is compulsory in Australia between ages of six and seventeen.
A. The/ Ø B. The/ the C. Ø / the D. A/ an
22. It is found that endangered species are often concentrated in areas that are poor and densely populated,
such as much of Asia and Africa.
A. disappeared B. increased C. reduced D. threatened
23. The children don’t like living in the countryside, ________?
A. do the children B. don’t they C. don’t the children D. do they
24. “I suggest fixing the dripping faucet.” – “________”
A. We will B. Good idea C. I’m afraid not D. Yes, please
25. Televisions, dishwashers and refrigerators are electrical________.
A. appliances B. tools C. things D. households
III. Complete the sentences by using the correct form of the words in brackets. (1.0 pt)
26. Students have a________between studying English or Russian. (CHOOSE)
27. He has overcome his difficulties with courage and________. (DETERMINE)
28. I live alone and I don’t have many________. (VISIT)
29. There should be parking spaces for the________. (DISABLE)
30. Robin Hood is a________hero who lived in Sherwood Forest in Nottingham. (LEGEND)
31. He was extremely________of her public image. (PROTECT)
32. All the________in the conference had an opportunity to speak. (PARTICIPATE)
33. What’s your nationality? - I’m a________ (JAPAN)
34. The recent floods cause________damage. (WIDE)
35. The police recovered the________jewelry yesterday. (STEAL)
IV. Use the correct form of the verbs in brackets to complete the sentences. (1.0 pt)
36-37. If I (have) ________wings, I (have to, not) ________take an airplane to fly home.
38-39. Peter (drive) ________towards Victoria Point when he (lose) ________control of his vehicle.
40-41. I (go) ________to Toronto last year for business. I (go) ________there several times since then.
42-43. I didn’t feel like (talk) ________to him after what (happen) ________.
44. It takes a long time (learn) ________a foreign language.
45. The names of the winners (give) ________out on the radio 2 hours ago.
V. Fill each gap with a suitable preposition to complete each of the sentences. (1.0 pt)
46. It was only________accident when I found out who the man really was.
47. Next month I’m going to Scotland________a short holiday.
48. Long skirts are________fashion nowadays.
49. I don’t care________money. It is not important to me.

160
50. I’ll be moving to a new address________the end of September.
51. I didn’t have any money on me, so I paid________cheque.
52. She broke the eggs carefully________the bowl.
53. I don’t feel well, but I don’t know what's the matter________me.
54. Do you know any songs________the Beatles?
55. They have complained________the council about the noise.
PART II - READING (2.5 pts)
I. Read the passage and fill each gap with ONE suitable word. (1.0 pt)
A national park is a large piece of land in (56) ________animals are free to come and go. Trees and (57)__
grow everywhere. People go to a national park to (58) ________nature. Many people stay in campgrounds in
natural parks. They sleep in tents and cook (59) ________food over campfires. They also walk (60) ________
trails or paths in the park.
Yellowstone is the world’s oldest (61) ________park. It became a national park in 1872. It is (62) ________
the world’s largest park. Yellowstone is (63) ________for its geysers. These holes in the ground shoot hot
water into the ah. There are about seventy geysers in the park. The most famous is Old Faithful. About (64)___
hour Old Faithful shoots hot water hundreds of feet into the air.
Two and a half million people visit this beautiful park each year. Visitors are not allowed to pick the
flowers. They (65) ________not feed or hunt the animals, either.
II. Read the text below and circle the option (A, B, C or D) that best fits each gap. (1.0 pt)
HOLIDAYS THAT DON’T COST THE EARTH
The tourist industry is considered to be the world's largest industry. Before 1950 about one million people
(66) ________abroad each year (67) ________by the 1990s the figure had (68) ________to over 400 million
every year.
(69) ________large numbers of tourists, however, are beginning to cause problems. For example, in the
Alps the skiers are destroying the mountains (70) ________came to enjoy. Even parts of Mount Everest in the
Himalayas are reported to be covered (71) ________old tins, tents and food that have been (72) ________
away.
But at a time when we have greater freedom to travel (73) ________ever before, more and more people are
asking how they can enjoy their holiday (74) ________causing . problems by spoiling the countryside. Now
there is a new holiday guide called Holidays That Don’t Cost the Earth. It (75) ________you how you can help
the tourist industry by asking your travel agent or your tour operator the right questions before you go on
holiday.
66. A. traveled B. came C. sailed D. were
67. A. as B. because C. but D. when
68. A. gone B. flown C. risen D. raised
69. A. Such B. More C. Few D. So
70. A. which B. you C. who D. they
71. A. for B. on C. with D. below
72. A. put B. thrown C. given D. tidied
73. A. than B. when C. then D. while
74. A. outside B. instead C. beside D. without
75. A. says B. offers C. tells D. gives
III. Read the text below and circle the correct ’answer A, B, C or D (0.5 pt)

161
Winter driving is dangerous because it’s so difficult to know what is going to happen and accidents take
place so easily. Fog can be waiting to meet you over the top of the hill. Ice might be hiding beneath the melting
snow, waiting to send you off the road. The car coming towards you may suddenly slide across the road.
Rule Number One for driving in icy roads is to drive smoothly. Uneven movements can make a car suddenly
very difficult to control. So every time you turn the wheel, touch the brakes or increase your speed, you must
be as gentle and as slow as possible. Imagine you are driving with a cup of hot coffee on the seat next to you.
Drive so that you wouldn’t spill it.
Rule Number Two is to pay attention to what might happen. The more ice there is, the further down the road
you have to look. Test how long it takes to stop by gently braking. Remember that you may be driving more
quickly than you think. In general, allow double your normal stopping distance when the road is wet, three
times this distance in snow, and even more on ice. Try to stay in control of your car at all times and you will
avoid trouble.
76. What is the writer trying to do in this text?
A. to complain about bad winter driving. B. to give information about winter weather.
C. to warn people against driving in winter. D. to advise people about safe driving in winter.
77. Why would somebody read this text?
A. to find out about the weather. B. to get information on driving lessons.
C. to learn about better driving. D. to decide when to travel.
78. What does the writer think?
A. People should avoid driving in the snow. B. Drivers should expect problems in winter.
C. People drive too fast in winter. D. Winter drivers should use their brakes less.
79. Why does the writer talk about a cup of coffee?
A. to explain the importance of smooth movements.
B. because he thinks refreshments are important for drivers
C. because he wants drivers to be more relaxed.
D. to show how it can be spilt.
80. Which traffic sign shows the main idea of the text?
A. Drive Carefully - Ice On Road Ahead. B. Reduce Speed Now - Fog Ahead.
C. Drive Carefully - Road Repairs Ahead. D. Slow Down - Accident Ahead.
PART III - WRITING (2.0 pts)
I. Circle the correct sentence (A, B, C or D) that is made from the words given. (0.6 pt)
81. My/ father/ teach/ me/ how/ use/ computer/.
A. My father taught me how to use the computer.
B. My father taught me how use the computer.
C. My father taught me how using the computer.
D. My father taught me how used the computer.
82. Mother/ ask/ me/I/ do/ all/ homework/.
A. My mother asked me whether I have done all my homework.
B. My mother asked me whether I have been doing all my homework.
C. My mother asked me whether I do all my homework.
D. My mother asked me whether I had done all my homework.
83. Nowadays/ children/ kept/ in/ day-care centre/ while/ then/ mothers/ are/ working/.
A. Nowadays, children kept in a day-care centre while their mother are working.
B. Nowadays, children are kept in a day-care centre while their mother are working.

162
C. Nowadays, children are kept in a day-care centre while their mother working.
D. Nowadays, children were kept in a day-care centre while their mother are working.
84. Ha Long Bay/ recognized/ eight wonder/ world/.
A. Ha Long Bay recognized the eighth wonder of the world.
B. Ha Long Bay had been recognized the eighth wonder of the world.
C. Ha Long Bay was recognized the eighth wonder of the world.
D. Ha Long Bay is recognized as the eighth wonder of the world.
85. My younger brother/ learnt/ use/ computer/ since/ he/ 8 years old/.
A. My younger brother learnt to use the computer since he was 8 years old.
B. My younger brother had learnt to use the computer since he was 8 years old.
C. My younger brother has learnt to use the computer since he was 8 years old.
D. My younger brother is learnt to use the computer since he was 8 years old.
II. Complete the second sentence so that it has a similar meaning to the first sentence. (0.8 pt)
86. I don’t like you to play the music so loudly at night.
→ I’d rather_____________________________________________________________
87. We can’t sit on the grass because it is too wet.
→ The grass isn’t_________________________________________________________
88. If you work hard, you will find it easy to get good marks.
→ The harder____________________________________________________________
89. Paul said he was going to visit China the following year.
→ “___________________________________________________________________” Paul said.
90. They sent millions of Christmas cards last month.
→ Millions
91. I have a terrible headache because I stayed up late last night.
→ If I_________________________________________________________________
92. “Don’t live in this area because it is very dangerous” they asked me
→ They told____________________________________________________________
93. We were late for the meeting due to the heavy traffic.
→ Because_____________________________________________________________
94. I regret that you didn’t give me a chance to tell you the truth.
→ I wish_______________________________________________________________
95. Nobody had invited Jane to the party, which annoyed her.
→ As Jane______________________________________________________________
III. Complete the second sentence so that it has a similar meaning to the first sentence, using the words in
brackets. Do not change the words given. (0.6 pt)
96. “Try your best study”, my father said to me.(encouraged)
→ _____________________________________________________________________
97. They have very little money. They are happy. (In spite of)
→ _____________________________________________________________________
98. Last year they painted our house. (had)
→ _____________________________________________________________________
99. I haven’t contacted my best friend for several months. (touch)
→ _____________________________________________________________________
100. He objects to people laughing at his appearance. (being)

163
TRƯỜNG THPT CHUYÊN NGUYỄN HUỆ ĐỀ KIỂM TRA KIẾN THỨC
ĐỀ THI CHÍNH THỨC VÀO LỚP 10 THPT CHUYÊN
Môn thi: TIẾNG ANH (CHUYÊN)
Ngày thi: 8 tháng 5 năm 2016
Thời gian làm bài: 120 phút
(Đề thi gồm 06 trang)

- Thí sinh điền đáp án vào khung kẻ trống trong đề.


- Thí sinh không được sử dụng bất kỳ loại tài liệu nào kể cả từ điển và điện thoại.
- Giám thị không giải thích gì thêm.
Họ & tên: ĐIỂM GIÁM KHẢO 1 GIÁM KHẢO 2
SBD: (Thống nhất) (Điểm/họ tên/chữ ký) (Điểm/họ tên/chữ ký)
Phòng thi số:
Học sinh trường THCS:

PART I: LISTENING - (Time for listening: 2’35 x 2 = 5’10) (15pts)


You are going to listen (TWICE) to a man talking about his trips. Listen and complete the tasks. Questions
1-5: Decide whether these statements are True (T) or False (F). Write your answers in the box below.
1. The man often travels for his job as a rug seller.
2. The man is asked to give a talk about his trips in Europe and Africa.
3. The man is willing to answer questions at the end of his talk.
4. It often takes the man at least three months to prepare carefully for his trips.
5. The man only get information about the places he is going to visit on the Internet.
Questions 6-15: Complete the table. Write NO MORE THAN TWO WORD AND/OR A NUMBER for each
answer. Write your answers into the box below.
Countries visited Interesting facts

Morocco many (6) _______ and beautiful mosques

Turkey travelled there by (7) _______

good for local (8) _______

bought a beautiful Turkish (9) _______

(10) _______ visited Gujarati Textile (11) _______

great examples of (12) _______ embroidery

lots of wildlife in (13) _______ areas

saw increadible colouful (14) _______ and several (15) _______ spiders

PART II: PRONUNCIATION, VOCABULARY & GRAMMAR (35 points)


Questions 16-17: Choose the word (A, B, C or D) whose underlined part is pronounced differently from that
of the others. Write your answers into the box below.
16. A. advent B. invent C. decent D. percent
17. A. pursue B. hesitate C. comparison D. poster
18. A. ragged B. sacred C. dogged D. scared

164
Questions 18-20: Choose the word (A, B, C or D) that differs from the others in the position of the primary
stress. Write your answers into the box below.
19. A. questionaire B. introduce C. recommend D. concentrate
20. A. continuous B. scandalous C. malicious D. delicious
Questions 21-30: Choose the best option (A, B, C or D) to complete each of the following sentences.
Write your answers into the box below.
21. I’d like to make _______ for crashing your car. Let me pay for the repairs.
A. improments B. correctione C. amends D. adjustments
22. Thanks for lending me your umbrella; it really came in _______.
A. used B. handy C. handful D. needy
23. James never shows his emotions; no matter what happens, he always keeps a stiff upper _______.
A. mouth B. eye C. head D. lip
24. I got a new baseball _______ yesterday.
A. stick B. racket C. club D. bat
25. Eat your vegetables. They’ll _______ you good.
A. do B. make C. get D. help
26. Don’t drop your sweet wrapper on the floor, _______?
A. do you B. aren’t you C. will you D. won’t you
27. She may have missed the train, _______ she won’t arrive for another hour.
A. at the case B. all the case C. just in case D. in which case
28. I should like to rent a house, modern, comfortable, and _______ in a quiet position.
A. most of all B. above all C. first of all D. after all
29. I was disappointed that the restaurant had _______ flowers on the table.
A. false B. untrue C. artificial D. forged
30._______ write with your left hand when you broke your arm?
A. Did you have to B. Had you have to C. Needed you to D. Must you
Complete the sentences with the appropriate phrasal verbs from the box in their correct form. There are two
extra ones that you do not need to use. Write your answers into the box below.
put through come by die down take over bear up get into
look on join in give away call for carry out drop off
31. I didn’t think he would _______ so well in that situation.
32. Come and help me to carry the boxes! Don’t just stand there _______!
33. I watched a horror film on TV last night, but I can’t remember how it ended. I must have _______ before
the end.
34. Tomorrow, we will be _______ an experiment to test this theory.
35. Just ask them if you can play and I’m sure they’ll let you _______.
36. Do you think the wind has _______ enough for us to go sailing without any danger?
37. Environmentalists are _______ stricter controls on the use of leaded petrol.
38. The robber couldn’t explain how he _______ such a large amount of money when the police caught him.
39. Did you hear about the millionaire who _______ his entire fortune to charity?
40. Who is going to _______ the family business when Arstha’s father retires?
Questions 41-50: Complete the following passage by writing the correct form of the words given in brackets.
Write your answers into the box below.
RAIN MAKING

165
When it rains, it does not always pour. During a typical storm, a (41. COMPARE) _______ small amount of
the lock-up moisture in each cloud reaches the ground as rain. So the idea that human intervention - a rain
dance, perhaps - might encourage the sky to give up a little (42. ADD) _______ water has been around since
prehistoric times. More recently, would-be rain makers have used a more direct procedure - that of throwing
(43. VARY) _______ chemicals out of aero-planes in an effort to wring more rain from the clouds, a practice
known as “cloud seeding”.
Yet such techniques, which were first developed in the 1940s, are (44. NOTORIETY) _______ difficult to
evaluate. It is hard to (45. CERTAIN) _______, for example, how much rain would have fallen anyway. So,
despite much anecdotal evidence of the advantages of cloud seeding, which has led to its adoption in more than
40 countries around the world, as far as scientists are concerned, results are still (46. CONCLUSIVE)
_______. That could be about to change. For the past three years (47. RESEARCH) _______
have been carrying out the most extensive and (48. RIGOUR) _______ evaluation to date of a revolutionary
new technique that will substantially boost the volume of (49. RAIN) _______.
The preliminary (50. FIND) _______ of their experiments indicate that solid evidence of the technique’s
effectiveness is now within the scientists’ grasp.
PART III:READING COMPREHENSION (25 points)
Questions 51-55: You are going to read a magazine article on space travel. Five paragraphs have been
removed from the article. Choose from the paragraphs A-G the one which fits each gap (51- 55). There is
one extra paragraph which you do not need to use. There is an example at the beginning (0). Write your
answers into the box below.
CHEAP ACCESS TO SPACE
Charles Conrad went to the moon with Apollo 12 and circled the Earth in Skylab. But from now on, he is
going to aim high for himself. His company, Universal Space Lines, hopes to produce a more economic rocket
that will be able to go in space again and again.
(0) _____ G _____
NASA, the U.S, government-owned space program, plans to develop such a rocket. However, the immediate
priority is missions to Mars, which will require different technology. So it is more likely that people outside
the NASA program will develop re-useable rocket design. Rick Tumlinson runs an independent organization
called the Space Frontier Foundation and firmly believes that it is time for business to get involved.
(51) __________
So Tumlinson is also in business to prove a point. Space is our destiny, he says, so why not get on with it a bit
more eagerly? To this end, the SFF is holding a conference in Los Angeles shortly, to be called Space: Open
for Business.
(52) __________
Another company, Kistler Aerospace, has similar plans; “Our goal is to become a delivery service to low Earth
orbit that will radically re-align the economics of doing business in space. Satellites will be our parcels: our
vehicles will be operated in repeated flights with air freights efficiency.” (53) __________
Their own view is that it is impossible for NASA, which is government-owned, to offer an “open frontier”.
This is not a matter of budgets or schedules, but of fundamental purpose and design. NASA is “elitist and
exclusive”, whereas the SFF believes in opportunities for everyone “a future of endlessly expanding new
choices”.
(54) __________

166
Of course, the ex-astronaut and businessman Charles Conrad agrees. “I’m trying to get affordable space
transportation up and operative so that everybody can enjoy space. And by the way, the Japanese are hard at
work building a space hotel.”
(55) __________
If he is right, mass space travel will have arrived by 2050 and space tourism will have become a viable
industry. More importantly, the human race will have made serious progress in crossing that final frontier.
A. Companies will always be looking for profit. For this reason, the SFF is not in favor of American
missions to Mars, claiming that there’s nothing in it for investors. At the same time, they do accept that these
missions could bring scientific benefits.
B. He sees the NASA program as a bit of a dinosaur. “25 years after the Wright Brothers, people could buy
a commercial plane ticket ... but many years after landing on the moon, we sat around watching old
astronauts on TV talking about the good old days.”
C. In 1997, the SFF ran a survey on the Internet, called “Cheap Access to Space”, where it asked
American taxpayers for their views on the U.S space program and on what American’s future priorities
should be in space transportation.
D. U.S government officials don’t see the future for space tourism. Here again, private companies may well
prove them wrong. David Ashford, director of Bristol Spaceplanes Limited, once said that space tourism
would begin ten years after people stopped laughing at the concept. Recently, he added this striking
comment: “people have stopped laughing.”
E. Charles Conrad is due to speak there. But his company is in fact only one of several that already have
blueprints for getting into space and back cheaply. Rotary is working on something that would be launched
like a rocket but return like a helicopter. Pioneer Rocket plane believes there could be a million dollar
market in delivering packages from one side of the planet to the other in an hour.
F. They would like to see “irreversible human settlement” in space as soon as possible and maintain that
this will only happen through free enterprise. “Building buildings and driving trucks is not what astronauts
should be doing; that’s what the private sector does.”
G. “Cheap” is an important word in space technology nowadays and re-useable rockets will be a key way
of controlling costs. They will deliver things to orbits, bring stuff back to Earth and then go up again,
perhaps with machinery for a space factory, or even carrying tourists.
Questions 56-65: Read the following passage and decide which answer A, B, C or D best fits each numbered
blank. Write your answers into the box below.
CRITICISM
It can take a long time to become successful in your chosen field, however (56) _______ you are. One thing
you have to be (57) _______ of is that you will face criticism along the way. The world is (58) _______ of
people who would rather say something negative than positive. If you’ve made up your (59) _______ to
achieve a certain goal, such as writing a novel, don’t let the negative criticism of others (60) _______ you from
reaching your target, and let constructive criticism have a positive effect on your work. If someone says you’re
totally (61) _______ talent, ignore them. That’s negative criticism. If, however, someone advises you to revise
your work and gives you good reasons for doing so, you should (62) _______ their suggestions carefully.
There are many film stars who were once out of work. There are many famous novelists who made a complete
(63) _______ of their first novel - or who didn’t, but had to (64) _______approaching hundreds of publishers
before they could get it published. Being successful does (65) _______ on luck, to a certain extent. But things
are more likely to turn out well if you preserve and stay positive.
56. A. talented B. invested C. mixed D. workable

167
57. A. alert B. cleaver C. intelligent D. aware
58. A. overflowing B. full C. filled D. packed
59. A. mind B. brain C. thought D. idea
60. A. cease B. remove C. avoid D. prevent
61. A. lacking B. short C. missing D. absent
62. A. think B. consider C. look round D. take
63. A. rubbish B. trash C. mess D. garbish
64. A. put off B. bank on C. keep on D. drop in on
65. A. require B. depend C. need D. trust
Questions 66-75: Read the passage carefully, then fill in each blank with ONE suitable word. Write your
answers into the box below.
Maybe you recycle cans, glass, and paper. Do you know that nature recycles, too? One of the things nature
(66) _______ is water. Water goes from oceans, lakes, and rivers into the air. Water falls from the air as (67)
_______ or snow. Rain and snow eventually find their way back to the oceans. Nature’s recycling program for
water is (68) _______ the water cycle.
The water cycle has four stages: storage, evaporation, precipitation, and runoff. Water on Earth gets stored
in oceans, lakes, rivers, ice, and even underground. Water goes from storage into the atmosphere by a (69)____
called evaporation. When water evaporates, it changes from a liquid (70) _______ a gas, called water vapor.
Water vapor goes up into (71) _______ atmosphere. Water returns to the Earth as precipitation in rain or snow
by changing into drops of water when the air (72) _______ cold enough. Clouds are collections of water
droplets. Most precipitation (73) _______ into the oceans and goes right back into storage.
Water that falls on land always flows from (74) _______ places to lower ones. This flow is called runoff.
Water from land flows into streams. Streams join (75) _______ to make rivers and eventually the water flows
into storage in the oceans. Then the water cycle starts all over again.
PART IV: WRITING (25 points)
Questions 76-80: Finish each of the following sentences in such a way that it means the same as the
sentence printed before it.
76. The teachers agreed to introduce the new methods.
→ There was _________________________________________________________________________
77. There was no need for you to have gone to all that trouble.
→ You _____________________________________________________________________________
78. It’s sad, but unemployment is unlikely to go down this year.
→ Sad ______________________________________________________________________________
79. You must never mention this secret to him.
→ Under ____________________________________________________________________________
80. If you were in the situation I’m in, you’d feel the same.
→ If you put _________________________________________________________________________
Questions 81-85: Complete the second sentence so that it has a similar meaning to the first sentence, using
the word given. Do not change the word given. You must use between two and five words, including the
word given.
81. I was about to leave when she turned up. (POINT)
→ I was ____________________________________ when she turned up.
82. I wasn’t expecting my colleagues to organise a farewell party on my last day at the company. (TAKEN)
→ I _____________________ when my colleagues organised a farewell party on my last day at the company.

168
83. Considering that Luke is so young, you must admit he’s making excellent progress as a musician.
(ACOOUNT)
→ If you ____________________________________ young Luke is, you must admit he’s making excellent
progress as a musician.
84. Peter was in trouble with his boss because he didn’t finish an important project by the deadline. (HOT)
→ Peter was ____________________________________ because he didn’t finish an important project by the
deadline.
85. You should punish him severely so that others will be afraid to behave as he did. (EXAMPLE)
→ You should ____________________________________ so that others will be afraid to behave as he did.
Questions 86-100: Do you agree or disagree with the following statement?
“Fast food is becoming a part of our daily life. Some people argue that fast food has negative effects on our
lifestyle and diet.”
Write about 250 words to support your opinion (and do not include your personal information).
______________________________________________________________________________________
______________________________________________________________________________________
______________________________________________________________________________________
______________________________________________________________________________________
______________________________________________________________________________________
______________________________________________________________________________________

169
ĐÁP ÁN CHI TIẾT
ĐỀ SỐ 1: 2018-2019
SECTION I: LISTENING (3.0 points)
Part 1.
1. B 2. A 3. B 4 . C 5. B 6. A
Part 2.
7. 13th 8. Lords 9. church 10. King Henry II 11. hiding places
th
12. lunch 13. 12 14. ceilings 15. five bridges
SECTION II: LEXICO - GRAMMAR (7.0 points)
Part 1:
16. A
Causative form have sth done
17. A
thank to (prep) nhờ vào
18. A
Preposition + Noun / noun phrase
19. B 20. C 21. D 22. C 23. C 24. A
25. D
Chill out = become quiet or calm, especially after a state of agitation
26. B 27. A 28. C
29. C
insist (v.) on doing sth = nhất quyết làm gì
30. D
remedial (adj.) = mang tính chữa trị, sửa chữa, để khắc phục (remedial actions/measures = các hành động/ biện
pháp nhằm sửa chữa)
31. D
increase (v.) = tăng decrease = giảm export (v.) sth = xuất khẩu cái gì; raise (v.) sth = tăng, nâng cái gì lên
32. B
faultless (adj.) - perfect = không có lỗi, hoàn hảo
imperfect = không hoàn hảo;
worthless (adj,) = không có giá trị, vô dụng;
unmarked (adj.) = không được đánh dấu, không có biển hiệu
33. C
make sense = có lý, có nghĩa hiểu được
34. C
tear (v.) (tore/ tom) sth up = xé nát cái gì (Cuối cùng, tôi ghê tởm., xé nát tờ đơn, vặn vứt nó đi);
fill in a form = điền vào một mẫu đơn;
work sth out = tính toán cái gì;
put sth off = trì hoãn cái gì
35. B
supply (v.) sb with sth = cung cấp cho ai cái gì
Part 2: Questions from 36 to 45. (2 points - 0.2/ each)
36. These high-heeled shoes are unsuitable for such rough terrain.

170
37. Colds are caused by viruses, so in the absence of a virus, you can't catch a cold.
38. You’re too young. This film is inappropriate for children your age.
39. Scientists now think that we may have viruses in our bodies already.
40. In cold weather, for example, blood vessels in the nose get smaller to stop heat escaping.
Unfortunately, this also allows the cold virus to attack the nose or throat more easily.
41. Victoria Falls in Africa is one of the most spectacular sights in the world.
42. It’s also an incredible place for water sports.
43. We need you to provide an accurate description of the situation.
44. On arrival, it's the noise that makes the greatest impression.
45. The surrounding landscape is also well worth a visit.
Part 3: Questions from 46 to 50.(1 point - 0.2/ each)
46. Hung really wishes that he can take part in the game. → could
47. They’re going to get their house decorate for the May Day. → decorated
48. I used to sit next to a man who name is Aimed. → whose
49. I’d rather typing this letter than write it because it’s faster. → type
50. He drives too fast that no one likes to ride with him. → so
Part 4: Questions from 51 to 55.(1 point - 0.2/ each)
51. It’s about time you told him the truth!
52. The soccer player was ejected because he had done something that was against the rules.
53. Being a nurse is hard work, especially if you’re on call all the time.
54. You’ll have to work very hard if you want to catch up with the rest of your classmates.
55. I came across my lost earring while I was sweeping in the floor.
SECTION III: READING (6.0 points)
Part 1. Questions 56 - 65 (2 points - 0.2/ each)
56. B 57. B 58. D 59. A 60. B 61. C 62. D 63. B 64. C 65. A
Part 2. Questions 66 - 75 (2.0 points - 0.2/ each)
66. B 67. A 68. A 69. D 70. B 71. A 72. B 73. D 74. B 75. B
Part 3: Questions 76 - 85 (2.0 points - 0.2/ each)
SAFE CAMPING
76. and 77. into 78. forecast 79. case 80. site
81. sure 82. fire 83. out 84. be 85. animal
SECTION IV: WRITING (4.0 points)
Part 1: Questions from 86 to 90. (2 point- 0.4/ each).
86. At no time was the result of the match in doubt.
87. If I had known you were coming, I would have waited for you.
88. On picking up my pen, I found that the nib had broken.
89. Sally finally succeeded in getting a job.
90. Such was Fiona’s disappointment that she could not keep on working.
Part 2: Questions from 91 to 92.(2 points - 0.4/ each)
Complete the second sentence so that it has a similar meaning to the first sentence, using the word given.
Do not change the word given.
91. I haven’t made a decision to continue my study in a foreign country.
I haven’t made up my mind to continue my study in a foreign country.
92. I can’t make (any) sense of Doris and what she has done.

171
93. If I were you I would make a complaint about that horrible meal.
94. My neighbors and I get on well with together
95. I don’t have a good memory of people’s surnames.

ĐỀ SỐ 2 (ĐIỀU KIỆN) NĂM 2017-2018

A. Ngữ âm (0,1 điểm/1 câu) Đ. I. viết lại câu (0,1 điểm/câu)


I. 1.C 2.B 3.C 4.B 5.D 1... ..had met... .the day before/previous day
II. 1.C 2.B 3.A 4.D 5.A 2....having….
B. Từ vựng (0,1 điểm/ câu) 3…go out….
I. 1.A 2.B 3.C 4.D 5.D 6.C 7.D 4......who has/ makes...
8.A 9.B 10.D 11.B 12.D 13.B 5….has been moved....
14.C 15.B 6... .to have developed....
II. Cho dạng đúng của từ 7....has been recycled...
1.successul 2.professional 8. Neither....nor...
3.assistance 4.secretarial 5.ability 9...if/whether.. .had
6. Additional 7. Fluently 8.negotiation done... .before/previous day
9.requirement 10. applicants/applications 10…you didn’t...
III. Điền giới từ II. Dùng từ gợi ý viết thành câu (0,5đ)
1.without 2.to 3. From 4.of 5.on 1. were doing their...at...
6.into 7.at/on/to 8.by 9.about 10.on 2. haved used/ have been using...
IV. Từ gần nghĩa 3. ...a/the/their..
1.B 2.A 3.A 4.B 5.C 4…was/ (is) disappointed.. .failed/had/has failed
C. Đọc hiểu (0,1 điểm/câu) 5....used....for/in....?
I. 1.in 2.what 3.were 4.had/needed III. Viết lại câu (0,2 đ/câu)
5.case 6. More 7. of 8.too 9. so 10.with 1. It’s interesting to...
II.1.B 2.A 3.C 4.B 5.A 6.C 7.C 2….needs improving/ to be improved....
8.D 9.C 10.C 3....being taken...by....
III. 1.F 2.T 3.F 4.T 5.T 4…If I were....I’d take...
5…had I solved... .when...

ĐỀ SỐ 3 (CHUYÊN): 2017-2018
Ngày 11/06/2017
A. Listening skill (20 sentence x 0,1 d)
Part 1: 1-melted 2-mass(-)producing 3-(some) popcorn 4-exploded 5-a restaurant
6-a competition 7-meat 8-counter 9- sales 10- rise
Part 2: 1.officer 5.F 8.F
2.information 6.T 9.T
3. appointment 4. instruction 7.NG 10.F
B. Grammar (30 sentence. X 0,1 pts)
I.1.A.to 2.B.catch on 3.B. adventurous 4.A. should have done 5.A.bargain
6.B. cultivated 7.A. cautious 8.D.hesitant 9.B.gave evidence 10.B.rebellious
II. 1. Income tax 2. Foos poisoning 3. Job satisfaction
4. parking meter 5. Fairy stories

172
III. 1. Suspension 2. Disinfect 3. Weakened 4. novelist 5. Cleanliness
IV. 1. Bad 2.appreciate 3.mark 4.head 5. Domestic
V. 1.will answer for 2.bear with 3.made off 4. hanging out 5.to keep up
C. Reading (25 sentences X 0.1 pts)
I. 1.B. giving 2.A.image 3.B.overwork 4. D. discrimination 5.A.genuinely
6. C. effective 7.B.conducted 8.D.staggering 9.D.short 10.A.clearly
II. 1. Adapt/adjust 2.cultures 3.norms/patterns 4.way 5.1ong
6.make7.out 8.sense/view 9.after 10.lasts/remains/stays
III. 1.D 2.E 3.F 4.A 5.C
D. Writing (10 sentences X 0,1 + essay = 1,5 pts)
I. 1.You can’t have enjoyed that party so long, can you?
You can’t have had a good time at that party so long, can you?
2. What I found surprising was her lack of confidence.
3. Were it not for her competence, she wouldn’t have got promotion/promoted.
4. There is absolutely no truth/not any truth in that rumor about the politician and the construction contract.
5. The passenger told the taxi driver to hurry up because/since/as he had to be at the station on time.
II. 1. Students are under no obligation (whatsoever) to attend the additional evening lectures.
2. All are eligible for (taking/to take part in) the contest regardless of race and sex.
3. whatever attractive advertisement they used, they couldn’t deceive customers;
4. It makes no difference to me if/Whether she writes to me/stays/keep in touch (or not)
5. I don’t know how/in what way he is going to come to terms with losing his job.
III. 1. Content: 1.0 pt: A provision of all main ideas and details as appropriate
2. Language: 0,25pt: A variety vocabularies and structures
3. Presentation: 0,25: Coherence. Cohesion and style

ĐỀ SỐ 4 (ĐIỀU KIỆN) : 2016-2017


Part A: (45 sentences X 0,1 pts) 16. dangerous 31. on
1. D 17. performers 32. of
2. C 18. living 33. from
3. C 19. incredibly 34. in
4. C 20. prove 35. up
5. D 21. safety 36. keep
6. C 22. tiniest 37.will live/live
7. B 23. careless 38. has taught/has been teaching
8. C 24. unlike 39. graduated
9. B 25. repeatedly 40. is usually kept
10. D 26. from 41. Have you ever been
11. A 27. over/on 42. went
12. C 28. after 43. broke
13. C 29. in 44. stole
14. A 30. into 45. were dancing
15. D 54. had 62. E
Part B:Reading ( 3 pts) 55. only 63. A
46. would/ should/could 56. B 64-65.Deforestation

173
47. suppose/imagine 57. D 66-67. overgrazing
48. for 58. I 68-69. overfishing
49. knew/remembered 59. F 70-71. Invasive species
50. rather/ sooner 60. C 72-73. intensive agriculture
51. will 61. H 74-75. urban development
52. hope/think/expect
53. wish

Part C. Writing (2, 5pts)


76. Did you enjoy yourself at the beach, Joe?
77. Because of being tired/ his tiredness, Mr Pike didn’t go to the meeting.
Because of Mr Pike’s tiredness, he didn’t go to the meeting.
78. Jane came up with the best solution.
79. The firm raised my salary after I had worked there for a year.
80. The assistant whom I spoke to was very helpful.
81. Whatever you say, I won’t believe you.
82. No sooner had you entered the building than the fire alarm went off.
83. Unless you keep your feet dry, you will catch a cold.
84. Had it not been for Nick’s advice, I would have gone bankrupt.
85. Robin suggested (that) Sarah (should) buy them that picture as a wedding present.
86. Our company is being supplied with furniture.
87. Nothing has been decided yet.
88. Nam asked Maria whether she was interested in singing karaoke.
89. Although the kids looked/had looked everywhere, they failed to find/couldn’t find the tennis ball.
90. The teacher warned us not to sit/against sitting/ in front of the computer for too long.
91. We wish we had done/could have done more sport when we were at school.
92. The children will have an exam soon, so they have to stay up late.
93. If I were you, I would/might/could/should/decide right now.
94. The teacher put off the theatre trip until the summer term.
95. The TV programme was too complicated for (any of) the children to understand/
96. A“ Beauty
97. D= three-week-old
98. B= rains
99. B= with
100. C= close

ĐỀ SỐ 5 (CHUYÊN): 2016-2017
I. Listening (20x 0,1 pt = 2, 0 pts)
a. 1.1,4 liters/litres 2. automatic 3. credit 4.Harries
5.Dr/Doctor 6. Alton 7. messages 8. reasonable
b. 1.B 2.B 3. B 4.A 5.B 6.C
7.C 8.A 9.B 10.C 11.A 12.A
II. Grammar (30 X 0,1= 3 pts)
a.1.A 2.A 3.B 4.D 5.D 6.A 7.D 8.D

174
b. 1.C 2.D 3.A 4.D 5.D
c. l. holiday makers 2.unforeseen/unforeseeable 3.beneficial
4. Unfamiliarity 5.undoubtedly/doubtlessly 6. Questionable
d. l. admission 2. Cheer 3.drinl 4.part 5. open
e. 1. Talk...into 2.came apart 3.gets...down
4. Had been paid off 5. Hold on 6. Getting.. .with
III. Reading (25 X 0,1= 2,5pts)
a. 1.industry 2.labour 3. Service 4. Decentralization 5. Entertainment
6.T 7.NG 8.T 9.F 10. Ng
b. 1.G 2.A 3.C 4.H 5.B 6.F 7.E
c. 1. A 2.D 3.B 4.A 5.C 6.A 7.B 8.A
IV. Writing (2,5 pts)
a. 1. She left the room without saying a word.
2. The keys were thought to have been found accidentally by the little girl.
3. No sooner had the kids dived straight into the pool than we arrived.
No sooner had we arrived than the kids dived straight into the pool.
4. John said (that) they were not doing enough to protect the environment from pollution those days.
5 . As a result of the recent drop in the sales, profits have declined.
b. 1. Jack and Rose never see eye to eye about bringing up I how to bring up their children.
2. I have (got/had) it on (good) authority that the local newspaper is going to shut down.
3. Not enough interest has been shown in the project to make it viable.
4. Our detectives determine/ are determined to get to the bottom of this case.
5. We were on the point of going to bed when the earthquake happened.
c. Writing: (1,5 pts)
1. Content: 50% of total provision of all main ideas and details as appropriate
2. Language: 30% A variety vocabularies and structures
3. Presentation: 20% Coherence. Cohesion and style

ĐỀ SỐ 6 (ĐỀ THI THỬ CHUYÊN): 2016-2017


PART A: 10 PTS
I. 1.C 2.D 3.A 4.B 5.B
II. 1.D 2.A 3.B 4.C 5.B
PART B: I. 20 PTS, II = 5PTS; m=5PTS; IV=10PTS
I. 1.D 2.B 3. A 4.B 5.A 6.A 7.C 8.B 9.A 10.A
11. A 12. A 13.D 14.C 15.B 16.C 17. A 18.C 19.B 20.D
II. 1.central 2.accessible 3.1eisurely 4.unforgettable 5.breath-takinng
6.mountaineer (s) 7.resistant 8.challenging 9.imperceptibly 10. Splender/(our)
III. 1.to 2.off 3.about/of 4.out,for 5.about/for 6.to 7.to 8.of
9.between 10.at
IV. L1: took→taken L3: them→it L5: THAT→what L7: inconvincing→unconvincing
L7: attractively→ attractive L10: concerned→ are concerned L12: among→between
L13: productivity→products L13: one-side→one-sided L15: repeated→repeatedly
PART C: 1=10 PTS, II =10 PTS
I.1.C 2.D 3.B 4.C 5.C 6.B 7.C 8.D 9.D 10.C

175
II. 1. affected/influenced 2.used 3.how 4.(al)though 5.finding/change
6.late 7.take 8. improvement 9.making 10. longer
PART D: 1= 10PTS, II = 20 PTS
I.1. Many’s the time that I have made stupid mistakes like that
2. I’m not in a/the habit of sleeping in the afternoon.
3. Contrary to your thought/belief/opinion/what you think, fat people are not always jolly.
4. Not for another five years did the whole truth about the murder come out.
5. Erika blamed me for causing/ having caused the accident.
6. My grandfather didn’t have any recollection of phoning me last night.
7. Many customers restrictions within the EC have been done away with.
8. At the moment me/my buying a new car is out of the question.
9. The Rainbow disco is out of bounds to ss at the school.
10. When they broke the news, she did not turn a hair.
II. 1. Content: 10pts -a provision of all main ideas and details as appropriate
2. Language: 5pts- A variety vocabularies and structures
3. Presentation: 5pts- Coherence. Cohesion and style

ĐỀ SỐ 7 (ĐIỀU KIỆN): 08-05-2016


I. 1A 2C 3C 4B 5D
II. 6D 7A 8D 9A 10A 11C 12C 13C 14A 15D
16A 17B 18B 19D 20A 21C 22D 23D 24B 25A
III. 26. Choice 27. determination 28.visitors 29.disabled 30.legendary
31. protective 32.participants 33. Japanese 34.widespread 35. Stolen
IV. 36.had 37.wouldn’t have to 38.was driving 39. Lost 40. went
41.have gone 42.talking 43 .had happened 44.to learn 45.were given
V. 46. By 47.for 48.out of/in 49.about 50.at
51.by 52. into 53. with 54.by 55. to
56. which 57.plants 58.enjoy 59.their 60.on
61. national 62. Also 63. Famous 64.every 65. Must
66.A 67.C 68.C 69.A 70.D
71.C 72.B 73.A 74.D 75.C
76.D 77.C 78.B 79. A 80.A
81.A 82.D 83.B 84.D 85.C
86. I’d rather you didn’t play the music so loudly at night,
87. The grass isn’t dry enough for us to sit on.
88. The harder you work, the easier you’ll find it to get good marks.
89. “I’m going to visit China next year”, Paul said.
90. Millions of Christmas cards were sent last month.
91. If I hadn’t stayed up late last night, I wouldn’t have a terrible headache.
92. They told me not to live in that are because it was very dangerous.
93. Because the traffic was heavy/There was a lot of traffic, we were late for the meeting.
94. I wish you had given me a chance to tell you the truth.
95. As Jane hadn’t been invited to the party, she was annoyed.
96. My father encouraged me to try my best to study.

176
97. In spite of having very little money/In spite of the fact that they have very little money, they are happy.
98. we had our house painted last year.
99. I haven’t kept in touch with my best friend for several months
100. He objects to his appearance being laughed at.

ĐỀ SỐ 8 (CHUYÊN): 2015-2016
1. balanced 51.B 52.D 53.A 54. E 55. C 56. C
2. 20 times 57. A 58. B 59. B 60. C 61.D 62.A
3. five/5 63. C 64. D 65. C
4. carbonated 66. regular/daily
5. dairy products 67. shared
6. four/4 68. downloading
7. salt 69. accessed/collected
8. regular light meals 70. bulletin/message
9. avocado/pear 71. judgment/judgement
10. twice as much 72. posted/shared/updated
11. A 12.C 13. B 14. A 15. C 16. C 73. communicating
17. B 18. D 19. D 20. C 21. A 22. A 74. dangerous
23. D 24. D 25. D 26. D 27. A 28. B 7 5. strangers
29. D 30. A 76. The only thing (that/which)Neil forgot to pack
31 .undrinkable/impure was his toothbrush.
32. optimists 77. It came as a surprise to meet Peter to see so many
33. temporary foreigners at the party.
34. laboratory/lab 78. Jacqui and I were sitting on out own at the back of
35. envhonmentalist/conservationist the coach
36. sensational 79. Everything possible will be done to avoid disaster
37. Apparently 80. So exhausted were the runners that none of them
38. Strangely finished the race.
39. musicologists 81. Sarah would have not rung if she hadn’t been
40. available worried about us.
41. scientific 82. My friends had left the party so it was not worth
42. authenticity staying there.
43. preparations 83. Sue paid Terry back for being rude to her.
44. authoritative 84. If you are careless/not careful, you will get the
45. originals sack/be given the sack.
46. fell out 85. I’m looking for a job on a level with my abilities.
47. standing over
48. seized up
49. taken in
50. let off

ĐỀ SỐ 9 (ĐỀ THI THỬ CHUYÊN): 2014-2015


Part A:
I. 1.C 2.A 3.D 4.A 5.A

177
II. 1.B 2. A 3.B 4. ĐỀ SAI 5.D
PART B
I.1.A 2C 3B 4A 5B 6B 7A 8B 9B 10D 11B 12A 13C 14B 15D
II. 1B 2A 3B 4A 5A
III. 1.implication 2.Product 3.Inescapable 4.Catchy 5.Easily
6.Beneficial 7. Informative 8.Brighten 9.Appointed 10.Misleading
IV. 1 .Pegged down 2.Put aside 3. Live through 4.Talk into 5.Crop up
6.Meddle in 7.Hold back 8.Buzz around 9.Passby 10.Ripped off
V.1. nothing 2.take 3.go 4.make 5.middle
6. track 7.age 8.treat 9.best 10.bear
PART C
I. 1D 2C 3A 4B 5D 6A 7B 8D
9B 10A 11A 12D 13B 14C 15C 16D
II.1A 2C 3D 4B 5B 6A 7C 8D
PART D
1. I’m fed up with always getting the donkey work.
2. The thief did time the crimes he had committed.
3. My brother lent me his car for the weekend as a favor.
4. The painter experimented with a lot of different techniques.
5. A number of young artists.
6. Ronaldo greatly regretted not having celebrated the New year with his mother.

ĐỀ SỐ 10 (ĐỀ THI THỬ KHÔNG CHUYÊN): 2014-2015


Part A:
I.1.A 2D 3B 4B 5C
II. ID 2A 3D 4B 5C
Part B:
I.1A 2D 3A 4B 5D 6C 7D 8B 9A 10D 11D 12A 13D 14B 15C
II. 1.informative 2. communication 3.friendship 4.nervously 5.disabled
6. impolite 7.careful 8. building 9.compulsory 10.researchers
III. 1D 2D 3C 4C 5B 6D 7C 8B 9C 10A
IV. 1.leaving 2.has gone 3.does..mean 4.have never seen 5.1ooking
6.accounts 7.am annoyed 8.doesn’t listen 9.are being carried 10.to ensure
Part C:
I.1C 2B 3D 4C 5B 6D 7B 8A 9B 10D 11A 12A 13D 14C 15C
II.1.B 2A 3C 4A 5C
Part D:
1. The result of the trial exam will be announced in two more weeks.
2. She wishes she could run as fast as her friend.
3. It’s the second time you have been late this week.
4. Foreign tourists would rather walk than drive due to traffic in Vietnam.
5. John offered to take me home on his motorbike.
II. 1.Regular exercise would do wonders for the way you look at life.
2. It is a unique opportunity to see African wildlife in its natural environment.

178
3. When a meteor enters the Earth’s atmosphere, it makes a bright line in the sky.
4. By together children learn more from their parents and parents learn more about their children.

ĐỀ SỐ 11 (ĐỀ THI THỬ KHÔNG CHUYÊN): 2013-2014


Part A:
I. 1D 2A 3C 4B 5D
II. 1B 2A 3D 4C 5B
Part B:
I.1D 2B 3A 4C 5C 6D 7C 8B 9D 10 A 11C 12 A 13 C 14C 15D
II. 1 .dangerous 2.popularity 3.divers 4.breath 5.immediately
6. carefully 7.fitness 8.confident 9.pressure 10.safety
III. 1B 2A 3B 4D 5C 6D 7C 8C 9B 10C
Part C:
I.1C 2B 3D 4C 5B 6D 7B 8A 9B 10D 11A 12A 13D 14C 15C
II.1.C 2F 3E 4A G
Part D:
1. I’ve already made arrangements for my next holiday.
2. The strong wind prevented us from enjoying our walk along the seafront.
3. My mother warned me not to sit in front of the computer for too long.
4. Mary had no difficulty in passing her driving test.
5. He will be made to train very hard by the basketball coach,
6. I wish I had told you the truth.
7. I don’t know the name of the woman to whom I spoke on the telephone,
8. She tried to stay cheerful in spite of feeling sick I her sickness.
9. You must take advantage of this opportunity to visit London,
10. If people realized how important it is to conserve energy, they would do something about it.
II. 1. The traditional Christmas dinner consists of coast turkey and Christmas pudding.
2. The villagers never throw their rubbish in the streets so the village is always clean.
3. Unlike children who are flexible, adults often find it hard to change the way they speak.
4. The solar energy that gets to the Earth can not provide enough power for the world’s population.

ĐỀ SỐ 12 (ĐỀ THI THỬ CHUYÊN): 2013-2014


Part A:
I.1C 2A 3B 4B 5A 6D 7A 8A 9A 10B 11C 12A 13B
14C 15C 16C
II. 1.Parenthood 2. Unfamiliar 3. Youth 4.Appreciable 5. Observation
6. Intimately 7. Characteristics 8. Unsympathetic 9. Selfless 10.Humanly
III. L1: took→taken L3: what→which L6: THAT→what L7: inconvincing→unconvincing
L9: parents→parents are 10: what→which L11: bỏ such L12: among →between
L13: productivity→products L15: repeated→repeatedly
IV. 1. Took apart 2.brings in 3.held back 4.dropped onto 5.coming down
6. Put aside 7.called out
V. 1.love 2.common 3.near 4.respect 5.features
Part B:

179
I.A
1.poisonous 2.traditional 3 .perceptional 4.Associated with
5. aggressive 6.combination 7. Consist of
I. B
1T 2T 3F 4F 5F 6T 7T 8F
II.1A 2D 3C 4A 5A 6C 7C 8B 9D 10C 11D 12A 13A 14A 15D
Part C:
1. Harry has within an ace of winning the big race.
2. He is making quite a name for himself as pianist.
3. I’m sure this new film will appeal to young children.
4. She gave an implausible explanation.
5. Erika likened the situation at work to a family argument.

ĐỀ SỐ 13 (ĐIỀU KIỆN): 2013-2014


1B 2D 3A 4D 5A 6A 7A 8A 9D 10C
11C 12D 13D 14D 15B 16A 17D 18C 19C 20A
21D 22C 23A 24D 25A
26.collection 27. Proud 28. Care 29.beautiful 30. Inexpensive
31. Luckily 32. Hopeful 33.invention 34.friendly 35. decision
36. were 37. repaired 38. written 39. have never heard 40 .was writing
41. rang 42.1ent 43.bought 44. Realized 45.did not have
46. after 47. In 48.after 49. Down 50. to
51.at 52. For 53. For 54. Of 55.on
56. is 57. Transport 58. Fit 59.about 60. Instead
61. oil 62. Whereas 63. gets 64. either 65. ride
66. these 67.for 68.number 69.outside 70. Chosen
71. My car was scratched on the door last week.
72. I didn’t know the man who walked into my office yesterday.
73. The TV programme was too complicated for the children to understand.
74. If I were you, I would see a dentist about that tooth.
75. I have never eaten/had the better meal (than this one) before.
76. it’s too far to walk so I think I will catch the bus.
77. Jae had to go home from school because she had a headache.
78. A heavy storm swept through the village and many people were killed.
79. Ben said everything had been different the day before.
80. My elder sister likes to wear “Ao dai” whenever she goes abroad because she thinks it is the symbol of
Vietnam.

ĐỀ SỐ 14 (ĐỀ THI CHUYÊN): 2013-2014


I.1A 2A 3B 4C 5B 6A 7A 8C
9. German and Spanish 10.(tour)guide 11. World travel 12. 4 months
13. advertising 14. Journalists 15 .adventure holidays
16. hometown 17. Presenter 18. Conference
II. 1C 2. C 3. D 4. B 5. D 6. C 7. A 8. B 9. C 10. D 11. B 12. D 13. A

180
14. B 15. A 16. B 17. A 18. D 19. C 20. C 21. C 22. A 23. D 24.B
25. D 26.out 27. around/about 28.on 29. into 30. in 31. at 32. deplored
33. dreaded 34. regretted 35. mourned 36. offended 37. loathed
38. stressed 39. reproached 40-41-42. spot 43-44-45. mind 46-47-48. best
49-50-51. style 52.collection 53. pollution 54. destruction 55. demand 56. wood
57. research 58. developed 59. costs 60. treatment
61.A 62.C 63.A 64. D 65. D 66. B 67. C 68. B 69. D 70. D 71. C
72. B 73. D 74. B 75. A
76. John reminded Peter to give him a ring the following day/ the next day.
77. She suggested inviting Teddy to the party on Saturday.
78. My mother warned the boys to be careful and not to go too near the edge of the cliff.
79. No sooner had the plane left the airport than a big problem occurred.
80. As far as payment is concerned, most major credit cards are acceptable.
81. Lonie gratulated Kate on having passed/ passing the exam.
82. There is no access to the village in winter because of the snow.
83. I can’t work out the answer without a calculator.
84. John made a mess of the composition because he was tired.
85. Your son seems to be incapable of concentrating on anything.

ĐỀ SỐ 15 (ĐIỀU KIỆN): 2012-2013

1.B 2.B 3.D 4.C 5.A 6.B 7.C 8.B 9.A 10.C 11.A 12.B 13.C 14.D 15.D
16.B 17.A 18.C
19. A 20.B 21.woke up 22. was raining 23.came 24. wasting 25. Watching
26. will come/am coming 27.take 28.was getting 29. driving 30. to work 31. watching
32. turned 33. is getting 34. stopped 35. making 36. illness
37.enjoyable 38. talkative 39. dissatisfied 40. importance 41. poisonous 42.
hopeless
43. surprising 44. unpolluted 45. scientific 46. after 47. for 48. off 49.on
50-51. on/in 52.at 53. on 54. across 55. with
56.D 57.B 58.A 59.B 60.C 61.D 62.A 63.C 64.C 65.E 66.C 67.A 68.F 69.B 70.D
71.C 72.C 73.B
74.B 75. C 76.D 77.C 78.A 79.A 80. B 81. C. hearing 82. C.so 83. D. started
84. B. could help 85. B. many
86. We left home at about 9.15 am to go to the station.
87. The train arrived at Vitoria station just before 10.45 am, and then we caught a bus to the London Eye.
88. When we arrived at the London Eye, we only had to queue for about 10 minutes.
89. I was really scared about going on the cabin, but I still wen on.
90. It was really nice and I would love to go on it again.
91. The wheel turned very slowly, and it took about half an hour to go right round,
92. When we came off the cabin, we went on a river trip from the Embankment.
93. We could see Big ben and the House of Parliament from the boat.
94. The weather was so lovely and hot so we had a picnic in the park.
95. We caught a bus to Vitoria station and arrived back in Crawley about 8.00 pm

181
96. I felt it was a long day walking I a long walking day but I really enjoyed it.
97. I don’t know the name of the girl who helped me yesterday.
98. He takes after his father in many ways.
99. These rooms haven’t been used for a long time.
100. The receptionist required me to bring back in half an hour.

1.B 18. A 33. get..of 49. A. located


2.D 19. A 34.run into 50. A. buildup
3.A 20.B 35. take to 51. B. performance
4.B 1.C 36. sit through 52. C. use
5.A 22. B 37. breakup 53. D. place
6.C 23. Leadership 38. let...down 54. C. goods
7.A 24. unproductive 39. control 55. E
8.B 25. unmanageable 40. natural 56. A
9.D 26. engineering 41. ring 57.F
10.A 27. remembrance 42. areas 58.B
11.A 28. ungrammatical 43. informed/warmed 59.D
12.C 29. unattended 44. leave 60.C
13.A 30. spacious 45. damage 61. A → cheap they are
14.B 31. come out 46. help/aid 62. B → depends on
15.C 32. account for 47. B. version 63. B → are attracted
16. A 48. D. check 64. B → useless
17. A 65.C → X
66. We’d rather you didn’t smoke/you stopped smoking
67. She didn’t deserve to be treated like that
68. There’s no point in complaining about him.
69. With the exception of David, everyone else at the meeting was a Party Member.
70. Get in touch with the student Service Office if you have any further problems.
71 -72. I think you have held the wrong end of the stick.
73-74. Our little son is really excited at the thought of going to Disney World
75-76. Despite her beauty and intelligence/ Despite the fact that she was beautiful and intelligent, she didn’t
win the Miss World Competition.
77-78. we must come to terms with the fact that we have lost the golden opportunity.
79-80. You neglected telling me/ neglected to tell me you were going to be away for a whole month.

ĐỀ SỐ 17 (ĐỀ THI THỬ KHÔNG CHUYÊN): 2011-2012


I.(10 pts)
a.1.B 2C 3B 4A 5B
b.1.B 2 A 3D 4C 5B
II. a =20pts, b=10 pts, c=10 pts, d=10 pts.
a.1b 2a 3c 4b 5d 6b 7a 8a 9c 10d
11a 12d 13b 14d 15b 16d 17a 18b 19b 20a
b. 1.retired 2.has prepared/has been preparing 3.is going to spend/is spending
4.have wanted 5.haven’thad 6. were 7.bought

182
8.has converted/has been converting 9.will take 10. Are going to spend/ are spending
c. 1. Was→were 2.neither have→have neither 3.both→x 4. Are→is
5.Mai plays→does Mai play 6.for→to 7.situating→situated
8.wakes up me→wakes me up 9.do not satisfy→am not satisfied 10.productions→products
d. 1. Knowledgeable 2. Deepen 3. disabled 4. Solar 5. Unlimited
6. Extraordinary 7. existence 8. Naturalists 9.protection 10. Conservation
III. a= 10 pts, b=8 pts
a.1. began/started/appeared 2. was 3.made 4.Because/As/Since 5.later
6. wearing 7.make 8. wore 9.didn’t 10. Like
b.1.T 2.T 3.NI 4.T 5.T 6.NI 7.NI 8.T
IV. Writing (22pts)
1. His mother wanted to know whether he was leaving that day or the following/the next day.
2. The boy said that he was addicted to the Internet and he was chatting with his aunt then.
3. He has designed/has been designing clothes for more than twenty years.
4. A new style of jeans has just been introduced.
5. Anne is looking forward to meeting her aunt, (soon)
6. I wish I could speak English perfectly.
7. If I had a car, I would/could take you to the airport.
8. The weather was so bad that we postponed the picnic.
9. This exercise must be done carefully.
10. He told me not to disturb him.
11. It is open to question (as to) whether Jones will get the job (or not).

ĐỀ SỐ 18 (ĐIỀU KIỆN): 2011-2012


1.B 14.C 27. went 40.T 53.C
2.B 15.B 28.were sitting 41.F 54. D
3.A 16. standing 29. to have 42.F 55.A
4.B 17. collecting 30. repaired 43 .F
5.C 18.collector 31. the 44.T
6.D 19. Success 32. that 45.F
7.B 20. unpolluted 33. for 46.C
8.A 21. To go 34. members 47.C
9.D 22. Visit 35. in 48.B
10. A 23. was listening 36. on 49.D
11.A 24. rang 37.people 50.B
12.C 25. are playing 38.T 51.D
13 A 26. have worked/have been working 39.T 52. B
56. Hyde Park is a good place for camping in fine weather.
57. Jane promised to keep in touch with us while she is in Australia.
58. What aspect of learning English do you find most difficult ?
59. If you cut down the big frees in the forests, there will be floods every year.
60. The Internet, which is fast and convenient is a way to get into update.
61. Thanks to the TV, people can enjoy interesting programmes and get latest information in an expensive and
convenient way.

183
62. Keep your passport handy in case you are asked to show it at customs.
63. In the afternoon we go boating in the river and had a picnic on the river bank before going home in the late
evening.
64. On-in
65. To go-going
66. Me-I
67. Bored-boring
68. More - bỏ “more”
69. Makes- takes
70. To lock - locking
71. Mean-means
72. Hear-Heard
73. Ban-banning
74. A new bike is going to be given to me by my mother.
75. He reminded Jane of booking a table for the next afternoon.
76. Mike suggested going to the night club
77. John whom I have known for 3 years is one of my closest friends.
78. If you work too much, you will be tired.
79. How long ago did you start learning Japanese?
80. The chancellor has been reported to be very satisfied with his visit to Japan.

ĐỀ SỐ 19 (ĐỀ THI THỬ CHUYÊN): 2011-2012


I. Phonetics (10 pts)
a.1.B 2A 3D 4B 5B
b.1.A 2B 3A 4C 5A
II. VOCABULARY
a.15 pts
1A 2D 3C 4C 5A 6B 7C 8A 9B 10D
11C 12C 13B 14D 15C
b. (10 pts)
1. out 2. For 3. By 4. To 5.off
6. away 7. By 8. Off 9. Off 10. For
c. (5 pts)
1. (a) called up 2.(a) broke down 3. (a) back up 4. (a) call on 5. (a) picked up
(b) call...up (b) broke down (b) back up (b) called oh (b) pick up
d. (10 pts)
1. Picturesque 2. Handful 3.severity 4.disregard 5.exhaustive
6. Comparatively 7.unaccompanied 8. tension 9. impoverished 10.arguments
e. (10 pts)
1. dead → deaths 2. Destructions → destruction 3. Damaged → damaging
4. neither → either 5. Direct → directly 6. Themselves → itself
7. was → is 8. The result → a result 9. In → from 10. Initiating → initiated
III. Reading
a.(15 pts)

184
1C 2A 3D 4A 5B 6C 7D 8B 9D 10A 11B 12B 13A 14A 15C
b.(5 pts)
1C 2A 3C 4B 5A 6B 7C 8B 9A 10D
c.(10 pts)
1.teachers 2.schools 3 .better 4.that 5 .talks 6.studying/learning
7.happen 8.place 9.computers 10. Much
IV. Writing (10 pts)
1. Nothing but the complete/whole story would satisfy Tim/ was enough for Tim.
2. Ever since (he had) his accident Philip has been unable to make decisions.
3. Mrs Obama sends her apologies for not having attended/attending the meeting yesterday morning.
4. Thanks to skillful surgery, he survided the operation.
5. We were on the point of going to bed when the earthquake happened.
6. Tony bought Sally a necklace to make up for not having a holiday.
7. My father is going up the wall when he finds out that I have lost the car key.
8. If things go wrong, John, do not lose your head.
9. At the moment I’m preoccupied with my examination.
10. The introduction of the new currency has had a great impact on the economy.

ĐỀ SỐ 20 (ĐỀ THI THỬ KHÔNG CHUYÊN): 2011-2012


I.
1C 2C 3A 4C 5A
1B 2B 3C 4B 5C
II.a
1C 2D 3B 4A 5A 6B 7D 8A 9C 10B
11A 12D 13C 14C 15A 16B 17D 18A 19B 20C
II.b
1 .Do you 2. Has just left 3.bought 4.Did you pay 5.Are you looking
6.To call 7.To do 8.Will drive 9. Wants 10. haven’t been
II.c
1A 2B 3A 4D 5A 6C 7B 8A 9B 10C
II. d
1C 2A 3A 4B 5C
III.
1C 2B 3B 4A 5C 6B 7A 8A 9A 10C
1.F 2T 3T 4F 5T
IV.
1. Although I am fond of music, I can’t play any musical instruments.
2. Tom asked Mary if she was going to visit her aunt the next day.
3. The new stadium which will be opened next month can hold 90,000 people.
4. Mr Richard, to whom I spoke yesterday, is very interested in our plan.
5. If I weren’t so busy, I would write to my friends regularly.
6. The warmer it is getting, the better she feels.
7. I suggest that Peter look for another job.
8. He has stayed at the job for 5 years.

185
9. We enjoy lying on the beach all day.
10. If you don’t work hard, you will fail the exam.

ĐỀ SỐ 21 (ĐỀ THI CHUYÊN): 2011-2012


1.B 19.C 37. downloaded 55. but
2C 20.D 38. interviewee 56. seriously
3C 21. B 39. overdraft 57.precenting
4.D 22. B 40. capitalist 58. lessons
5.B 23. B 41. make off 59. D
6.B 24. B 42. keep...on 60.G
7.A 25. C 43. taking over 61.E
8.C 26. B 44. planned out 62.F/B
9.A 27.C 45. broke down 63.B/E
10.C 28.C 46. make out 64. A
11.C 29.B 47. sneezed out 65. D
12.C 30. B 48. made for/headed 66.F
13.C 31.B for 67.B
14.A 32.A 49. head up/take over 68.E
15.D 33. renovation 50. doing away 69.B
16. A 34. accountant 51. made 70.H
17.C 35. subscription 52. of 71.C 72.G
18.D 36. protective 53. same 73.D 74.F
54. has
75. William’s mother told him not to leave the house until she got back.
76. Mai mistook the man in front of her for her former teacher.
77. The older he got/ grew/ became, the more his memory fail.
78. If I had known you were still reading newspaper, I wouldn’t have thrown them away.
79. They are being made to study hard in order to pass the exam by their teacher.
80. Peter came up with the best solution.
81. He takes his wife for granted.
82. Get in touch/ in contact with the receptionists if you have any further problems.
83.I felt quite let down by this lecture.
84. It was hard to keep a straight face when she started to sing.
85. My sister took exception to being left out of the English club.

ĐỀ SỐ 22 (ĐỀ THI CHUYÊN): 2010-2011


23-06-2010
1.A 2.C 3.A 4.A 5.D 6.C 7.C 8.A 9.B 10. A 11.C 12.D 13.A 14.C 15.B
16.C 17.D 18.C 19.B 20.A 21.B 22.B 23.B 24.D 25.A 26.breathing 27. Consideration
28. harmful 29. importance 30. judgment 31. demanding32. steadily 33. difference 34. had never seen
35. have.. .been waiting/have waited 36. is increasing 37. was...doing 38. driving 39. would be
40. will be exhibited 41. had...taken 42. went 43 .will get 44.from 45.take 46.1ook
47. learning 48.understanding 49. without
50.A 51.D 52.C 53.D 54.C 55.A 56.B 57.A 58.A 59.D 60.A 61.B 62.B 63.C

186
64. You need to have your hair cut.
65. I wish I had taken my teacher’s advice.
66. Neither the fan nor the light is working. I’m getting angry.
67. The road is so bad that I can’t drive fast.
68. Laura invited Tom to have dinner with her.
69. All the switches must be turned off before leaving the workshop/ you leave the workshop.
70. He couldn’t stop in time to avoid the accident.
71. He spent two hours making up his mind which seeds to buy.
It took him two hours to make up his mind which seeds to buy.
72. We spoke in whispers so as not to wake the baby up.
73. Claire’s son came home late so often, which made her worried.
Claire’s son’s coming home late so often made her worried.
74. I’m afraid to say that we have run out of sugar.

ĐỀ SỐ 23 (ĐỀ THI THỬ): 2009-2010


1D 2D 3C 4A 5C 6C 7C 8D 9B 10C
11B 12A 13B 14B 15C 16D 17A 18D 19B 20A
21C 22B 23A 24D 25D 26C 27C 28B 29B 30A
31B 32D 33B 34A 35D 36A 37C 38D 39C 40B
41. skillful/skilled 42. Technological 43 .tasty 44.helpless 45.inventor
46. later 47.appearance 48. Electrician 49.successful 50.widely
51.one 52.a 53.any 54.a/each/per/every 55.are
56.how 57.what 58.from 59.If/should 60.Take
61.I stayed at/in your hotel from 24 April to/until 30 April in room 415.
62. On arriving home I realized I had left my book in the room.
63. I wonder/I am wondering if it has been found and handed in.
64. It is/ was a small, hardback book with a blue cover called “Gallion Reach” by H.M Tomlimson.
65. I’m very anxious to get the/my/this book back.
66. The book-is now out of print and it is/will be difficult to find in bookshops.
67. Also the/ this book was given to me by the author whose/and his signature in on the front of cover.
68.I enclose/I’m enclosing a cheque for $150 to cover the cost of posting/to post it back to me.
69. If, unfortunately, you don’t/can’t find the book, please return me the cheque to me.
70.I look forward to hearing from you soon.
71. I couldn’t understand either question.
72. No fewer than 20,000 people are thought to have attended the concert.
73. Neither of these pens writes properly.
74. Only after seeing Hamlet/I had seen Hamlet on the stage did I understand it.
75. We were unable to have our picnic because of/due to/ owing to the heavy rain.
76. Mr. Brown is known to be writing a new novel.
77. I will not help you unless you tidy your room/ if you don’t tidy your room.
78. He promised to give her $5 back the following week/the next week/the week after.
79. I wish I hadn’t told her the truth.
80. I’d rather you didn’t let the dog sit in front of the car.

187
ĐỀ SỐ 24 (ĐỀ THI CHUYÊN): 2009-2010
1.D 17.B 33.B 49.argumentative
2.B 18.D 34.C 50.extensiveness
3.C 19.C 35.D 51. favourite/best
4.B 20.D 36.landed/in 52/. Grows
5.C 21.B 37. meet up 53. containers
6.D 22.C 38.come/with 54. piece
7.B 23.D 39. reduced/to 55. covered
8.C 24.A 40.made/for 56. about
9.A 25.D 41. noticeable 57. sunlight
10.D 26.C 42. operator 58. easily
11.C 27.D 43. unreadable 59. E
12.C 28.B 44. performers 60. C
13.B 28.A 45. impoverished 61. D
14.D 30.C 46. notoriety 62. B
15.C 31.C 47. favoritism 63. G
16.D 32.C 48.environmentalists
64-65-66-67-68-69-70: Dựa vào đoạn văn trả lời các câu hỏi: phần này dễ rồi.
71. Mr Jones didn’t forget the date and neither did his wife/his wife didn’t, either.
72. He made no effort to conceal his dislike to me.
73. Sally came up with the best solution.
74. John said they moved there in 1998.
75. The magazine comes out every week.
76. John’s mother disapproves of his going to work.
77. You are so lucky to have come into all that money.
78. Paula always lost her temper when she made a mistake.
79. Everyone who spoke to the victim is under suspicion.
80. Don’t let his long words take you in/ Don’t be taken in by his long words.

ĐỀ SỐ 25 (ĐỀ THI OLYMPIC): 2008-2009


1C 2D 3B 4B 5D 6C 7A 8B 9A 10C
11D 12B 13D 14A 15 A 16D 17A 18D 19C 20B
21D 22C 23C 24B 25A
26. prolonged 27.widespread 28.throughout 29.developing 30.barely
31. cut-off 32. Farmland 33.failure 34. Flooding 35.pests
36. weigh → weight 37.dormantly → dormant 38. And → but 39. With → from
40. used → was used 41.senses → sense 42. Thousands → thousand
43. who that/ which 44. The → their 45. As → that
46. took it apart 47. Brings in 48. Put aside 49.runoutof
50. struck down 51. Called out 52. Letting off 53. Looking forward to
54. take it up 55. Take over 56. Being 57. On
58.for 59.fit 60. Qualified 61.therefore 62. Of 63.test
64.D 65.C 66.A 67.D 68.A 69.C 70.B
71. The car was thought to have been stolen by him.

188
72. It came as no surprise to US that he was/had been successful.
73. Contrary to what we had expected, they were seasoned travelers.
74. It wasn’t until he was nearly 30 that he passed his driving test.
75. Had it not been for his help, we would all have died.
76. The scientists are/have been shocked to find out that more than one billion species will be lost by 2050
77. There were far fewer people at the meeting than I had expected.
78. It is not worth trying to explain everything to him.
79. Without his father’s interference, everything would have gone smoothly yesterday.
80. He is the most likely person to succeed in solving the problem.

ĐỀ SỐ 26: 2007-2008
1.D 2.D 3.B 4.A 5.D 6.C 7.A 8.C 9.D 10.C
11.C 12.A 13.B 14.D 15. A 16.D 17.D 18.C 19.B 20.C
21.A 22. A 23 .B 24.B 25.C 26.C
27. citizen 28. inhabitants 29. Unemployed 30. Timetable
31. uncomfortable 32. Receipt 33. Inhuman 34. Immobilize/immobilize
35. troublesome 36. Environmentalists 37. Against 38.of
39. by 40. To 41. Into 42. Under
43. of 44. With 45. Did not realize 46. Was playing
47. asked 48. Warned 49. Would eventually end 50.surprised
51. before 52. Physical 53. Doctor 54. World 55.of
56. them 57. Orbit 58. Cabin 59.like 60. Feel
61.D 62.D 63.A 64.A 65.B
66.C 67.B 68.C 69.C 70.A
71-72.A-was delayed 73-74. D-X 75-76.D-for 77-78.C-as
79-80.D-will be shipped out
81-82. Against everybody’s expectation, she lost.
83-84. You must apply yourself to your work more.
85-86.She stands a good chance of being elected.
87-88. The boss objected to his secretary (‘s) coming to work late.
The boss objected to the fact that his secretary came to work late.
89-90. It was such a great demand for tickets that people queued day and night.

TRƯỜNG THPT CHUYÊN NGUYỄN HUỆ


Ngày thi: 13 tháng 3 năm 2016
- Đề thi gồm 100 câu; mỗi câu = 1 điểm
- Điểm cuối cùng = 10 điểm; làm tròn đến 0.25
(ví dụ: 87/100 = 8.75; 92/100 = 9.25; 76/100 = 7.75)

PART ONE: PHONETICS – GRAMMAR & VOCABULARY (55 pts)


I. Questions 1-5: Choose the word whose underlined part is pronounced differently from that of the
others by circling A, B, C or D and write your answers into the box below. (5.0 pts)
1. B 2. C 3. C 4. A 5. D
II. Questions 6-25: Circle the best option (A, B, C or D) to complete each of the following sentences and
write your answers into the box below. (20 pts)

189
6. D 7. A 8. B 9. C 10. C
11. D 12. B 13. A 14. A 15. D
16. B 17. A 18. A 19. B 20. B
21. C 22. B 23. C 24. D 25. A
III. Questions 26-35: Use the correct form of the words in brackets to complete sentences and write your
answers into the box below. (10 pts)
26. popularity 27. logically 28. insufficient 29. misunderstood 30. preservation
31. traditionally 32. poverty 33. healthier 34. dishonest 35. Deforestation
IV. Questions 36-45: Put the verbs in brackets into the correct form or tense and write your answers into
the box below. (10 pts)
36. has worn / has been wearing 37. moved 38. have been killed 39. would die 40. being stopped
41. stealing /having stolen 42. are going to have 43. repaired 44. will have completed 45. cutting
V. Questions 46-55: Fill each gap with a suitable preposition to complete each of the sentences. (10 pts)
46. to 47. on 48. for 49. by 50. on
51.with 52. for 53. on 54. from 55. off
PART TWO: READING (25 pts)
I. Questions 56-65: Read the passage carefully, then fill in each blank with ONE suitable word and write
your answers into the box below. (10 pts)
56. of 57. being 58. however 59. which/that 60. rather
61. based 62. ways/types/kinds 63. best 64. what 65. least/times
II. Questions 66-75: Read the text below and decide which answer A, B, C or D best fits each numbered
blank and write your answers into the box below. (10 pts)
66. B 67. C 68. C 69. B 70. C
71. B 72. D 73. D 74. A 75. A
III. Questions 76-80: Read the following passage and choose the correct answer (A, B, C or D). Write
your answers into the box below. (5pts)
76. A 77. D 78. B 79. D 80. C
PART THREE: WRITING (20 pts)
I. Questions 81-85: Choose the correct sentence (A, B, C or D) that is made from the words given. Write
your answers into the box below. (5pts)
81. A 82. B 83. D 84. A 85. A
II. Questions 86-90: Choose the underlined part (A, B, C or D) that needs correcting and CORRECT IT.
(5 pts)
86. A → Asked about 87. D → like 88. A → is 89. C → tastes 90. B → have been proposed
III. Questions 91-95: Rewrite each of the following sentences in such a way that it means the same as the
sentence printed before it. Use the bold words given in brackets. Do not alter the given words in any way.
(5 pts)
91. We spent three hours finding a room for the night./We spent three hours looking for a room.
92. Would you prefer me to stay with you during the holidays?
93. The English test was so difficult that I could not do it well.
94. The police accused Jim of having stolen the money.
95. His illness made it impossible for him to work effectively. / His illness made him impossible to work
effectively.
IV. Questions 96-100: Rewrite each of the following sentences in such a way that it means the same as
the sentence printed before it. Use the bold words given in brackets. Do not alter the given words in any
way. (5 pts)
96. The nearer New Year comes, the busier people are.
97. Neither Peter nor John likes tea.
98. Unless Peter is invited to the party, Mary will not (won’t) come.
99. Although the weather was bad, we went swimming.
100. He apologies for not coming / having come to the meeting yesterday.

190
Ngày thi: ___ tháng 4 năm 2016
PART ONE: PHONETICS – GRAMMAR & VOCABULARY (55 pts)
I. Questions 1-5: Choose the word whose underlined part is pronounced differently from that of the
others by circling A, B, C or D and write your answers into the box below. (5.0 pts)
1. B 2. A 3. C 4. A 5. D
II. Questions 6-25: Circle the best option (A, B, C or D) to complete each of the following sentences and
write your answers into the box below. (20 pts)
6. D 7. B 8. C 9. B 10. B
11. C 12. A 13. D 14. C 15. B
16. D 17. A 18. A 19. C 20. A
21. B 22. C 23. C 24. A 25. D
III. Questions 26-35: Use the correct form of the words in brackets to complete sentences and write your
answers into the box below. (10 pts)
26. surprising 27. explanation 28. well 29. success 30. beautiful
31. colourful/colorful 32. exist 33. replaced 34. Probably 35. ending
IV. Questions 36-45: Put the verbs in brackets into the correct form or tense and write your answers into
the box below. (10 pts)
36. turned 37. is getting 38. reading 39. learning 40. having
41. be repaired 42. hadn’t had to 43. wouldn’t have caught 44. took 45. are held
V. Questions 46-55: Fill each gap with a suitable preposition to complete each of the sentences. (10 pts)
46. for 47. with 48. off 49. of 50. on
51. without 52. on 53. until 54. at 55. of
PART TWO: READING (25 pts)
I. Questions 56-65: Choose the words in the box to fill in each blank in the following passage. (10 pts)
56. like 57. grow 58. heat 59. from 60. in
61. enough 62. like 63. it 64. into 65. but
II. Questions 66-75: Read the text below and decide which answer A, B, C or D best fits each numbered
blank and write your answers into the box below. (10 pts)
66. A 67. B 68. A 69. C 70. A
71. B 72. A 73. C 74. D 75. C
III. Questions 76-80: Read the following passage and choose the correct answer (A, B, C or D). Write
your answers into the box below. (5pts)
76. D 77. A 78. C 79. A 80. C
PART THREE: WRITING (20 pts)
I. Questions 81-85:Choose A, B, C or D that best fits the meaning of the given sentence. (5pts)
81. A 82. A 83. C 84. D 85. B
II. Questions 86-90: Choose the underlined part (A, B, C or D) that needs correcting. (5 pts)
86. B 87. A 88. D 89.A 90. A
III. Questions 91-95: Rewrite each of the following sentences in such a way that it means the same as the
sentence printed before it. Use the bold words given in brackets. Do not alter the given words in any way.
(5 pts)
91. It was such a great demand for tickets that people queued day and night.
92. The last time I was her was two years ago.
93. On account of his mother’s being there, they said nothing about it/On account the fact that this mother
was being there, they said nothing about it.
94. Had his aunt not died and left/given him (a legacy of) $10.000, he would not have been able to buy the
house he wanted.
95. It looked as if/ as though the house had been unoccupied for several months.
IV. Questions 96-100: Rewrite each of the following sentences in such a way that it means the same as
the sentence printed before it. Use the bold words given in brackets. Do not alter the given words in any
way. (5 pts)

191
96. Despite her severe disability/Despite the fact that Judy was severely disabled, Judy participated in many
sports.
97. I apologize for being rude/having been rude to you yesterday.
98. It is a fact that she has a beautiful voice.
99. She thought Rome is even more boring that Paris (is).
100. Al the switches must be turned off before leaving the workshop.

Ngày thi: 10 tháng 4 năm 2016


- Tổng điểm toàn bài: 100 điểm (100 câu x 1 điểm) → Điểm cuối cùng rút gọn = 10 điểm
- Từ câu 1 đến câu 85 x 1 điểm = 85 điểm; từ câu 85-100: bài viết luận = 15 điểm
- Chú ý: Khi cộng điểm toàn bài, giám khảo chấm làm tròn điểm đến 0,25/0,5/0,75
(lẻ 0,1 – 0,2 làm tròn thành 0,25; lẻ 0,3 – 0,4 làm tròn thành 0,5; lẻ 0,6 – 0,7 làm tròn thành 0,75)
PART I: LISTENING (15 points)
1. (a/the) head librarian 2. library assistant 3. 8.30 a.m
4. 4.30 p.m 5. second floor / 2nd floor 6. third floor / 3rd floor
Questions 7-11:
7. F 8. T 9. F 10. F 11. T
Questions 12-15:
12. Story Time 13. Reference Room 14. Saturday at 2:30 15. Lecture Series / Lectures
PART II: PRONUNCIATION, VOCABULARY & GRAMMAR (35 points)
16. B 17. A 18. C 19. D 20. B
Questions 21-30:
21. B 22. B 23. B 24. D 25. D
26. A 27. C 28. C 29. A 30. B
Questions 31-35:
31. APPLE 32. SCHOOL 33. FIGURES 34. LAY 35. RUN
Questions 36-45:
36. compatitive 37. knowledgeable 38. argumentative 39. traditional 40. impossible
41. excessive 42. beneficial 43. productive 44. resistant 45. preferable
Questions 46-50:
46. draw up 47. turn in 48. came into 49. turned down 50. take after
PART III:READING COMPREHENSION (25 points)
Questions 51-55:
51. C 52. E 53. A 54. F 55. D
Questions 56-65:
56. A 57. A 58. B 59. C 60. A
61. A 62. D 63. C 64. B 65. C
Questions 66-75:
66. about 67. end 68. result 69. an 70. who/that
71. single 72. tested 73. not 74. if 75. allowed
PART IV: WRITING (25 poitns)
Questions 76-80:
76. Only after/when all the guests had gone (home) could we be / were we able to relax
77. There has been a sharp increase/ rise in (the) house prices this year
78. Such was her fury that she threw the book across the room.
79. All dogs are thought to have evolved from wolves
80. The annual rainfall in/ for the northwest of Britain is higher/ greater than (that in) the southeast
Questions 81-85:
81. The two theories (appear to) have (got) nothing in common. /There is nothing in common between the
two theories.
82. I can’t/ won’t put up with his rude behaviour.
83. I have (considerable) difficulty (in) reading his hand writing.

192
84. He has neither money nor time to go on holiday.
85. A quiet holiday would do you good.
86-100. (In about 200 words) Write an essay about the advantages and disadvantages of studying in
groups.

Ngày thi: 08 tháng 05 năm 2016


Mỗi câu đúng được 1 điểm. Tổng điểm toàn bài = tổng số câu đúng / 10.
Lưu ý:- Khi cộng tổng điểm toàn bài, giám khảo chấm làm tròn điểm đến 0,25 (lẻ 0,1 - 0,2 làm tròn thành
0,25; lẻ 0,3 - 0,4 làm tròn thành 0,5;…)
- Nếu thí sinh có cách viết khác đáp án nhưng đúng ngữ pháp, ngữ nghĩa giám khảo thống nhất và cho
điểm.
PART I - PHONETICS, GRAMMAR AND VOCABULARY (55 pts)
I. Circle the word whose underlined part is pronounced differently from that of the others. (5 pt)
1. A 2. C 3. C 4. B 5. D
II. Circle the best option (A, B, C or D) to complete each of the following sentences. (20 pts)
6. D 7. A 8. D 9. A 10. A 11. C 12. C 13. C 14.A 15. D
16. A 17. B 18. B 19. D 20. A 21. C 22. D 23. D 24. B 25. A
III. Complete the sentences by using the correct form of the words in brackets. (10 pts)
26. choice 27. determination 28. visitors 29. disabled 30. legendary
31. protective 32. participants 33. Japanese 34. widespread 35. stolen
IV. Use the correct form of the verbs in brackets to complete the sentences. (10 pts)
36. had 37. wouldn’t have to 38. was driving 39. lost 40. went
41. have gone 42. talking 43. had happened 44. to learn 45. were given
V. Fill each gap with a suitable preposition to complete each of the sentences. (10 pts)
46. by 47. for 48. out of/ in 49. about 50. at
51. by 52. into 53. with 54. by 55. to
PART II - READING (25 pts)
I. Read the passage and fill each gap with ONE suitable word. (10 pts)
56. which 57. plants 58. enjoy 59. their 60. on
61. national 62. also 63. famous 64. every 65. must
II. Read the text below and circle the option (A, B, C or D) that best fits each gap. (10 pts)
66. A 67. C 68. C 69. A 70. D
71. C 72. B 73. A 74. D 75. C
III. Read the text below and circle the correct answer A, B, C or D. (5 pts)
76. D 77. C 78. B 79. A 80. A
PART III - WRITING (20 pts)
I. Circle the correct sentence (A, B, C or D) that is made from the words given. (6 pts)
81.A 82. D 83. B 84. D 85. C
II. Complete the second sentence so that it has a similar meaning to the first sentence. (8 pts)
86. I’d rather you didn’t play the music so loudly at night.
87. The grass isn’t dry enough for us to sit on.
88. The harder you work, the easier you will find it to get good marks.
89. “I am going to visit China next year.” Paul said
90. Millions of Christmas cards were sent last month.
91. If I hadn’t stayed up late last night, I wouldn’t have a terrible headache.
92. They told me not to live in that area because it was very dangerous.
93. Because the traffic was heavy/ there was a lot of traffic, we were late for the meeting.
94. I wish you had given me a chance to tell you the truth.
95. As Jane hadn't been invited to the party, she was annoyed.
III. Complete the second sentence so that it has a similar meaning to the first sentence, using the words
in brackets. Do not change the words given. (6 pts)
96. My father encouraged me to try my best to study.

193
97. Inspite of having very little money/ Inspite of the fact that they have very little money, they are happy. /
They are happy in spite of having very little money.
98. We had our house painted last year.
99. I haven’t kept in touch with my best friend for several months.
100. He objects to his appearance being laughed at.

Ngày thi: 08 tháng 05 năm 2016 (Đề số 2)


- Tổng điểm toàn bài: 100 điểm (100 câu x 1 điểm)  Điểm cuối cùng rút gọn = 10 điểm
- Từ câu 1 đến câu 85 x 1 điểm = 85 điểm; từ câu 85-100: bài viết luận = 15 điểm
- Chú ý: Khi cộng điểm toàn bài, giám khảo chấm làm tròn điểm đến 0,25/0,5/0,75
(lẻ 0,1 - 0,2 làm tròn thành 0,25; lẻ 0,3 - 0,4 làm tròn thành 0,5; lẻ 0,6 - 0,7 làm tròn thành 0,75)
PART I: LISTENING: (15pts)
You are going to listen (TWICE) to a man talking about his trips. Listen and complete the tasks.
Questions 1-5: Decide whether these statements are True (T) or False (F). Write your answers in the box
below. (5.0 poitns)
1. F (rug buyer) 2. F (Europe and Asia) 3. T 4. T 5. F (on the Internet and in books)
Questions 6-15: Complete the table. Write NO MORE THAN TWO WORD AND/OR A NUMBER for
each answer. Write your answers into the box below. (10 poitns)
6. historical buildings 7. boat 8. crafts 9. carpet 10. India
11. Museum 12. Indian silk 13. remote 14. birds 15. poisonous
PART II: PRONUNCIATION, VOCABULARY & GRAMMAR (35 points)
Questions 16-17: Choose the word (A, B, C or D) whose underlined part is pronounced differently from
that of the others. Write your answers into the box below.
Questions 18-20: Choose the word (A, B, C or D) that differs from the others in the position of the
primary stress. Write your answers into the box below. (5.0 poitns)
16. C 17. B 18. D 19. D 20. B
Questions 21-30: Choose the best option (A, B, C or D) to complete each of the following sentences. Write
your answers into the box below. (10 poitns)
21. C 22. B 23. D 24. D 25. A
26. C 27. D 28. B 29. C 30. A
Complete the sentences with the appropriate phrasal verbs from the box in their correct form. There are
two extra ones that you do not need to use. Write your answers into the box below. (10 poitns) Your
answers:
31. bear up 32. looking on 33. dropped off 34. carrying out 35. join in
36. died down 37. calling for 38. came by / had come by 39. gave away 40. take over
Questions 41-50: Complete the following passage by writing the correct form of the words given in
brackets. Write your answers into the box below. (10 poitns)
41. comparatively 42. additional 43. various 44. notoriously 45. ascertain
46. inconclusive 47. researchers48. rigorous 49. rainfall 50. findings
PART III:READING COMPREHENSION (25 points)
Questions 51-55: You are going to read a magazine article on space travel. Five paragraphs have been
removed from the article. Choose from the paragraphs A-G the one which fits each gap (51- 55). There is
one extra paragraph which you do not need to use. There is an example at the beginning (0). Write your
answers into the box below.
0. G
51. B 52. E 53. C 54. F 55. D
Questions 56-65: Read the following passage and decide which answer A, B, C or D best fits each
numbered blank. Write your answers into the box below.
56. A 57. D 58. B 59. A 60. D
61. A 62. B 63. C 64. C 65. B
Questions 66-75: Read the passage carefully, then fill in each blank with ONE suitable word. Write your
answers into the box below.

194
66. recycles 67. rain 68. called 69. process 70. into
71. the 72. gets / is 73. falls 74. high 75. together
PART IV: WRITING (25 poitns)
Questions 76-80: Finish each of the following sentences in such a way that it means the same as the
sentence printed before it. (5.0 poitns)
76. There was an agreement among the teachers to introduce the new methods.
77. You needn’t have gone to all that trouble. / You didn’t need to go to all that trouble.
78. Sad as it is, unemployment is unlikely to go down this year.
79. Under no circumstance must you mention this secret to him.
80. If you put yourself in my place/position/situation, you would feel the same.
Questions 81-85: Complete the second sentence so that it has a similar meaning to the first sentence,
using the word given. Do not change the word given. You must use between two and five words, including
the word given. (5.0 poitns)
81. I was on the point of leaving when she turned up.
82. I was taken by surprise when my colleagues organised a farewell party on my last day at the company.
83. If you take into account how young Luke is, you must admit he’s making excellent progress as a
musician.
84. Peter was in hot water because he didn’t finish an important project by the deadline.
85. You should make an example of him so that others will be afraid to behave as he did.
Questions 86-100 (15 poitns): Do you agree or disagree with the following statement?
“Fast food is becoming a part of our daily life. Some people argue that fast food has negative effects on our
lifestyle and diet.”
Write about 250 words to support your opinion (and do not include your personal information).

195

You might also like